Should you consider antibiotics for exacerbations of mild COPD?

Article Type
Changed
Tue, 07/14/2020 - 10:28
Display Headline
Should you consider antibiotics for exacerbations of mild COPD?

 

Practice changer

Consider antibiotics for patients with exacerbations of mild to moderate chronic obstructive pulmonary disease (COPD).1

Strength of recommendation

B: Based on a single well-done multicenter randomized controlled trial (RCT) with quality evidence.

Llor C, Moragas A, Hernández S, et al. Efficacy of antibiotic therapy for acute exacerbations of mild to moderate chronic obstructive pulmonary disease. Am J Respir Crit Care Med. 2012;186:716-723.

Illustrative case

A 45-year-old man with a history of mild COPD seeks treatment for worsening dyspnea and increased (nonpurulent) sputum production. He denies fever or chills. On exam, he has coarse breath sounds and scattered wheezes. Should you add antibiotics to his treatment?

COPD exacerbations—a worsening of symptoms beyond day-to-day variations that leads to a medication change—are part of the disease course and can accelerate lung function decline, decrease quality of life, and, when severe, increase mortality.2 Infections cause an estimated 50% to 70% of COPD exacerbations.2-4

Global Initiative for Chronic Obstructive Lung Disease (GOLD) guidelines recommend using antibiotics to treat exacerbations in patients with moderate or severe COPD who:
• have increased dyspnea, sputum volume, and sputum purulence;
• have 2 of these 3 symptoms if increased sputum purulence is one of the symptoms; or
• require mechanical ventilation.2

According to the GOLD guidelines, the choice of antibiotic should be based on local antibiograms; common options include amoxicillin, amoxicillin/clavulanate, azithromycin, and doxycycline.2 Although the GOLD guidelines cover use of antibiotics for COPD exacerbations, this recommendation is based on analyses of studies that focused on patients with moderate or severe COPD.2 There has been little research on using antibiotics for exacerbations of mild COPD.

STUDY SUMMARY: Using antibiotics often resolves symptoms

Llor et al1 conducted a multicenter, doubleblind placebo-controlled RCT to examine the effectiveness of antibiotic treatment for COPD exacerbations. Participants (ages ≥40 years) had mild to moderate COPD, defined as ≥10 pack-years of smoking, a forced expiratory volume in 1 second (FEV1) >50%, and a FEV1-to-forced vital capacity ratio <0.7. An exacerbation was defined as at least one of the following: increased dyspnea, increased sputum volume, or sputum purulence. Patients were randomized to receive amoxicillin/clavulanate 500/125 mg or placebo 3 times a day for 8 days. The primary endpoints were clinical cure (resolution of symptoms) and clinical success (resolution or improvement of symptoms) at Days 9 to 11 as determined by physician assessment. Secondary measures included cure and clinical success at Day 20 and time until next exacerbation. Patients were monitored for one year after the exacerbation.

There were 162 patients in the antibiotic group and 156 in the placebo group; the 2 groups were demographically similar. In each group, 4 patients withdrew consent and were removed from analysis. By the 9- to 11-day follow- up visit, 74.1% of patients in the antibiotic group had clinical cure, compared with 59.9% in the placebo group (P=.016; number needed to treat [NNT]=7). Clinical success also was significantly greater with antibiotics compared with placebo (90.5% vs 80.9%; P=.022).

By Day 11, symptoms resolved in 74.1% of patients in the antibiotic group, compared with 59.9% in the placebo group.The clinical cure rate at Day 20 also was significantly greater in patients on antibiotics compared with placebo (81.6% vs 67.8%; P=.006, NNT=7). During the one-year followup, 58% of patients in the antibiotic group and 73.2% of those in the placebo group experienced additional exacerbations. Time to next exacerbation was significantly longer in patients taking antibiotics (233 days vs 160 days; P=.015).

Can CRP level help determine who should—and shouldn’t—receive antibiotics? Previous studies have identified biomarkers, including C-reactive protein (CRP), that indicate COPD exacerbation, but have not linked them to clinical course.5-7 In this study, researchers measured CRP in patients receiving placebo to determine if this biomarker could predict clinical outcomes.

The researchers found that the clinical success rate among patients with a CRP <40 mg/L was 87.6%, while only 34.5% of patients with a CRP >40 mg/L experienced clinical success (sensitivity and specificity for clinical success at this cutoff was 0.655 and 0.876, respectively). This suggests that antibiotics might be appropriate for patients with an exacerbation of mild or moderate COPD who have a CRP >40 mg/L.

A total of 35 adverse events were reported: 23 in patients taking antibiotics and 12 among patients receiving placebo. Only 2 patients discontinued treatment due to adverse events in antibiotics group. Most of these reactions were mild gastrointestinal problems.

WHAT'S NEW?: Evidence supports antibiotics for mild to moderate COPD

Few placebo-controlled trials have addressed antibiotic use for exacerbations in patients with mild to moderate COPD.2,8,9 This study demonstrated that compared with placebo, symptom resolution and clinical success is greater with amoxicillin/clavulanate, and that antibiotic treatment also may increase time until next exacerbation.

 

 

The study also looked at the relationship of CRP and exacerbations in the placebo group. Higher spontaneous clinical cure rates were noted when the CRP was <40 mg/L.

CAVEATS: Effects of concomitant medications are unclear

In both the placebo and antibiotic groups, patients were taking other medications (including short-and long-acting beta-agonists, anticholinergics, theophyllines, and oral or inhaled corticosteroids). Roughly the same number of patients in each group took additional medications, but researchers did not conduct a subgroup analysis to see if patients treated with these medications responded differently than those who received antibiotics alone.

Clinical success also was significantly greater with antibiotics compared with placebo.GOLD guidelines already suggest antibiotics for exacerbations in patients with moderate COPD.2 In this study, 89% of patients met criteria for moderate COPD and 11% for mild COPD. Though the percentage of patients who had mild COPD was small, we believe the results of this study warrant consideration of antibiotic use in patients with mild disease. Local antibiograms may show increased resistance to amoxicillin/clavulanate; this study did not address the use of other antibiotics.

CHALLENGES TO IMPLEMENTATION: Antibiotic overuse may be a concern

With increased awareness of inappropriate antibiotic use, physicians might have concerns about antibiotic resistance developing as a result of using antibiotics for exacerbations of mild to moderate COPD.

ACKNOWLEDGEMENT
The PURLs Surveillance System was supported in part by Grant Number UL1RR024999 from the National Center For Research Resources, a Clinical Translational Science Award to the University of Chicago. The content is solely the responsibility of the authors and does not necessarily represent the official views of the National Center For Research Resources or the National Institutes of Health.

Files
References

 

1. Llor C, Moragas A, Hernández S, et al. Efficacy of antibiotic therapy for acute exacerbations of mild to moderate chronic obstructive pulmonary disease. Am J Respir Crit Care Med. 2012;186:716-723.

2. Global strategy for the diagnosis, management, and prevention of chronic obstructive pulmonary disease. January 2014. Global Initiative for Chronic Obstructive Lung Disease (GOLD) Web site. Available at: http://www.goldcopd.org/guidelines-globalstrategy-for-diagnosis-management.html. Accessed March 19, 2014.

3. Donaldson GC, Seemungal TA, Bhowmik A, et al. Relationship between exacerbation frequency and lung function decline in chronic obstructive pulmonary disease. Thorax. 2002;57:847-852.

4. Soler-Cataluña JJ, Martínez-García MA, Román Sánchez P, et al. Severe acute exacerbations and mortality in patients with chronic obstructive pulmonary disease. Thorax. 2005;60:925-931.

5. Vollenweider DJ, Jarrett H, Steurer-Stey CA, et al. Antibiotics for exacerbations of chronic obstructive pulmonary disease. Cochrane Database Syst Rev. 2012;12:CD010257.

6. Bartlett, JG, Sethi S. Management of infection in acute exacerbations of chronic obstructive pulmonary disease. In: Basow DS, ed. UpToDate [database online]. Available at: http://www.uptodate. com. Last updated March 27, 2012. Accessed January 2, 2013.

7. Lacoma A, Prat C, Andreo F, et al. Value of procalcitonin, C-reactive protein, and neopterin in exacerbations of chronic obstructive pulmonary disease. Int J Chron Obstruct Pulmon Dis. 2011;6:157-169.

8. Antonescu-Turcu AL, Tomic R. C-reactive protein and copeptin: prognostic predictors in chronic obstructive pulmonary disease exacerbations. Curr Opin Pulm Med. 2009;15:120-125.

9. Thomsen M, Ingebrigtsen TS, Marott JL, et al. Inflammatory biomarkers and exacerbations in chronic obstructive pulmonary disease. JAMA. 2013;309:2353-2361.

Article PDF
Author and Disclosure Information

 

Tanner Nissly, DO
Shailendra Prasad, MBBS, MPH

North Memorial Family Medicine Residency, University of Minnesota, Minneapolis

PURLs EDITOR
James J. Stevermer, MD
University of Missouri, Department of Family Medicine

Issue
The Journal of Family Practice - 63(4)
Publications
Topics
Page Number
E11-E13
Legacy Keywords
Tanner Nissly; DO; Shailendra Prasad; MBBS; MPH; COPD; chronic obstructive pulmonary disease; antibiotics; GOLD guidelines; Global Initiative for Chronic Obstructive Lung Disease
Sections
Files
Files
Author and Disclosure Information

 

Tanner Nissly, DO
Shailendra Prasad, MBBS, MPH

North Memorial Family Medicine Residency, University of Minnesota, Minneapolis

PURLs EDITOR
James J. Stevermer, MD
University of Missouri, Department of Family Medicine

Author and Disclosure Information

 

Tanner Nissly, DO
Shailendra Prasad, MBBS, MPH

North Memorial Family Medicine Residency, University of Minnesota, Minneapolis

PURLs EDITOR
James J. Stevermer, MD
University of Missouri, Department of Family Medicine

Article PDF
Article PDF

 

Practice changer

Consider antibiotics for patients with exacerbations of mild to moderate chronic obstructive pulmonary disease (COPD).1

Strength of recommendation

B: Based on a single well-done multicenter randomized controlled trial (RCT) with quality evidence.

Llor C, Moragas A, Hernández S, et al. Efficacy of antibiotic therapy for acute exacerbations of mild to moderate chronic obstructive pulmonary disease. Am J Respir Crit Care Med. 2012;186:716-723.

Illustrative case

A 45-year-old man with a history of mild COPD seeks treatment for worsening dyspnea and increased (nonpurulent) sputum production. He denies fever or chills. On exam, he has coarse breath sounds and scattered wheezes. Should you add antibiotics to his treatment?

COPD exacerbations—a worsening of symptoms beyond day-to-day variations that leads to a medication change—are part of the disease course and can accelerate lung function decline, decrease quality of life, and, when severe, increase mortality.2 Infections cause an estimated 50% to 70% of COPD exacerbations.2-4

Global Initiative for Chronic Obstructive Lung Disease (GOLD) guidelines recommend using antibiotics to treat exacerbations in patients with moderate or severe COPD who:
• have increased dyspnea, sputum volume, and sputum purulence;
• have 2 of these 3 symptoms if increased sputum purulence is one of the symptoms; or
• require mechanical ventilation.2

According to the GOLD guidelines, the choice of antibiotic should be based on local antibiograms; common options include amoxicillin, amoxicillin/clavulanate, azithromycin, and doxycycline.2 Although the GOLD guidelines cover use of antibiotics for COPD exacerbations, this recommendation is based on analyses of studies that focused on patients with moderate or severe COPD.2 There has been little research on using antibiotics for exacerbations of mild COPD.

STUDY SUMMARY: Using antibiotics often resolves symptoms

Llor et al1 conducted a multicenter, doubleblind placebo-controlled RCT to examine the effectiveness of antibiotic treatment for COPD exacerbations. Participants (ages ≥40 years) had mild to moderate COPD, defined as ≥10 pack-years of smoking, a forced expiratory volume in 1 second (FEV1) >50%, and a FEV1-to-forced vital capacity ratio <0.7. An exacerbation was defined as at least one of the following: increased dyspnea, increased sputum volume, or sputum purulence. Patients were randomized to receive amoxicillin/clavulanate 500/125 mg or placebo 3 times a day for 8 days. The primary endpoints were clinical cure (resolution of symptoms) and clinical success (resolution or improvement of symptoms) at Days 9 to 11 as determined by physician assessment. Secondary measures included cure and clinical success at Day 20 and time until next exacerbation. Patients were monitored for one year after the exacerbation.

There were 162 patients in the antibiotic group and 156 in the placebo group; the 2 groups were demographically similar. In each group, 4 patients withdrew consent and were removed from analysis. By the 9- to 11-day follow- up visit, 74.1% of patients in the antibiotic group had clinical cure, compared with 59.9% in the placebo group (P=.016; number needed to treat [NNT]=7). Clinical success also was significantly greater with antibiotics compared with placebo (90.5% vs 80.9%; P=.022).

By Day 11, symptoms resolved in 74.1% of patients in the antibiotic group, compared with 59.9% in the placebo group.The clinical cure rate at Day 20 also was significantly greater in patients on antibiotics compared with placebo (81.6% vs 67.8%; P=.006, NNT=7). During the one-year followup, 58% of patients in the antibiotic group and 73.2% of those in the placebo group experienced additional exacerbations. Time to next exacerbation was significantly longer in patients taking antibiotics (233 days vs 160 days; P=.015).

Can CRP level help determine who should—and shouldn’t—receive antibiotics? Previous studies have identified biomarkers, including C-reactive protein (CRP), that indicate COPD exacerbation, but have not linked them to clinical course.5-7 In this study, researchers measured CRP in patients receiving placebo to determine if this biomarker could predict clinical outcomes.

The researchers found that the clinical success rate among patients with a CRP <40 mg/L was 87.6%, while only 34.5% of patients with a CRP >40 mg/L experienced clinical success (sensitivity and specificity for clinical success at this cutoff was 0.655 and 0.876, respectively). This suggests that antibiotics might be appropriate for patients with an exacerbation of mild or moderate COPD who have a CRP >40 mg/L.

A total of 35 adverse events were reported: 23 in patients taking antibiotics and 12 among patients receiving placebo. Only 2 patients discontinued treatment due to adverse events in antibiotics group. Most of these reactions were mild gastrointestinal problems.

WHAT'S NEW?: Evidence supports antibiotics for mild to moderate COPD

Few placebo-controlled trials have addressed antibiotic use for exacerbations in patients with mild to moderate COPD.2,8,9 This study demonstrated that compared with placebo, symptom resolution and clinical success is greater with amoxicillin/clavulanate, and that antibiotic treatment also may increase time until next exacerbation.

 

 

The study also looked at the relationship of CRP and exacerbations in the placebo group. Higher spontaneous clinical cure rates were noted when the CRP was <40 mg/L.

CAVEATS: Effects of concomitant medications are unclear

In both the placebo and antibiotic groups, patients were taking other medications (including short-and long-acting beta-agonists, anticholinergics, theophyllines, and oral or inhaled corticosteroids). Roughly the same number of patients in each group took additional medications, but researchers did not conduct a subgroup analysis to see if patients treated with these medications responded differently than those who received antibiotics alone.

Clinical success also was significantly greater with antibiotics compared with placebo.GOLD guidelines already suggest antibiotics for exacerbations in patients with moderate COPD.2 In this study, 89% of patients met criteria for moderate COPD and 11% for mild COPD. Though the percentage of patients who had mild COPD was small, we believe the results of this study warrant consideration of antibiotic use in patients with mild disease. Local antibiograms may show increased resistance to amoxicillin/clavulanate; this study did not address the use of other antibiotics.

CHALLENGES TO IMPLEMENTATION: Antibiotic overuse may be a concern

With increased awareness of inappropriate antibiotic use, physicians might have concerns about antibiotic resistance developing as a result of using antibiotics for exacerbations of mild to moderate COPD.

ACKNOWLEDGEMENT
The PURLs Surveillance System was supported in part by Grant Number UL1RR024999 from the National Center For Research Resources, a Clinical Translational Science Award to the University of Chicago. The content is solely the responsibility of the authors and does not necessarily represent the official views of the National Center For Research Resources or the National Institutes of Health.

 

Practice changer

Consider antibiotics for patients with exacerbations of mild to moderate chronic obstructive pulmonary disease (COPD).1

Strength of recommendation

B: Based on a single well-done multicenter randomized controlled trial (RCT) with quality evidence.

Llor C, Moragas A, Hernández S, et al. Efficacy of antibiotic therapy for acute exacerbations of mild to moderate chronic obstructive pulmonary disease. Am J Respir Crit Care Med. 2012;186:716-723.

Illustrative case

A 45-year-old man with a history of mild COPD seeks treatment for worsening dyspnea and increased (nonpurulent) sputum production. He denies fever or chills. On exam, he has coarse breath sounds and scattered wheezes. Should you add antibiotics to his treatment?

COPD exacerbations—a worsening of symptoms beyond day-to-day variations that leads to a medication change—are part of the disease course and can accelerate lung function decline, decrease quality of life, and, when severe, increase mortality.2 Infections cause an estimated 50% to 70% of COPD exacerbations.2-4

Global Initiative for Chronic Obstructive Lung Disease (GOLD) guidelines recommend using antibiotics to treat exacerbations in patients with moderate or severe COPD who:
• have increased dyspnea, sputum volume, and sputum purulence;
• have 2 of these 3 symptoms if increased sputum purulence is one of the symptoms; or
• require mechanical ventilation.2

According to the GOLD guidelines, the choice of antibiotic should be based on local antibiograms; common options include amoxicillin, amoxicillin/clavulanate, azithromycin, and doxycycline.2 Although the GOLD guidelines cover use of antibiotics for COPD exacerbations, this recommendation is based on analyses of studies that focused on patients with moderate or severe COPD.2 There has been little research on using antibiotics for exacerbations of mild COPD.

STUDY SUMMARY: Using antibiotics often resolves symptoms

Llor et al1 conducted a multicenter, doubleblind placebo-controlled RCT to examine the effectiveness of antibiotic treatment for COPD exacerbations. Participants (ages ≥40 years) had mild to moderate COPD, defined as ≥10 pack-years of smoking, a forced expiratory volume in 1 second (FEV1) >50%, and a FEV1-to-forced vital capacity ratio <0.7. An exacerbation was defined as at least one of the following: increased dyspnea, increased sputum volume, or sputum purulence. Patients were randomized to receive amoxicillin/clavulanate 500/125 mg or placebo 3 times a day for 8 days. The primary endpoints were clinical cure (resolution of symptoms) and clinical success (resolution or improvement of symptoms) at Days 9 to 11 as determined by physician assessment. Secondary measures included cure and clinical success at Day 20 and time until next exacerbation. Patients were monitored for one year after the exacerbation.

There were 162 patients in the antibiotic group and 156 in the placebo group; the 2 groups were demographically similar. In each group, 4 patients withdrew consent and were removed from analysis. By the 9- to 11-day follow- up visit, 74.1% of patients in the antibiotic group had clinical cure, compared with 59.9% in the placebo group (P=.016; number needed to treat [NNT]=7). Clinical success also was significantly greater with antibiotics compared with placebo (90.5% vs 80.9%; P=.022).

By Day 11, symptoms resolved in 74.1% of patients in the antibiotic group, compared with 59.9% in the placebo group.The clinical cure rate at Day 20 also was significantly greater in patients on antibiotics compared with placebo (81.6% vs 67.8%; P=.006, NNT=7). During the one-year followup, 58% of patients in the antibiotic group and 73.2% of those in the placebo group experienced additional exacerbations. Time to next exacerbation was significantly longer in patients taking antibiotics (233 days vs 160 days; P=.015).

Can CRP level help determine who should—and shouldn’t—receive antibiotics? Previous studies have identified biomarkers, including C-reactive protein (CRP), that indicate COPD exacerbation, but have not linked them to clinical course.5-7 In this study, researchers measured CRP in patients receiving placebo to determine if this biomarker could predict clinical outcomes.

The researchers found that the clinical success rate among patients with a CRP <40 mg/L was 87.6%, while only 34.5% of patients with a CRP >40 mg/L experienced clinical success (sensitivity and specificity for clinical success at this cutoff was 0.655 and 0.876, respectively). This suggests that antibiotics might be appropriate for patients with an exacerbation of mild or moderate COPD who have a CRP >40 mg/L.

A total of 35 adverse events were reported: 23 in patients taking antibiotics and 12 among patients receiving placebo. Only 2 patients discontinued treatment due to adverse events in antibiotics group. Most of these reactions were mild gastrointestinal problems.

WHAT'S NEW?: Evidence supports antibiotics for mild to moderate COPD

Few placebo-controlled trials have addressed antibiotic use for exacerbations in patients with mild to moderate COPD.2,8,9 This study demonstrated that compared with placebo, symptom resolution and clinical success is greater with amoxicillin/clavulanate, and that antibiotic treatment also may increase time until next exacerbation.

 

 

The study also looked at the relationship of CRP and exacerbations in the placebo group. Higher spontaneous clinical cure rates were noted when the CRP was <40 mg/L.

CAVEATS: Effects of concomitant medications are unclear

In both the placebo and antibiotic groups, patients were taking other medications (including short-and long-acting beta-agonists, anticholinergics, theophyllines, and oral or inhaled corticosteroids). Roughly the same number of patients in each group took additional medications, but researchers did not conduct a subgroup analysis to see if patients treated with these medications responded differently than those who received antibiotics alone.

Clinical success also was significantly greater with antibiotics compared with placebo.GOLD guidelines already suggest antibiotics for exacerbations in patients with moderate COPD.2 In this study, 89% of patients met criteria for moderate COPD and 11% for mild COPD. Though the percentage of patients who had mild COPD was small, we believe the results of this study warrant consideration of antibiotic use in patients with mild disease. Local antibiograms may show increased resistance to amoxicillin/clavulanate; this study did not address the use of other antibiotics.

CHALLENGES TO IMPLEMENTATION: Antibiotic overuse may be a concern

With increased awareness of inappropriate antibiotic use, physicians might have concerns about antibiotic resistance developing as a result of using antibiotics for exacerbations of mild to moderate COPD.

ACKNOWLEDGEMENT
The PURLs Surveillance System was supported in part by Grant Number UL1RR024999 from the National Center For Research Resources, a Clinical Translational Science Award to the University of Chicago. The content is solely the responsibility of the authors and does not necessarily represent the official views of the National Center For Research Resources or the National Institutes of Health.

References

 

1. Llor C, Moragas A, Hernández S, et al. Efficacy of antibiotic therapy for acute exacerbations of mild to moderate chronic obstructive pulmonary disease. Am J Respir Crit Care Med. 2012;186:716-723.

2. Global strategy for the diagnosis, management, and prevention of chronic obstructive pulmonary disease. January 2014. Global Initiative for Chronic Obstructive Lung Disease (GOLD) Web site. Available at: http://www.goldcopd.org/guidelines-globalstrategy-for-diagnosis-management.html. Accessed March 19, 2014.

3. Donaldson GC, Seemungal TA, Bhowmik A, et al. Relationship between exacerbation frequency and lung function decline in chronic obstructive pulmonary disease. Thorax. 2002;57:847-852.

4. Soler-Cataluña JJ, Martínez-García MA, Román Sánchez P, et al. Severe acute exacerbations and mortality in patients with chronic obstructive pulmonary disease. Thorax. 2005;60:925-931.

5. Vollenweider DJ, Jarrett H, Steurer-Stey CA, et al. Antibiotics for exacerbations of chronic obstructive pulmonary disease. Cochrane Database Syst Rev. 2012;12:CD010257.

6. Bartlett, JG, Sethi S. Management of infection in acute exacerbations of chronic obstructive pulmonary disease. In: Basow DS, ed. UpToDate [database online]. Available at: http://www.uptodate. com. Last updated March 27, 2012. Accessed January 2, 2013.

7. Lacoma A, Prat C, Andreo F, et al. Value of procalcitonin, C-reactive protein, and neopterin in exacerbations of chronic obstructive pulmonary disease. Int J Chron Obstruct Pulmon Dis. 2011;6:157-169.

8. Antonescu-Turcu AL, Tomic R. C-reactive protein and copeptin: prognostic predictors in chronic obstructive pulmonary disease exacerbations. Curr Opin Pulm Med. 2009;15:120-125.

9. Thomsen M, Ingebrigtsen TS, Marott JL, et al. Inflammatory biomarkers and exacerbations in chronic obstructive pulmonary disease. JAMA. 2013;309:2353-2361.

References

 

1. Llor C, Moragas A, Hernández S, et al. Efficacy of antibiotic therapy for acute exacerbations of mild to moderate chronic obstructive pulmonary disease. Am J Respir Crit Care Med. 2012;186:716-723.

2. Global strategy for the diagnosis, management, and prevention of chronic obstructive pulmonary disease. January 2014. Global Initiative for Chronic Obstructive Lung Disease (GOLD) Web site. Available at: http://www.goldcopd.org/guidelines-globalstrategy-for-diagnosis-management.html. Accessed March 19, 2014.

3. Donaldson GC, Seemungal TA, Bhowmik A, et al. Relationship between exacerbation frequency and lung function decline in chronic obstructive pulmonary disease. Thorax. 2002;57:847-852.

4. Soler-Cataluña JJ, Martínez-García MA, Román Sánchez P, et al. Severe acute exacerbations and mortality in patients with chronic obstructive pulmonary disease. Thorax. 2005;60:925-931.

5. Vollenweider DJ, Jarrett H, Steurer-Stey CA, et al. Antibiotics for exacerbations of chronic obstructive pulmonary disease. Cochrane Database Syst Rev. 2012;12:CD010257.

6. Bartlett, JG, Sethi S. Management of infection in acute exacerbations of chronic obstructive pulmonary disease. In: Basow DS, ed. UpToDate [database online]. Available at: http://www.uptodate. com. Last updated March 27, 2012. Accessed January 2, 2013.

7. Lacoma A, Prat C, Andreo F, et al. Value of procalcitonin, C-reactive protein, and neopterin in exacerbations of chronic obstructive pulmonary disease. Int J Chron Obstruct Pulmon Dis. 2011;6:157-169.

8. Antonescu-Turcu AL, Tomic R. C-reactive protein and copeptin: prognostic predictors in chronic obstructive pulmonary disease exacerbations. Curr Opin Pulm Med. 2009;15:120-125.

9. Thomsen M, Ingebrigtsen TS, Marott JL, et al. Inflammatory biomarkers and exacerbations in chronic obstructive pulmonary disease. JAMA. 2013;309:2353-2361.

Issue
The Journal of Family Practice - 63(4)
Issue
The Journal of Family Practice - 63(4)
Page Number
E11-E13
Page Number
E11-E13
Publications
Publications
Topics
Article Type
Display Headline
Should you consider antibiotics for exacerbations of mild COPD?
Display Headline
Should you consider antibiotics for exacerbations of mild COPD?
Legacy Keywords
Tanner Nissly; DO; Shailendra Prasad; MBBS; MPH; COPD; chronic obstructive pulmonary disease; antibiotics; GOLD guidelines; Global Initiative for Chronic Obstructive Lung Disease
Legacy Keywords
Tanner Nissly; DO; Shailendra Prasad; MBBS; MPH; COPD; chronic obstructive pulmonary disease; antibiotics; GOLD guidelines; Global Initiative for Chronic Obstructive Lung Disease
Sections
PURLs Copyright

Copyright © 2014 Family Physicians Inquiries Network. All rights reserved.

Disallow All Ads
Alternative CME
Use ProPublica
Hide sidebar & use full width
render the right sidebar.
Conference Recap Checkbox
Not Conference Recap
Clinical Edge
Display the Slideshow in this Article
Article PDF Media
Media Files

Prolotherapy: A nontraditional approach to knee osteoarthritis

Article Type
Changed
Tue, 07/14/2020 - 10:14
Display Headline
Prolotherapy: A nontraditional approach to knee osteoarthritis

 

PRACTICE CHANGER

Recommend prolotherapy for patients with knee osteoarthritis (OA) that does not respond to conventional therapies.1

Strength of recommendation

B: Based on a 3-arm, blinded, randomized controlled trial (RCT).

Rabago D, Patterson JJ, Mundt M, et al. Dextrose prolotherapy for knee osteoarthritis: a randomized controlled trial. Ann Fam Med. 2013;11:229-237.

Illustrative case

A 59-year-old woman with OA comes to your office with chronic knee pain. She has tried acetaminophen, ibuprofen, intra-articular corticosteroid injections, and physical therapy without significant improvement in pain or functioning. She wants to avoid daily medications or surgery and wonders if there are any interventions that will not lead to prolonged time away from work. What would you consider?

Additional options needed for knee OA

More than 25% of adults ages 55 years and older suffer from knee pain, and OA is an increasingly common cause.2 Knee pain is a major source of morbidity in the United States; it limits patients’ activities and increases comorbidities such as depression and obesity.

Conventional outpatient treatments for knee pain range from acetaminophen, nonsteroidal anti-inflammatory drugs, glucosamine, chondroitin, and opiates to topical capsaicin therapy, intra-articular hyaluronic acid, and corticosteroid injections. Cost, efficacy, and safety limit these therapies.3

Prolotherapy is another option used to treat musculoskeletal pain. It involves repeatedly injecting a sclerosing solution (usually dextrose) into the sites of chronic musculoskeletal pain.4 The mechanism of action is thought to be the result of local tissue irritation stimulating inflammatory pathways, which leads to the release of growth factors and subsequent healing.4,5 Previous studies evaluating the usefulness of prolotherapy have lacked methodological rigor, have not been randomized adequately, or have lacked a placebo comparison.6-9

STUDY SUMMARY: Prolotherapy reduces pain more than exercise or placebo

Rabago et al1 randomized 90 participants to dextrose prolotherapy, placebo saline injections, or at-home exercise. Participants had a ≥3 month history of painful knee OA based on a self-reported pain scale, radiographic evidence of knee OA within the past 5 years, and tenderness of ≥1 or more anterior knee structures on exam.

Sixty-six percent of participants were female. The mean age was 56.7 years and 74% were overweight (body mass index [BMI], 25-29.9) or obese (BMI ≥30). Participants chose to have one or both knees treated; 43 knees were injected in the dextrose group, 41 received saline injections, and 47 were assessed in the exercise group. There were no significant differences among groups at baseline.

Participants in the prolotherapy and saline groups received injections at 1, 5, and 9 weeks, plus optional injections at 13 and 17 weeks per physician and participant preference. Injections were administered both extra- and intra-articularly. Intra-articular injections were delivered using a 25-gauge needle with a mixture of 25% dextrose, 1% saline, and 1% lidocaine for a total volume of 6 mL. Extra-articular injections were delivered with a peppering technique with a maximum of 15 punctures over painful ligaments and tendons around the knee. The extra-articular solution was similar to the intra-articular except 15% dextrose was used, with a total maximum volume of 22.5 mL.

The placebo injection group received injections in the same pattern and technique, but the solution was the same quantity of 1% lidocaine plus 1% saline to achieve the same volume. The injector, outcome assessor, primary investigator, and participants were blinded to injection group.

In the exercise group, a study coordinator taught participants knee exercises and gave them a pamphlet with 10 exercises to perform at home. Adherence to at-home exercises was assessed with monthly logs that participants mailed in for the first 20 weeks of the study. Seventy-seven percent of participants reported doing their at-home exercises.

The primary outcome measure was change in composite score on the Western Ontario McMaster Universities Osteoarthritis Index (WOMAC), a validated questionnaire used to evaluate knee-related quality of life that features subscales for pain, stiffness, and function.10 The minimal clinically important difference in change in score on this 100-point instrument is 12 points; higher scores indicate better quality of life.11 The secondary outcome was change in score on the Knee Pain Scale (KPS), a validated questionnaire that uses a 4-point scale to measure pain frequency and a 5-point scale to measure pain severity; higher scores indicate worse symptoms.12

Improvements seen in both scores

Using an intention-to-treat analysis for all groups, WOMAC composite scores improved at 9 weeks and remained improved through 52 weeks. At 9 weeks, the dextrose group increased 13.91 points, compared with 6.75 (P=.020) in the saline group and 2.51 (P=.001) points in the exercise group.

Prolotherapy for knee OA reduced pain frequency and severity more effectively than exercise or saline injections.At 52 weeks, the dextrose group showed an improvement of 15.32 points compared with 7.59 (P=.022) in the saline group and 8.24 (P=.034) in the exercise group. Fifty percent (15/30) of participants in the dextrose group had clinically meaningful improvement as measured by an increase of ≥12 points on the WOMAC, compared with 34% (10/29) and 26% (8/31) in the saline and exercise groups, respectively. At 52 weeks, the dextrose group had significantly decreased KPS knee pain frequency scores compared with the saline group (mean difference [MD], -1.20 vs. -0.60; P<.05) and exercise group (MD, -1.20 vs. -0.40; P<.05). Knee pain severity scores also decreased in the dextrose group compared to the saline (MD, -0.92 vs. -0.32, P<.05) and exercise groups (MD, -0.92 vs. -0.11; P<.05). There were no significant differences in KPS score decreases between the saline and exercise groups.

 

 

What about patient satisfaction?

At week 52, all participants were asked, “Would you recommend the therapy you received in this study to others with knee OA like yours?” Ninety-one percent of the dextrose group, 82% of the saline group, and 89% of the exercise group answered “Yes.”

All participants who received injections reported mild to moderate post-injection pain. Five participants in the saline group and 3 in the dextrose group experienced bruising. No other side effects or adverse events were documented. According to daily logs of medication use in the 7 days after injection, 74% of patients in the dextrose group used acetaminophen and 47% used oxycodone, compared with 63% and 43%, respectively, in the saline group. The study authors did not comment on the significance of these differences.

WHAT'S NEW: A randomized study provides support for prolotherapy

This study is the first to adequately demonstrate improvement in knee-related quality of life with prolotherapy compared with placebo (saline) or exercise. Family physicians can now add this therapy to their “toolbox” for patient complaints of OA pain.

CAVEATS: Efficacy is unknown in patients with certain comorbidities

Efficacy is unknown in patients with certain comorbidities Of 894 people screened, only 118 met initial eligibility criteria. This study did not include patients who were taking daily opioids, had diabetes, or had a BMI >40, so its results may not be generalizable to such patients.

Prolotherapy for knee OA can be performed in an outpatient setting <15 minutes; the challenge is finding a certified prolotherapist to do it.Also, while the study demonstrated no side effects or adverse events other than bruising in 8 patients, the sample size may have been too small to detect less common adverse events. However, prior studies of prolotherapy have not revealed any substantial adverse effects.7

Strong evidence for some conditions… not for others. The strongest data support the efficacy of prolotherapy for focal tendinopathy (lateral epicondylosis) and knee OA. Evidence supporting prolotherapy for multimodal conditions, such as chronic low back pain, is less robust.4

CHALLENGES TO IMPLEMENTATION: Finding a prolotherapist near you may not be easy

The main challenge to implementation is finding a certified prolotherapist, or obtaining training in the technique. The prolotherapy knee protocol can be performed in an outpatient setting in less than 15 minutes, but the technique requires training. Prolotherapy training is available from multiple organizations, including the American Association of Orthopaedic Medicine, which requires 100 course hours for prolotherapy certification.4 No formal survey on the number of prolotherapists in the United States has been conducted since 1993,13 but Rabago et al1 estimated that the number is in the hundreds.

Insurance coverage frequently is a challenge. Most third-party payers do not cover prolotherapy, and currently most patients pay out-of-pocket. Rabago et al1 indicated that at their institution, the cost is $218 per injection session. Another study published in 2010 put the average total cost of 4 to 6 prolotherapy sessions at $1800.14

And from the patient’s perspective … The multiple needle sticks involved in prolotherapy can be painful.

ACKNOWLEDGEMENT
The PURLs Surveillance System was supported in part by Grant Number UL1RR024999 from the National Center for Research Resources, a Clinical Translational Science Award to the University of Chicago. The content is solely the responsibility of the authors and does not necessarily represent the official views of the National Center of Research Resources or the National Institutes of Health.

Files
References

 

1. Rabago D, Patterson JJ, Mundt M, et al. Dextrose prolotherapy for knee osteoarthritis: a randomized controlled trial. Ann Fam Med. 2013;11:229-237.

2. Peat G, McCarney R, Croft P. Knee pain and osteoarthritis in older adults: a review of community burden and current use of primary health care. Ann Rheum Dis. 2001;60:91-97.

3. Lawrence RC, Felson DT, Helmick CG, et al; National Arthritis Data Workgroup. Estimates of the prevalence of arthritis and other rheumatic conditions in the United States. Part II. Arthritis Rheum. 2008;58:26-35.

4. Rabago D, Slattengren A, Zgierska A. Prolotherapy in primary care practice. Prim Care. 2010;37:65-80.

5. Hackett GS, Hemwall GA, Montgomery GA. Ligament and Tendon Relaxation Treated by Prolotherapy. 5th ed. Oak Park, IL: Institute in Basic Life Principles; 1991.

6. Schultz LW. A treatment for subluxation of the temporomandibular joint. JAMA. 1937;109:1032-1035.

7. Rabago D, Best TM, Beamsley M, et al. A systematic review of prolotherapy for chronic musculoskeletal pain. Clin J Sport Med. 2005;15:376-380.

8. Reeves KD, Hassanein KM. Long-term effects of dextrose prolotherapy for anterior cruciate ligament laxity. Altern Ther Health Med. 2003;9:58-62.

9. Reeves KD, Hassanein K. Randomized prospective double-blind placebo-controlled study of dextrose prolotherapy for knee osteoarthritis with or without ACL laxity. Altern Ther Health Med. 2000;6:68-74,77-80.

10. Roos EM, Klässbo M, Lohmander LS. WOMAC osteoarthritis index. Reliability, validity, and responsiveness in patients with arthroscopically assessed osteoarthritis. Western Ontario and MacMaster Universities. Scand J Rheumatol. 1999;28:210-215.

11. Ehrich EW, Davies GM, Watson DJ, et al. Minimal perceptible clinical improvement with the Western Ontario and McMaster Universities osteoarthritis index questionnaire and global assessments in patients with osteoarthritis. J Rheumatol. 2000;27:2635-2641.

12. Rejeski WJ, Ettinger WH Jr, Shumaker S, et al. The evaluation of pain in patients with knee osteoarthritis: the knee pain scale. J Rheumatol. 1995;22:1124-1129.

13. Dorman TA. Prolotherapy: A survey. J Orthop Med. 1993;15:28-32.

14. Hauser RA, Hauser MA, Baird NM, et al. Prolotherapy as an alternative to surgery: A prospective pilot study of 34 patients from a private medical practice. J Prolotherapy. 2010;2:272-281.

Article PDF
Author and Disclosure Information

 

Andrew H. Slattengren, DO
Trent Christensen, MD
Shailendra Prasad, MBBS, MPH
Kohar Jones, MD

North Memorial Family Medicine Residency, University of Minnesota, Minneapolis (Drs. Slattengren, Christensen, and Prasad); Department of Family Medicine, The University of Chicago (Dr. Jones)

PURLs EDITOR
Kate Rowland, MD, MS
Department of Family Medicine, The University of Chicago

Issue
The Journal of Family Practice - 63(4)
Publications
Topics
Page Number
206-208
Legacy Keywords
Andrew H. Slattengren;, DO; Trent Christensen; MD; Shailendra Prasad; MBBS; MPH; Kohar Jones; MD; prolotherapy; knee osteoarthritis; OA; exercise; WOMAC; Western Ontario McMaster Universities Osteoarthritis Index; KPS; Knee Pain Scale
Sections
Files
Files
Author and Disclosure Information

 

Andrew H. Slattengren, DO
Trent Christensen, MD
Shailendra Prasad, MBBS, MPH
Kohar Jones, MD

North Memorial Family Medicine Residency, University of Minnesota, Minneapolis (Drs. Slattengren, Christensen, and Prasad); Department of Family Medicine, The University of Chicago (Dr. Jones)

PURLs EDITOR
Kate Rowland, MD, MS
Department of Family Medicine, The University of Chicago

Author and Disclosure Information

 

Andrew H. Slattengren, DO
Trent Christensen, MD
Shailendra Prasad, MBBS, MPH
Kohar Jones, MD

North Memorial Family Medicine Residency, University of Minnesota, Minneapolis (Drs. Slattengren, Christensen, and Prasad); Department of Family Medicine, The University of Chicago (Dr. Jones)

PURLs EDITOR
Kate Rowland, MD, MS
Department of Family Medicine, The University of Chicago

Article PDF
Article PDF

 

PRACTICE CHANGER

Recommend prolotherapy for patients with knee osteoarthritis (OA) that does not respond to conventional therapies.1

Strength of recommendation

B: Based on a 3-arm, blinded, randomized controlled trial (RCT).

Rabago D, Patterson JJ, Mundt M, et al. Dextrose prolotherapy for knee osteoarthritis: a randomized controlled trial. Ann Fam Med. 2013;11:229-237.

Illustrative case

A 59-year-old woman with OA comes to your office with chronic knee pain. She has tried acetaminophen, ibuprofen, intra-articular corticosteroid injections, and physical therapy without significant improvement in pain or functioning. She wants to avoid daily medications or surgery and wonders if there are any interventions that will not lead to prolonged time away from work. What would you consider?

Additional options needed for knee OA

More than 25% of adults ages 55 years and older suffer from knee pain, and OA is an increasingly common cause.2 Knee pain is a major source of morbidity in the United States; it limits patients’ activities and increases comorbidities such as depression and obesity.

Conventional outpatient treatments for knee pain range from acetaminophen, nonsteroidal anti-inflammatory drugs, glucosamine, chondroitin, and opiates to topical capsaicin therapy, intra-articular hyaluronic acid, and corticosteroid injections. Cost, efficacy, and safety limit these therapies.3

Prolotherapy is another option used to treat musculoskeletal pain. It involves repeatedly injecting a sclerosing solution (usually dextrose) into the sites of chronic musculoskeletal pain.4 The mechanism of action is thought to be the result of local tissue irritation stimulating inflammatory pathways, which leads to the release of growth factors and subsequent healing.4,5 Previous studies evaluating the usefulness of prolotherapy have lacked methodological rigor, have not been randomized adequately, or have lacked a placebo comparison.6-9

STUDY SUMMARY: Prolotherapy reduces pain more than exercise or placebo

Rabago et al1 randomized 90 participants to dextrose prolotherapy, placebo saline injections, or at-home exercise. Participants had a ≥3 month history of painful knee OA based on a self-reported pain scale, radiographic evidence of knee OA within the past 5 years, and tenderness of ≥1 or more anterior knee structures on exam.

Sixty-six percent of participants were female. The mean age was 56.7 years and 74% were overweight (body mass index [BMI], 25-29.9) or obese (BMI ≥30). Participants chose to have one or both knees treated; 43 knees were injected in the dextrose group, 41 received saline injections, and 47 were assessed in the exercise group. There were no significant differences among groups at baseline.

Participants in the prolotherapy and saline groups received injections at 1, 5, and 9 weeks, plus optional injections at 13 and 17 weeks per physician and participant preference. Injections were administered both extra- and intra-articularly. Intra-articular injections were delivered using a 25-gauge needle with a mixture of 25% dextrose, 1% saline, and 1% lidocaine for a total volume of 6 mL. Extra-articular injections were delivered with a peppering technique with a maximum of 15 punctures over painful ligaments and tendons around the knee. The extra-articular solution was similar to the intra-articular except 15% dextrose was used, with a total maximum volume of 22.5 mL.

The placebo injection group received injections in the same pattern and technique, but the solution was the same quantity of 1% lidocaine plus 1% saline to achieve the same volume. The injector, outcome assessor, primary investigator, and participants were blinded to injection group.

In the exercise group, a study coordinator taught participants knee exercises and gave them a pamphlet with 10 exercises to perform at home. Adherence to at-home exercises was assessed with monthly logs that participants mailed in for the first 20 weeks of the study. Seventy-seven percent of participants reported doing their at-home exercises.

The primary outcome measure was change in composite score on the Western Ontario McMaster Universities Osteoarthritis Index (WOMAC), a validated questionnaire used to evaluate knee-related quality of life that features subscales for pain, stiffness, and function.10 The minimal clinically important difference in change in score on this 100-point instrument is 12 points; higher scores indicate better quality of life.11 The secondary outcome was change in score on the Knee Pain Scale (KPS), a validated questionnaire that uses a 4-point scale to measure pain frequency and a 5-point scale to measure pain severity; higher scores indicate worse symptoms.12

Improvements seen in both scores

Using an intention-to-treat analysis for all groups, WOMAC composite scores improved at 9 weeks and remained improved through 52 weeks. At 9 weeks, the dextrose group increased 13.91 points, compared with 6.75 (P=.020) in the saline group and 2.51 (P=.001) points in the exercise group.

Prolotherapy for knee OA reduced pain frequency and severity more effectively than exercise or saline injections.At 52 weeks, the dextrose group showed an improvement of 15.32 points compared with 7.59 (P=.022) in the saline group and 8.24 (P=.034) in the exercise group. Fifty percent (15/30) of participants in the dextrose group had clinically meaningful improvement as measured by an increase of ≥12 points on the WOMAC, compared with 34% (10/29) and 26% (8/31) in the saline and exercise groups, respectively. At 52 weeks, the dextrose group had significantly decreased KPS knee pain frequency scores compared with the saline group (mean difference [MD], -1.20 vs. -0.60; P<.05) and exercise group (MD, -1.20 vs. -0.40; P<.05). Knee pain severity scores also decreased in the dextrose group compared to the saline (MD, -0.92 vs. -0.32, P<.05) and exercise groups (MD, -0.92 vs. -0.11; P<.05). There were no significant differences in KPS score decreases between the saline and exercise groups.

 

 

What about patient satisfaction?

At week 52, all participants were asked, “Would you recommend the therapy you received in this study to others with knee OA like yours?” Ninety-one percent of the dextrose group, 82% of the saline group, and 89% of the exercise group answered “Yes.”

All participants who received injections reported mild to moderate post-injection pain. Five participants in the saline group and 3 in the dextrose group experienced bruising. No other side effects or adverse events were documented. According to daily logs of medication use in the 7 days after injection, 74% of patients in the dextrose group used acetaminophen and 47% used oxycodone, compared with 63% and 43%, respectively, in the saline group. The study authors did not comment on the significance of these differences.

WHAT'S NEW: A randomized study provides support for prolotherapy

This study is the first to adequately demonstrate improvement in knee-related quality of life with prolotherapy compared with placebo (saline) or exercise. Family physicians can now add this therapy to their “toolbox” for patient complaints of OA pain.

CAVEATS: Efficacy is unknown in patients with certain comorbidities

Efficacy is unknown in patients with certain comorbidities Of 894 people screened, only 118 met initial eligibility criteria. This study did not include patients who were taking daily opioids, had diabetes, or had a BMI >40, so its results may not be generalizable to such patients.

Prolotherapy for knee OA can be performed in an outpatient setting <15 minutes; the challenge is finding a certified prolotherapist to do it.Also, while the study demonstrated no side effects or adverse events other than bruising in 8 patients, the sample size may have been too small to detect less common adverse events. However, prior studies of prolotherapy have not revealed any substantial adverse effects.7

Strong evidence for some conditions… not for others. The strongest data support the efficacy of prolotherapy for focal tendinopathy (lateral epicondylosis) and knee OA. Evidence supporting prolotherapy for multimodal conditions, such as chronic low back pain, is less robust.4

CHALLENGES TO IMPLEMENTATION: Finding a prolotherapist near you may not be easy

The main challenge to implementation is finding a certified prolotherapist, or obtaining training in the technique. The prolotherapy knee protocol can be performed in an outpatient setting in less than 15 minutes, but the technique requires training. Prolotherapy training is available from multiple organizations, including the American Association of Orthopaedic Medicine, which requires 100 course hours for prolotherapy certification.4 No formal survey on the number of prolotherapists in the United States has been conducted since 1993,13 but Rabago et al1 estimated that the number is in the hundreds.

Insurance coverage frequently is a challenge. Most third-party payers do not cover prolotherapy, and currently most patients pay out-of-pocket. Rabago et al1 indicated that at their institution, the cost is $218 per injection session. Another study published in 2010 put the average total cost of 4 to 6 prolotherapy sessions at $1800.14

And from the patient’s perspective … The multiple needle sticks involved in prolotherapy can be painful.

ACKNOWLEDGEMENT
The PURLs Surveillance System was supported in part by Grant Number UL1RR024999 from the National Center for Research Resources, a Clinical Translational Science Award to the University of Chicago. The content is solely the responsibility of the authors and does not necessarily represent the official views of the National Center of Research Resources or the National Institutes of Health.

 

PRACTICE CHANGER

Recommend prolotherapy for patients with knee osteoarthritis (OA) that does not respond to conventional therapies.1

Strength of recommendation

B: Based on a 3-arm, blinded, randomized controlled trial (RCT).

Rabago D, Patterson JJ, Mundt M, et al. Dextrose prolotherapy for knee osteoarthritis: a randomized controlled trial. Ann Fam Med. 2013;11:229-237.

Illustrative case

A 59-year-old woman with OA comes to your office with chronic knee pain. She has tried acetaminophen, ibuprofen, intra-articular corticosteroid injections, and physical therapy without significant improvement in pain or functioning. She wants to avoid daily medications or surgery and wonders if there are any interventions that will not lead to prolonged time away from work. What would you consider?

Additional options needed for knee OA

More than 25% of adults ages 55 years and older suffer from knee pain, and OA is an increasingly common cause.2 Knee pain is a major source of morbidity in the United States; it limits patients’ activities and increases comorbidities such as depression and obesity.

Conventional outpatient treatments for knee pain range from acetaminophen, nonsteroidal anti-inflammatory drugs, glucosamine, chondroitin, and opiates to topical capsaicin therapy, intra-articular hyaluronic acid, and corticosteroid injections. Cost, efficacy, and safety limit these therapies.3

Prolotherapy is another option used to treat musculoskeletal pain. It involves repeatedly injecting a sclerosing solution (usually dextrose) into the sites of chronic musculoskeletal pain.4 The mechanism of action is thought to be the result of local tissue irritation stimulating inflammatory pathways, which leads to the release of growth factors and subsequent healing.4,5 Previous studies evaluating the usefulness of prolotherapy have lacked methodological rigor, have not been randomized adequately, or have lacked a placebo comparison.6-9

STUDY SUMMARY: Prolotherapy reduces pain more than exercise or placebo

Rabago et al1 randomized 90 participants to dextrose prolotherapy, placebo saline injections, or at-home exercise. Participants had a ≥3 month history of painful knee OA based on a self-reported pain scale, radiographic evidence of knee OA within the past 5 years, and tenderness of ≥1 or more anterior knee structures on exam.

Sixty-six percent of participants were female. The mean age was 56.7 years and 74% were overweight (body mass index [BMI], 25-29.9) or obese (BMI ≥30). Participants chose to have one or both knees treated; 43 knees were injected in the dextrose group, 41 received saline injections, and 47 were assessed in the exercise group. There were no significant differences among groups at baseline.

Participants in the prolotherapy and saline groups received injections at 1, 5, and 9 weeks, plus optional injections at 13 and 17 weeks per physician and participant preference. Injections were administered both extra- and intra-articularly. Intra-articular injections were delivered using a 25-gauge needle with a mixture of 25% dextrose, 1% saline, and 1% lidocaine for a total volume of 6 mL. Extra-articular injections were delivered with a peppering technique with a maximum of 15 punctures over painful ligaments and tendons around the knee. The extra-articular solution was similar to the intra-articular except 15% dextrose was used, with a total maximum volume of 22.5 mL.

The placebo injection group received injections in the same pattern and technique, but the solution was the same quantity of 1% lidocaine plus 1% saline to achieve the same volume. The injector, outcome assessor, primary investigator, and participants were blinded to injection group.

In the exercise group, a study coordinator taught participants knee exercises and gave them a pamphlet with 10 exercises to perform at home. Adherence to at-home exercises was assessed with monthly logs that participants mailed in for the first 20 weeks of the study. Seventy-seven percent of participants reported doing their at-home exercises.

The primary outcome measure was change in composite score on the Western Ontario McMaster Universities Osteoarthritis Index (WOMAC), a validated questionnaire used to evaluate knee-related quality of life that features subscales for pain, stiffness, and function.10 The minimal clinically important difference in change in score on this 100-point instrument is 12 points; higher scores indicate better quality of life.11 The secondary outcome was change in score on the Knee Pain Scale (KPS), a validated questionnaire that uses a 4-point scale to measure pain frequency and a 5-point scale to measure pain severity; higher scores indicate worse symptoms.12

Improvements seen in both scores

Using an intention-to-treat analysis for all groups, WOMAC composite scores improved at 9 weeks and remained improved through 52 weeks. At 9 weeks, the dextrose group increased 13.91 points, compared with 6.75 (P=.020) in the saline group and 2.51 (P=.001) points in the exercise group.

Prolotherapy for knee OA reduced pain frequency and severity more effectively than exercise or saline injections.At 52 weeks, the dextrose group showed an improvement of 15.32 points compared with 7.59 (P=.022) in the saline group and 8.24 (P=.034) in the exercise group. Fifty percent (15/30) of participants in the dextrose group had clinically meaningful improvement as measured by an increase of ≥12 points on the WOMAC, compared with 34% (10/29) and 26% (8/31) in the saline and exercise groups, respectively. At 52 weeks, the dextrose group had significantly decreased KPS knee pain frequency scores compared with the saline group (mean difference [MD], -1.20 vs. -0.60; P<.05) and exercise group (MD, -1.20 vs. -0.40; P<.05). Knee pain severity scores also decreased in the dextrose group compared to the saline (MD, -0.92 vs. -0.32, P<.05) and exercise groups (MD, -0.92 vs. -0.11; P<.05). There were no significant differences in KPS score decreases between the saline and exercise groups.

 

 

What about patient satisfaction?

At week 52, all participants were asked, “Would you recommend the therapy you received in this study to others with knee OA like yours?” Ninety-one percent of the dextrose group, 82% of the saline group, and 89% of the exercise group answered “Yes.”

All participants who received injections reported mild to moderate post-injection pain. Five participants in the saline group and 3 in the dextrose group experienced bruising. No other side effects or adverse events were documented. According to daily logs of medication use in the 7 days after injection, 74% of patients in the dextrose group used acetaminophen and 47% used oxycodone, compared with 63% and 43%, respectively, in the saline group. The study authors did not comment on the significance of these differences.

WHAT'S NEW: A randomized study provides support for prolotherapy

This study is the first to adequately demonstrate improvement in knee-related quality of life with prolotherapy compared with placebo (saline) or exercise. Family physicians can now add this therapy to their “toolbox” for patient complaints of OA pain.

CAVEATS: Efficacy is unknown in patients with certain comorbidities

Efficacy is unknown in patients with certain comorbidities Of 894 people screened, only 118 met initial eligibility criteria. This study did not include patients who were taking daily opioids, had diabetes, or had a BMI >40, so its results may not be generalizable to such patients.

Prolotherapy for knee OA can be performed in an outpatient setting <15 minutes; the challenge is finding a certified prolotherapist to do it.Also, while the study demonstrated no side effects or adverse events other than bruising in 8 patients, the sample size may have been too small to detect less common adverse events. However, prior studies of prolotherapy have not revealed any substantial adverse effects.7

Strong evidence for some conditions… not for others. The strongest data support the efficacy of prolotherapy for focal tendinopathy (lateral epicondylosis) and knee OA. Evidence supporting prolotherapy for multimodal conditions, such as chronic low back pain, is less robust.4

CHALLENGES TO IMPLEMENTATION: Finding a prolotherapist near you may not be easy

The main challenge to implementation is finding a certified prolotherapist, or obtaining training in the technique. The prolotherapy knee protocol can be performed in an outpatient setting in less than 15 minutes, but the technique requires training. Prolotherapy training is available from multiple organizations, including the American Association of Orthopaedic Medicine, which requires 100 course hours for prolotherapy certification.4 No formal survey on the number of prolotherapists in the United States has been conducted since 1993,13 but Rabago et al1 estimated that the number is in the hundreds.

Insurance coverage frequently is a challenge. Most third-party payers do not cover prolotherapy, and currently most patients pay out-of-pocket. Rabago et al1 indicated that at their institution, the cost is $218 per injection session. Another study published in 2010 put the average total cost of 4 to 6 prolotherapy sessions at $1800.14

And from the patient’s perspective … The multiple needle sticks involved in prolotherapy can be painful.

ACKNOWLEDGEMENT
The PURLs Surveillance System was supported in part by Grant Number UL1RR024999 from the National Center for Research Resources, a Clinical Translational Science Award to the University of Chicago. The content is solely the responsibility of the authors and does not necessarily represent the official views of the National Center of Research Resources or the National Institutes of Health.

References

 

1. Rabago D, Patterson JJ, Mundt M, et al. Dextrose prolotherapy for knee osteoarthritis: a randomized controlled trial. Ann Fam Med. 2013;11:229-237.

2. Peat G, McCarney R, Croft P. Knee pain and osteoarthritis in older adults: a review of community burden and current use of primary health care. Ann Rheum Dis. 2001;60:91-97.

3. Lawrence RC, Felson DT, Helmick CG, et al; National Arthritis Data Workgroup. Estimates of the prevalence of arthritis and other rheumatic conditions in the United States. Part II. Arthritis Rheum. 2008;58:26-35.

4. Rabago D, Slattengren A, Zgierska A. Prolotherapy in primary care practice. Prim Care. 2010;37:65-80.

5. Hackett GS, Hemwall GA, Montgomery GA. Ligament and Tendon Relaxation Treated by Prolotherapy. 5th ed. Oak Park, IL: Institute in Basic Life Principles; 1991.

6. Schultz LW. A treatment for subluxation of the temporomandibular joint. JAMA. 1937;109:1032-1035.

7. Rabago D, Best TM, Beamsley M, et al. A systematic review of prolotherapy for chronic musculoskeletal pain. Clin J Sport Med. 2005;15:376-380.

8. Reeves KD, Hassanein KM. Long-term effects of dextrose prolotherapy for anterior cruciate ligament laxity. Altern Ther Health Med. 2003;9:58-62.

9. Reeves KD, Hassanein K. Randomized prospective double-blind placebo-controlled study of dextrose prolotherapy for knee osteoarthritis with or without ACL laxity. Altern Ther Health Med. 2000;6:68-74,77-80.

10. Roos EM, Klässbo M, Lohmander LS. WOMAC osteoarthritis index. Reliability, validity, and responsiveness in patients with arthroscopically assessed osteoarthritis. Western Ontario and MacMaster Universities. Scand J Rheumatol. 1999;28:210-215.

11. Ehrich EW, Davies GM, Watson DJ, et al. Minimal perceptible clinical improvement with the Western Ontario and McMaster Universities osteoarthritis index questionnaire and global assessments in patients with osteoarthritis. J Rheumatol. 2000;27:2635-2641.

12. Rejeski WJ, Ettinger WH Jr, Shumaker S, et al. The evaluation of pain in patients with knee osteoarthritis: the knee pain scale. J Rheumatol. 1995;22:1124-1129.

13. Dorman TA. Prolotherapy: A survey. J Orthop Med. 1993;15:28-32.

14. Hauser RA, Hauser MA, Baird NM, et al. Prolotherapy as an alternative to surgery: A prospective pilot study of 34 patients from a private medical practice. J Prolotherapy. 2010;2:272-281.

References

 

1. Rabago D, Patterson JJ, Mundt M, et al. Dextrose prolotherapy for knee osteoarthritis: a randomized controlled trial. Ann Fam Med. 2013;11:229-237.

2. Peat G, McCarney R, Croft P. Knee pain and osteoarthritis in older adults: a review of community burden and current use of primary health care. Ann Rheum Dis. 2001;60:91-97.

3. Lawrence RC, Felson DT, Helmick CG, et al; National Arthritis Data Workgroup. Estimates of the prevalence of arthritis and other rheumatic conditions in the United States. Part II. Arthritis Rheum. 2008;58:26-35.

4. Rabago D, Slattengren A, Zgierska A. Prolotherapy in primary care practice. Prim Care. 2010;37:65-80.

5. Hackett GS, Hemwall GA, Montgomery GA. Ligament and Tendon Relaxation Treated by Prolotherapy. 5th ed. Oak Park, IL: Institute in Basic Life Principles; 1991.

6. Schultz LW. A treatment for subluxation of the temporomandibular joint. JAMA. 1937;109:1032-1035.

7. Rabago D, Best TM, Beamsley M, et al. A systematic review of prolotherapy for chronic musculoskeletal pain. Clin J Sport Med. 2005;15:376-380.

8. Reeves KD, Hassanein KM. Long-term effects of dextrose prolotherapy for anterior cruciate ligament laxity. Altern Ther Health Med. 2003;9:58-62.

9. Reeves KD, Hassanein K. Randomized prospective double-blind placebo-controlled study of dextrose prolotherapy for knee osteoarthritis with or without ACL laxity. Altern Ther Health Med. 2000;6:68-74,77-80.

10. Roos EM, Klässbo M, Lohmander LS. WOMAC osteoarthritis index. Reliability, validity, and responsiveness in patients with arthroscopically assessed osteoarthritis. Western Ontario and MacMaster Universities. Scand J Rheumatol. 1999;28:210-215.

11. Ehrich EW, Davies GM, Watson DJ, et al. Minimal perceptible clinical improvement with the Western Ontario and McMaster Universities osteoarthritis index questionnaire and global assessments in patients with osteoarthritis. J Rheumatol. 2000;27:2635-2641.

12. Rejeski WJ, Ettinger WH Jr, Shumaker S, et al. The evaluation of pain in patients with knee osteoarthritis: the knee pain scale. J Rheumatol. 1995;22:1124-1129.

13. Dorman TA. Prolotherapy: A survey. J Orthop Med. 1993;15:28-32.

14. Hauser RA, Hauser MA, Baird NM, et al. Prolotherapy as an alternative to surgery: A prospective pilot study of 34 patients from a private medical practice. J Prolotherapy. 2010;2:272-281.

Issue
The Journal of Family Practice - 63(4)
Issue
The Journal of Family Practice - 63(4)
Page Number
206-208
Page Number
206-208
Publications
Publications
Topics
Article Type
Display Headline
Prolotherapy: A nontraditional approach to knee osteoarthritis
Display Headline
Prolotherapy: A nontraditional approach to knee osteoarthritis
Legacy Keywords
Andrew H. Slattengren;, DO; Trent Christensen; MD; Shailendra Prasad; MBBS; MPH; Kohar Jones; MD; prolotherapy; knee osteoarthritis; OA; exercise; WOMAC; Western Ontario McMaster Universities Osteoarthritis Index; KPS; Knee Pain Scale
Legacy Keywords
Andrew H. Slattengren;, DO; Trent Christensen; MD; Shailendra Prasad; MBBS; MPH; Kohar Jones; MD; prolotherapy; knee osteoarthritis; OA; exercise; WOMAC; Western Ontario McMaster Universities Osteoarthritis Index; KPS; Knee Pain Scale
Sections
PURLs Copyright

Copyright © 2014 Family Physicians Inquiries Network. All rights reserved.

Disallow All Ads
Alternative CME
Use ProPublica
Hide sidebar & use full width
render the right sidebar.
Conference Recap Checkbox
Not Conference Recap
Clinical Edge
Display the Slideshow in this Article
Article PDF Media
Media Files

Should you screen for postpartum depression?

Article Type
Changed
Tue, 07/14/2020 - 15:31
Display Headline
Should you screen for postpartum depression?
Practice changer

Make depression screening a routine part of the postpartum visit.1

Strength of recommendation

B: Based on a single blinded randomized controlled trial (RCT).

Yawn BP, Dietrich AJ, Wollan P, et al. TRIPPD practices. TRIPPD: a practice-based network effectiveness study of postpartum depression screening and management. Ann Fam Med. 2012;10:320-329.

 

Illustrative case

A 20-year-old patient comes in at 6 weeks’ postpartum for routine care. Should you screen her for postpartum depression?

The incidence of depression in the first 3 months’ postpartum is estimated at about 14%,2,3 and the consequences can be severe. A new mom with a mood disorder in the first year of her child’s life can disrupt the mother-infant relationship, thereby contributing to both short- and long-term adverse outcomes for the child. These include behavior problems, low self-esteem, poor self-regulation, and an increased risk of impaired mental and motor development.4,5

Postpartum depression often goes undetected

Despite this correlation, postpartum depression is both under-recognized and undertreated.6 A prospective randomized study of 5169 women who were screened for postpartum depression bears this out. Researchers found that about one in 4 (26%) of the 674 mothers who had positive screens were not asked about their emotional state by their clinicians.2

This may be due to a lack of evidence of the efficacy of screening for postpartum depression. In the Healthy Start Depression Initiative (n=1336), universal screening—with referrals to mental health care outside of the primary care physicians’ offices—did not alter either the level of depressive symptoms over a 10-month period or depression treatment.7

The American College of Obstetricians and Gynecologists’ Committee on Obstetric Practice noted in 2010 (and reaffirmed in 2012) that there is insufficient evidence to support a firm recommendation for universal postpartum screening for depression.8 Neither the US Preventive Services Task Force nor the American Academy of Family Physicians has a specific recommendation regarding postpartum depression.

STUDY SUMMARY

Screening—with follow-up—leads to better outcomes

The study by Yawn et al included 28 family medicine practices in 21 states, randomized to either usual care or intervention.1 Eligible practices had to have provided well-baby or maternity care to more than 30 patients in the previous year, but not to routinely screen for postpartum depression. A total of 2343 women aged 18 years or older—all of whom were between 5 and 12 weeks’ postpartum and planning on continuing care at the family medicine practice at the time they entered the study—were enrolled.

Staff at the 14 intervention sites received training in postpartum depression screening and diagnosis. They were also given a set of tools to facilitate management of postpartum depression, including an outline for follow-up visits and scripts for nurses to use for telephone calls relating to antidepressants. Therapy options were left to the discretion of the physician and the patient, with the help of tools that detailed the doses and adverse effects of various medications and described cognitive behavioral therapy.

Low socioeconomic status has been shown to be a strong risk factor for the development of postpartum depression.At the start of the study, participants at all 28 sites were given survey packets containing the Edinburgh Postnatal Depression Scale (EPDS) and the 9-item Patient Health Questionnaire (PHQ-9). Patients subsequently received surveys by mail at 6 and 12 months’ postpartum for self-reported outcomes. Clinicians at the intervention sites had routine access to the EPDS and PHQ-9 scores; those at the usual care sites did not.

The primary outcome was ≥5 point drop in the PHQ-9 score from baseline at 6 or 12 months’ postpartum, considered to be an indicator of clinical improvement and/or response to therapy. The PHQ-9 is a validated measure of depression severity, with the proven ability to detect changes over time.9,10

Of the 2343 women initially enrolled in the study, 1897 (81%) provided outcome information and were included in the analysis. The rates of women with elevated depression scores (EPDS and/or PHQ-9 ≥10) at the start were comparable between the intervention and usual care groups (29.5% vs 25.8%, respectively).

Of those whose scores were initially elevated, 219 women in the intervention group and 178 women in the usual care group returned surveys at 12 months and were included in the final analysis. The results: 45% of those in the intervention group met the primary outcome—a decline in self-reported depressive symptom levels, as indicated by a PHQ-9 decrease ≥5 points—compared with 35% of the women in the usual care group (odds ratio, 1.8; 95% confidence interval, 1.14-2.9; P=.001). Not surprisingly, medical record review also indicated that those in the intervention group who initially had elevated depression scores were more likely to have received a diagnosis (66% vs 41%; P=.0006) and therapy (20% vs 11%; P=.02) for postpartum depression.

 

 

 

WHAT'S NEW?

We now have evidence of the efficacy of postpartum screening

This is the first large study of a primary care-based approach to screening, diagnosis, and management of postpartum depression to show any improvement in maternal outcomes at 12 months. Prior universal screening and referral support in the Healthy Start program was done by paraprofessionals, who referred women with positive screens for mental health care outside of the primary care setting and did not reduce the rate of depression in perinatal women.7

CAVEATS

Dropout rate, socioeconomic status may affect results

Among the women who initially were found to be positive for postpartum depression, 38% did not return questionnaires at 12 months’ postpartum. While this loss to follow-up is high, it is comparable to that of most effectiveness trials11 with similar rates in the intervention and usual care groups.

Within the intervention group, there was no statistical difference between women who did and did not return the questionnaires with regard to marital status, history of depression, income, or uninsured status. However, women in the usual care group who did not return the 12-month questionnaire were more likely to be poor (89% vs 57%; P<.01) and uninsured (49% vs 29%; P<.01) than those who did return the questionnaire.

The impact of these differences and the loss to follow-up in this study is unknown. However, low socioeconomic status has been shown to be a strong risk factor for the development of postpartum depression.12 The authors of the study suggest (and we agree) that the difference in socioeconomic status in women who did not return the questionnaire may underestimate the positive impact of this screening approach.

CHALLENGES TO IMPLEMENTATION

Screening requires extra work

Personnel at the intervention sites received a half day of training in postpartum depression screening, diagnosis, and nursing telephone follow-up. The workload at these sites also increased, as most women found to have postpartum depression received one to 2 follow-up telephone calls and an average of one to 2 follow-up visits after the start of therapy. These measures, while seemingly modest, could pose a challenge to implementation. This could potentially be alleviated by the additional payments for care coordination promised in the Patient Protection and Affordable Care Act.13

ACKNOWLEDGEMENT

The PURLs Surveillance System was developed in part by Grant Number UL1RR024999 from the National Center for Research Resources, a Clinical Translational Science Award to The University of Chicago. The content is solely the responsibility of the authors and does not necessarily represent the official views of the National Center of Research Resources or the National Institutes of Health.

Files
References

1. Yawn BP, Dietrich AJ, Wollan P, et al; TRIPPD practices. TRIPPD: a practice-based network effectiveness study of postpartum depression screening and management. Ann Fam Med. 2012;10:320-329.

2. Horowitz JA, Murphy CA, Gregory KE, et al. Best practices: community-based postpartum depression screening: results from the CARE study. Psychiatr Serv. 2009;60:1432-1434.

3. Gaynes BN, Gavin N, Meltzer-Brody S, et al. Perinatal Depression: Prevalence, Screening Accuracy, and Screening Outcomes: Summary. AHRQ Evidence Report Summaries. Rockville, MD: Agency for Healthcare Research and Quality; Feb, 2005.

4. Murray L, Cooper PJ. The impact of postpartum depression on child development. In: Goodyer I, ed. Aetiological Mechanisms in Developmental Psychopathology. Oxford, England: Oxford University Press; 2003.

5. Goodman SH, Gotlib IH. Risk for psychopathology in the children of depressed mothers: a developmental model for understanding mechanisms of transmission. Psychol Rev. 1999;106:458-490.

6. Pearlstein T, Howard M, Salisbury A, et al. Postpartum depression. Am J Obstet Gynecol. 2009;200:357-364.

7. Yonkers KA, Smith MV, Lin H, et al. Depression screening of perinatal women: an evaluation of the healthy start depression initiative. Psychiatr Serv. 2009;60:322-328.

8. American College of Obstetricians and Gynecologists. Committee on Obstetric Practice. Committee opinion no. 453: Screening for depression during and after pregnancy. Obstet Gynecol. 2010;115(2 pt 1):394-395.

9. Löwe B, Kroenke K, Herzog W, et al. Measuring depression outcome with a brief self-report instrument: sensitivity to change of the Patient Health Questionnaire (PHQ-9). J Affect Disord. 2004;81:61-66.

10. Kroenke K, Spitzer RL, Williams JB. The PHQ-9: validity of a brief depression severity measure. J Gen Intern Med. 2001;16:606–613.

11. Ribisl KM, Walton MA, Mowbray CT, et al. Minimizing participant attrition in panel studies through the use of effective retention and tracking strategies: review and recommendations. Eval Program Planning. 1996;19:1-25.

12. Dolbier CL, Rush TE, Sahadeo LS, et al; Community Health Network Investigators. Relationships of race and socioeconomic status to postpartum depressive symptoms in rural african american and non-Hispanic white women. Matern Child Health J. 2013;17:1277-1287.

13. Centers for Medicare and Medicaid Services. The Affordable Care Act: Helping Providers Help Patients. Available at: http://www.cms.gov/Medicare/Medicare-Fee-for-Service-Payment/ACO/downloads/ACO-Menu-Of-Options.pdf. Accessed September 6, 2013.

Article PDF
Author and Disclosure Information

Andrew H. Slattengren, DO
Shailendra Prasad, MBBS, MPH
M. Altaf Kaiseruddin, MD

North Memorial Family Medicine Residency, University of Minnesota, Minneapolis (Drs. Slattengren and Prasad); Department of Family Medicine, The University of Chicago (Dr. Kaiseruddin)

PURLs EDITOR
Bernard Ewigman, MD, MSPH
Department of Family Medicine, The University of Chicago

Issue
The Journal of Family Practice - 62(11)
Publications
Topics
Page Number
E1-E3
Legacy Keywords
Andrew H. Slattengren; DO; Shailendra Prasad; MBBS; MPH; M. Altaf Kaiseruddin; MD; postpartum; depression; screening; postpartumdepression; PURLs; Healthy Start Depression Initiative; Patient Protection and Affordable Care Act
Sections
Files
Files
Author and Disclosure Information

Andrew H. Slattengren, DO
Shailendra Prasad, MBBS, MPH
M. Altaf Kaiseruddin, MD

North Memorial Family Medicine Residency, University of Minnesota, Minneapolis (Drs. Slattengren and Prasad); Department of Family Medicine, The University of Chicago (Dr. Kaiseruddin)

PURLs EDITOR
Bernard Ewigman, MD, MSPH
Department of Family Medicine, The University of Chicago

Author and Disclosure Information

Andrew H. Slattengren, DO
Shailendra Prasad, MBBS, MPH
M. Altaf Kaiseruddin, MD

North Memorial Family Medicine Residency, University of Minnesota, Minneapolis (Drs. Slattengren and Prasad); Department of Family Medicine, The University of Chicago (Dr. Kaiseruddin)

PURLs EDITOR
Bernard Ewigman, MD, MSPH
Department of Family Medicine, The University of Chicago

Article PDF
Article PDF
Practice changer

Make depression screening a routine part of the postpartum visit.1

Strength of recommendation

B: Based on a single blinded randomized controlled trial (RCT).

Yawn BP, Dietrich AJ, Wollan P, et al. TRIPPD practices. TRIPPD: a practice-based network effectiveness study of postpartum depression screening and management. Ann Fam Med. 2012;10:320-329.

 

Illustrative case

A 20-year-old patient comes in at 6 weeks’ postpartum for routine care. Should you screen her for postpartum depression?

The incidence of depression in the first 3 months’ postpartum is estimated at about 14%,2,3 and the consequences can be severe. A new mom with a mood disorder in the first year of her child’s life can disrupt the mother-infant relationship, thereby contributing to both short- and long-term adverse outcomes for the child. These include behavior problems, low self-esteem, poor self-regulation, and an increased risk of impaired mental and motor development.4,5

Postpartum depression often goes undetected

Despite this correlation, postpartum depression is both under-recognized and undertreated.6 A prospective randomized study of 5169 women who were screened for postpartum depression bears this out. Researchers found that about one in 4 (26%) of the 674 mothers who had positive screens were not asked about their emotional state by their clinicians.2

This may be due to a lack of evidence of the efficacy of screening for postpartum depression. In the Healthy Start Depression Initiative (n=1336), universal screening—with referrals to mental health care outside of the primary care physicians’ offices—did not alter either the level of depressive symptoms over a 10-month period or depression treatment.7

The American College of Obstetricians and Gynecologists’ Committee on Obstetric Practice noted in 2010 (and reaffirmed in 2012) that there is insufficient evidence to support a firm recommendation for universal postpartum screening for depression.8 Neither the US Preventive Services Task Force nor the American Academy of Family Physicians has a specific recommendation regarding postpartum depression.

STUDY SUMMARY

Screening—with follow-up—leads to better outcomes

The study by Yawn et al included 28 family medicine practices in 21 states, randomized to either usual care or intervention.1 Eligible practices had to have provided well-baby or maternity care to more than 30 patients in the previous year, but not to routinely screen for postpartum depression. A total of 2343 women aged 18 years or older—all of whom were between 5 and 12 weeks’ postpartum and planning on continuing care at the family medicine practice at the time they entered the study—were enrolled.

Staff at the 14 intervention sites received training in postpartum depression screening and diagnosis. They were also given a set of tools to facilitate management of postpartum depression, including an outline for follow-up visits and scripts for nurses to use for telephone calls relating to antidepressants. Therapy options were left to the discretion of the physician and the patient, with the help of tools that detailed the doses and adverse effects of various medications and described cognitive behavioral therapy.

Low socioeconomic status has been shown to be a strong risk factor for the development of postpartum depression.At the start of the study, participants at all 28 sites were given survey packets containing the Edinburgh Postnatal Depression Scale (EPDS) and the 9-item Patient Health Questionnaire (PHQ-9). Patients subsequently received surveys by mail at 6 and 12 months’ postpartum for self-reported outcomes. Clinicians at the intervention sites had routine access to the EPDS and PHQ-9 scores; those at the usual care sites did not.

The primary outcome was ≥5 point drop in the PHQ-9 score from baseline at 6 or 12 months’ postpartum, considered to be an indicator of clinical improvement and/or response to therapy. The PHQ-9 is a validated measure of depression severity, with the proven ability to detect changes over time.9,10

Of the 2343 women initially enrolled in the study, 1897 (81%) provided outcome information and were included in the analysis. The rates of women with elevated depression scores (EPDS and/or PHQ-9 ≥10) at the start were comparable between the intervention and usual care groups (29.5% vs 25.8%, respectively).

Of those whose scores were initially elevated, 219 women in the intervention group and 178 women in the usual care group returned surveys at 12 months and were included in the final analysis. The results: 45% of those in the intervention group met the primary outcome—a decline in self-reported depressive symptom levels, as indicated by a PHQ-9 decrease ≥5 points—compared with 35% of the women in the usual care group (odds ratio, 1.8; 95% confidence interval, 1.14-2.9; P=.001). Not surprisingly, medical record review also indicated that those in the intervention group who initially had elevated depression scores were more likely to have received a diagnosis (66% vs 41%; P=.0006) and therapy (20% vs 11%; P=.02) for postpartum depression.

 

 

 

WHAT'S NEW?

We now have evidence of the efficacy of postpartum screening

This is the first large study of a primary care-based approach to screening, diagnosis, and management of postpartum depression to show any improvement in maternal outcomes at 12 months. Prior universal screening and referral support in the Healthy Start program was done by paraprofessionals, who referred women with positive screens for mental health care outside of the primary care setting and did not reduce the rate of depression in perinatal women.7

CAVEATS

Dropout rate, socioeconomic status may affect results

Among the women who initially were found to be positive for postpartum depression, 38% did not return questionnaires at 12 months’ postpartum. While this loss to follow-up is high, it is comparable to that of most effectiveness trials11 with similar rates in the intervention and usual care groups.

Within the intervention group, there was no statistical difference between women who did and did not return the questionnaires with regard to marital status, history of depression, income, or uninsured status. However, women in the usual care group who did not return the 12-month questionnaire were more likely to be poor (89% vs 57%; P<.01) and uninsured (49% vs 29%; P<.01) than those who did return the questionnaire.

The impact of these differences and the loss to follow-up in this study is unknown. However, low socioeconomic status has been shown to be a strong risk factor for the development of postpartum depression.12 The authors of the study suggest (and we agree) that the difference in socioeconomic status in women who did not return the questionnaire may underestimate the positive impact of this screening approach.

CHALLENGES TO IMPLEMENTATION

Screening requires extra work

Personnel at the intervention sites received a half day of training in postpartum depression screening, diagnosis, and nursing telephone follow-up. The workload at these sites also increased, as most women found to have postpartum depression received one to 2 follow-up telephone calls and an average of one to 2 follow-up visits after the start of therapy. These measures, while seemingly modest, could pose a challenge to implementation. This could potentially be alleviated by the additional payments for care coordination promised in the Patient Protection and Affordable Care Act.13

ACKNOWLEDGEMENT

The PURLs Surveillance System was developed in part by Grant Number UL1RR024999 from the National Center for Research Resources, a Clinical Translational Science Award to The University of Chicago. The content is solely the responsibility of the authors and does not necessarily represent the official views of the National Center of Research Resources or the National Institutes of Health.

Practice changer

Make depression screening a routine part of the postpartum visit.1

Strength of recommendation

B: Based on a single blinded randomized controlled trial (RCT).

Yawn BP, Dietrich AJ, Wollan P, et al. TRIPPD practices. TRIPPD: a practice-based network effectiveness study of postpartum depression screening and management. Ann Fam Med. 2012;10:320-329.

 

Illustrative case

A 20-year-old patient comes in at 6 weeks’ postpartum for routine care. Should you screen her for postpartum depression?

The incidence of depression in the first 3 months’ postpartum is estimated at about 14%,2,3 and the consequences can be severe. A new mom with a mood disorder in the first year of her child’s life can disrupt the mother-infant relationship, thereby contributing to both short- and long-term adverse outcomes for the child. These include behavior problems, low self-esteem, poor self-regulation, and an increased risk of impaired mental and motor development.4,5

Postpartum depression often goes undetected

Despite this correlation, postpartum depression is both under-recognized and undertreated.6 A prospective randomized study of 5169 women who were screened for postpartum depression bears this out. Researchers found that about one in 4 (26%) of the 674 mothers who had positive screens were not asked about their emotional state by their clinicians.2

This may be due to a lack of evidence of the efficacy of screening for postpartum depression. In the Healthy Start Depression Initiative (n=1336), universal screening—with referrals to mental health care outside of the primary care physicians’ offices—did not alter either the level of depressive symptoms over a 10-month period or depression treatment.7

The American College of Obstetricians and Gynecologists’ Committee on Obstetric Practice noted in 2010 (and reaffirmed in 2012) that there is insufficient evidence to support a firm recommendation for universal postpartum screening for depression.8 Neither the US Preventive Services Task Force nor the American Academy of Family Physicians has a specific recommendation regarding postpartum depression.

STUDY SUMMARY

Screening—with follow-up—leads to better outcomes

The study by Yawn et al included 28 family medicine practices in 21 states, randomized to either usual care or intervention.1 Eligible practices had to have provided well-baby or maternity care to more than 30 patients in the previous year, but not to routinely screen for postpartum depression. A total of 2343 women aged 18 years or older—all of whom were between 5 and 12 weeks’ postpartum and planning on continuing care at the family medicine practice at the time they entered the study—were enrolled.

Staff at the 14 intervention sites received training in postpartum depression screening and diagnosis. They were also given a set of tools to facilitate management of postpartum depression, including an outline for follow-up visits and scripts for nurses to use for telephone calls relating to antidepressants. Therapy options were left to the discretion of the physician and the patient, with the help of tools that detailed the doses and adverse effects of various medications and described cognitive behavioral therapy.

Low socioeconomic status has been shown to be a strong risk factor for the development of postpartum depression.At the start of the study, participants at all 28 sites were given survey packets containing the Edinburgh Postnatal Depression Scale (EPDS) and the 9-item Patient Health Questionnaire (PHQ-9). Patients subsequently received surveys by mail at 6 and 12 months’ postpartum for self-reported outcomes. Clinicians at the intervention sites had routine access to the EPDS and PHQ-9 scores; those at the usual care sites did not.

The primary outcome was ≥5 point drop in the PHQ-9 score from baseline at 6 or 12 months’ postpartum, considered to be an indicator of clinical improvement and/or response to therapy. The PHQ-9 is a validated measure of depression severity, with the proven ability to detect changes over time.9,10

Of the 2343 women initially enrolled in the study, 1897 (81%) provided outcome information and were included in the analysis. The rates of women with elevated depression scores (EPDS and/or PHQ-9 ≥10) at the start were comparable between the intervention and usual care groups (29.5% vs 25.8%, respectively).

Of those whose scores were initially elevated, 219 women in the intervention group and 178 women in the usual care group returned surveys at 12 months and were included in the final analysis. The results: 45% of those in the intervention group met the primary outcome—a decline in self-reported depressive symptom levels, as indicated by a PHQ-9 decrease ≥5 points—compared with 35% of the women in the usual care group (odds ratio, 1.8; 95% confidence interval, 1.14-2.9; P=.001). Not surprisingly, medical record review also indicated that those in the intervention group who initially had elevated depression scores were more likely to have received a diagnosis (66% vs 41%; P=.0006) and therapy (20% vs 11%; P=.02) for postpartum depression.

 

 

 

WHAT'S NEW?

We now have evidence of the efficacy of postpartum screening

This is the first large study of a primary care-based approach to screening, diagnosis, and management of postpartum depression to show any improvement in maternal outcomes at 12 months. Prior universal screening and referral support in the Healthy Start program was done by paraprofessionals, who referred women with positive screens for mental health care outside of the primary care setting and did not reduce the rate of depression in perinatal women.7

CAVEATS

Dropout rate, socioeconomic status may affect results

Among the women who initially were found to be positive for postpartum depression, 38% did not return questionnaires at 12 months’ postpartum. While this loss to follow-up is high, it is comparable to that of most effectiveness trials11 with similar rates in the intervention and usual care groups.

Within the intervention group, there was no statistical difference between women who did and did not return the questionnaires with regard to marital status, history of depression, income, or uninsured status. However, women in the usual care group who did not return the 12-month questionnaire were more likely to be poor (89% vs 57%; P<.01) and uninsured (49% vs 29%; P<.01) than those who did return the questionnaire.

The impact of these differences and the loss to follow-up in this study is unknown. However, low socioeconomic status has been shown to be a strong risk factor for the development of postpartum depression.12 The authors of the study suggest (and we agree) that the difference in socioeconomic status in women who did not return the questionnaire may underestimate the positive impact of this screening approach.

CHALLENGES TO IMPLEMENTATION

Screening requires extra work

Personnel at the intervention sites received a half day of training in postpartum depression screening, diagnosis, and nursing telephone follow-up. The workload at these sites also increased, as most women found to have postpartum depression received one to 2 follow-up telephone calls and an average of one to 2 follow-up visits after the start of therapy. These measures, while seemingly modest, could pose a challenge to implementation. This could potentially be alleviated by the additional payments for care coordination promised in the Patient Protection and Affordable Care Act.13

ACKNOWLEDGEMENT

The PURLs Surveillance System was developed in part by Grant Number UL1RR024999 from the National Center for Research Resources, a Clinical Translational Science Award to The University of Chicago. The content is solely the responsibility of the authors and does not necessarily represent the official views of the National Center of Research Resources or the National Institutes of Health.

References

1. Yawn BP, Dietrich AJ, Wollan P, et al; TRIPPD practices. TRIPPD: a practice-based network effectiveness study of postpartum depression screening and management. Ann Fam Med. 2012;10:320-329.

2. Horowitz JA, Murphy CA, Gregory KE, et al. Best practices: community-based postpartum depression screening: results from the CARE study. Psychiatr Serv. 2009;60:1432-1434.

3. Gaynes BN, Gavin N, Meltzer-Brody S, et al. Perinatal Depression: Prevalence, Screening Accuracy, and Screening Outcomes: Summary. AHRQ Evidence Report Summaries. Rockville, MD: Agency for Healthcare Research and Quality; Feb, 2005.

4. Murray L, Cooper PJ. The impact of postpartum depression on child development. In: Goodyer I, ed. Aetiological Mechanisms in Developmental Psychopathology. Oxford, England: Oxford University Press; 2003.

5. Goodman SH, Gotlib IH. Risk for psychopathology in the children of depressed mothers: a developmental model for understanding mechanisms of transmission. Psychol Rev. 1999;106:458-490.

6. Pearlstein T, Howard M, Salisbury A, et al. Postpartum depression. Am J Obstet Gynecol. 2009;200:357-364.

7. Yonkers KA, Smith MV, Lin H, et al. Depression screening of perinatal women: an evaluation of the healthy start depression initiative. Psychiatr Serv. 2009;60:322-328.

8. American College of Obstetricians and Gynecologists. Committee on Obstetric Practice. Committee opinion no. 453: Screening for depression during and after pregnancy. Obstet Gynecol. 2010;115(2 pt 1):394-395.

9. Löwe B, Kroenke K, Herzog W, et al. Measuring depression outcome with a brief self-report instrument: sensitivity to change of the Patient Health Questionnaire (PHQ-9). J Affect Disord. 2004;81:61-66.

10. Kroenke K, Spitzer RL, Williams JB. The PHQ-9: validity of a brief depression severity measure. J Gen Intern Med. 2001;16:606–613.

11. Ribisl KM, Walton MA, Mowbray CT, et al. Minimizing participant attrition in panel studies through the use of effective retention and tracking strategies: review and recommendations. Eval Program Planning. 1996;19:1-25.

12. Dolbier CL, Rush TE, Sahadeo LS, et al; Community Health Network Investigators. Relationships of race and socioeconomic status to postpartum depressive symptoms in rural african american and non-Hispanic white women. Matern Child Health J. 2013;17:1277-1287.

13. Centers for Medicare and Medicaid Services. The Affordable Care Act: Helping Providers Help Patients. Available at: http://www.cms.gov/Medicare/Medicare-Fee-for-Service-Payment/ACO/downloads/ACO-Menu-Of-Options.pdf. Accessed September 6, 2013.

References

1. Yawn BP, Dietrich AJ, Wollan P, et al; TRIPPD practices. TRIPPD: a practice-based network effectiveness study of postpartum depression screening and management. Ann Fam Med. 2012;10:320-329.

2. Horowitz JA, Murphy CA, Gregory KE, et al. Best practices: community-based postpartum depression screening: results from the CARE study. Psychiatr Serv. 2009;60:1432-1434.

3. Gaynes BN, Gavin N, Meltzer-Brody S, et al. Perinatal Depression: Prevalence, Screening Accuracy, and Screening Outcomes: Summary. AHRQ Evidence Report Summaries. Rockville, MD: Agency for Healthcare Research and Quality; Feb, 2005.

4. Murray L, Cooper PJ. The impact of postpartum depression on child development. In: Goodyer I, ed. Aetiological Mechanisms in Developmental Psychopathology. Oxford, England: Oxford University Press; 2003.

5. Goodman SH, Gotlib IH. Risk for psychopathology in the children of depressed mothers: a developmental model for understanding mechanisms of transmission. Psychol Rev. 1999;106:458-490.

6. Pearlstein T, Howard M, Salisbury A, et al. Postpartum depression. Am J Obstet Gynecol. 2009;200:357-364.

7. Yonkers KA, Smith MV, Lin H, et al. Depression screening of perinatal women: an evaluation of the healthy start depression initiative. Psychiatr Serv. 2009;60:322-328.

8. American College of Obstetricians and Gynecologists. Committee on Obstetric Practice. Committee opinion no. 453: Screening for depression during and after pregnancy. Obstet Gynecol. 2010;115(2 pt 1):394-395.

9. Löwe B, Kroenke K, Herzog W, et al. Measuring depression outcome with a brief self-report instrument: sensitivity to change of the Patient Health Questionnaire (PHQ-9). J Affect Disord. 2004;81:61-66.

10. Kroenke K, Spitzer RL, Williams JB. The PHQ-9: validity of a brief depression severity measure. J Gen Intern Med. 2001;16:606–613.

11. Ribisl KM, Walton MA, Mowbray CT, et al. Minimizing participant attrition in panel studies through the use of effective retention and tracking strategies: review and recommendations. Eval Program Planning. 1996;19:1-25.

12. Dolbier CL, Rush TE, Sahadeo LS, et al; Community Health Network Investigators. Relationships of race and socioeconomic status to postpartum depressive symptoms in rural african american and non-Hispanic white women. Matern Child Health J. 2013;17:1277-1287.

13. Centers for Medicare and Medicaid Services. The Affordable Care Act: Helping Providers Help Patients. Available at: http://www.cms.gov/Medicare/Medicare-Fee-for-Service-Payment/ACO/downloads/ACO-Menu-Of-Options.pdf. Accessed September 6, 2013.

Issue
The Journal of Family Practice - 62(11)
Issue
The Journal of Family Practice - 62(11)
Page Number
E1-E3
Page Number
E1-E3
Publications
Publications
Topics
Article Type
Display Headline
Should you screen for postpartum depression?
Display Headline
Should you screen for postpartum depression?
Legacy Keywords
Andrew H. Slattengren; DO; Shailendra Prasad; MBBS; MPH; M. Altaf Kaiseruddin; MD; postpartum; depression; screening; postpartumdepression; PURLs; Healthy Start Depression Initiative; Patient Protection and Affordable Care Act
Legacy Keywords
Andrew H. Slattengren; DO; Shailendra Prasad; MBBS; MPH; M. Altaf Kaiseruddin; MD; postpartum; depression; screening; postpartumdepression; PURLs; Healthy Start Depression Initiative; Patient Protection and Affordable Care Act
Sections
PURLs Copyright

Copyright © 2013 Family Physicians Inquiries Network. All rights reserved.

Disallow All Ads
Alternative CME
Use ProPublica
Hide sidebar & use full width
render the right sidebar.
Conference Recap Checkbox
Not Conference Recap
Clinical Edge
Display the Slideshow in this Article
Article PDF Media
Media Files

Easing the Discomfort of a Speculum Exam

Article Type
Changed
Tue, 12/13/2016 - 12:08
Display Headline
Easing the Discomfort of a Speculum Exam
Applying a small amount of gel to the blades of a speculum before insertion won’t alter test results—but will reduce the patient’s discomfort.

PRACTICE CHANGER

Put lubricating gel, not water, on the speculum every time you do a pelvic exam.1

STRENGTH OF ­RECOMMENDATION

B: Based on one good-quality, randomized controlled trial.

Hill DA, Lamvu G. Effect of lubricating gel on patient comfort during vaginal speculum examination. Obstet Gynecol. 2012;119 (2 pt 1):
227-231.

ILLUSTRATIVE CASE

A 24-year-old woman presents for an annual exam, including Pap smear and testing for sexually transmitted diseases. She tells you how painful her previous speculum exam was and how worried she is about having another. To ease her discomfort, should you apply lubricating gel or water to the speculum before vaginal insertion?

Physiology teaches us that vaginal entry requires lubrication. But traditional teaching has held that lubricating gel on a speculum can interfere with the results of a Pap smear and chlamydia tesing.2 Pelvic exams performed without lubricating gel on the speculum can cause significant discomfort—possibly bad enough to prevent some women from undergoing the recommended screening tests.3

Until now, we’ve only ­evaluated gel’s impact on test results

Studies comparing lubricating gel and water have conclusively shown that a small amount of gel, used on the outside of the speculum blades, does not interfere with either Pap testing or detection of Chlamydia trachomatis.4,5 One liquid-based cytology manufacturer, however, discourages the use of lubricants with “carbomers” or “carbopol polymers,” but states that water-based lubricants have not been shown to interfere with Pap smear results.6 No studies have evaluated lubricants from a patient perspective—until now.

STUDY SUMMARY

Lubricating gel eases discomfort

The study by Hill and Lamvu1 was a six-month, single-blind, randomized trial of women ages 18 to 50 who sought care at an Orlando obstetrics and gynecology department for conditions requiring vaginal speculum examination.1 The study excluded women who might have an altered perception of pain during speculum insertion: those who were menopausal, pregnant, or within six weeks’ postpartum; had dyspareunia, vaginitis, vulvar pain, or vulvar lesions; were undergoing a procedure; or had never had vaginal intercourse. Women who were not fluent in English were excluded, as well.

The study included 120 women who underwent computer-generated randomization into two groups with no marked differences in demographics. A single examiner performed all the speculum exams, using a standard protocol with a medium-size Graves speculum. The examiner applied 0.3 mL water-based lubricating gel to the speculum before insertion for the women in one group, and used 3 mL water for the other.

Immediately after the speculum was inserted and opened—before the examiner attempted to visualize the cervix—patients were given a visual analog scale and told to indicate the level of pain with insertion, using a scale of 0 (no pain) to 10 (the worst pain imaginable). The gel group had lower pain scores for speculum insertion compared with the water group (1.41 ± 1.55 vs 2.15 ± 1.93), a statistically significant difference of 0.74. Twenty of the 59 patients in the gel group (33.9%) rated their pain as 0, compared with six of 60 (10%) in the water group.

Although pain, rather than sampling quality, was the primary outcome of the study, the authors also reported that all of the women who underwent Pap screening (73) had adequate cytology.

WHAT’S NEW?

It’s time for lubrication to become standard practice

This trial is the first to study speculum lubrication from this patient-oriented outcome and to show that women experience less pain when lubricating gel, rather than water, is applied to the speculum. This knowledge, combined with previous studies showing that a small amount of water-based lubricating gel does not interfere with liquid-based cytology or chlamydia test results, should make the use of lubricating gel standard practice when performing speculum examinations.

CAVEATS

We see no downside

The exclusion criteria of this study were meant to eliminate women who had an altered perception of pain that could skew study results. Yet those who met the exclusion criteria may also benefit from a pelvic exam with gel lubrication. We see no harm in trying a small amount of lubricant when examining them, as well.

In addition, the study did not compare various types and sizes of specula. However, we see no reason why the benefit of a gel lubricant would be limited to the type of speculum used by the examiner.

Studies in emergency departments that have used visual analog scales to measure interventions that decrease pain have used a mean difference of 0.9 as “clinically meaningful.” 7,8 The 0.74 difference observed in this study does not rise to that level. However, one in three patients in the gel group marked 0 on the pain scale, indicating that they had no pain, compared with only one in 10 in the water group.

 

 

We believe that the higher proportion of women experiencing no pain and the mean difference of 0.74 on the pain scale (both statistically significant), combined with the lack of risk associated with the use of a water-based lubricant, makes this a clinically useful practice changer.

CHALLENGES TO IMPLEMENTATION

There aren’t any

Other than clinical inertia, we see no challenges to the implementation of this recommendation.

References

1. Hill DA, Lamvu G. Effect of lubricating gel
on patient comfort during vaginal specu-
lum examination. Obstet Gynecol. 2012;119 (2 pt 1):227-231.

2. Harmanli O, Jones KA. Using lubricant for speculum insertion. Obstet Gynecol. 2010; 116:415-417.

3. Hoyo C, Yarnall KSH, Skinner CS, et al. Pain predicts non-adherence to pap smear screening among middle-aged African American women. Prev Med. 2005;41:439-445.

4. Amies AE, Miller L, Lee S, et al. The effect of vaginal speculum lubrication on the rate of unsatisfactory cervical cytology diagnosis. Obstet Gynecol. 2002;100:889-892.

5. Griffith WF, Stuart GS, Gluck KL, et al. Vaginal speculum lubrication and its effects on cervical cytology and microbiology. Contraception. 2005;72:60-64.

6. Evantash E. Lubricant use during Pap test collection. Bedford, Mass: Hologic, Inc; 2009. www.thinprep.com/hcp/specimen_collection/common_questions.html. Ac-
cessed October 9, 2013.

7. Bijur PE, Silver W, Gallagher EJ. Reliability of the visual analog scale for measurement of acute pain. Acad Emerg Med. 2001;8:1153-1157.

8. Kelly AM. Does the clinically significant difference in visual analog scale pain scores vary with gender, age, or cause of pain? Acad Emerg Med. 1998;5:1086-1090.

Acknowledgement

The PURLs Surveillance System is supported in part by Grant Number UL 1RR 024999 from the National Center for Research Resources, a Clinical Translational Science Award to the University of Chicago. The content is solely the responsibility of the authors and does not necessarily represent the official views of the National Center for Research Resources or the National Institutes of Health.

Copyright © 2013. The Family Physicians Inquiries Network. All rights reserved.

Reprinted with permission from the Family Physicians Inquiries Network and The Journal of Family Practice. 2013;61(9):E1-E3.

Author and Disclosure Information

Nicole VandenBerg, MD, Shailendra Prasad, MBBS, MPH

Issue
Clinician Reviews - 23(11)
Publications
Topics
Page Number
28
Legacy Keywords
Speculum, exam, comfort, gel, lubrication, pelvic exam, pap smear, annual,
Sections
Author and Disclosure Information

Nicole VandenBerg, MD, Shailendra Prasad, MBBS, MPH

Author and Disclosure Information

Nicole VandenBerg, MD, Shailendra Prasad, MBBS, MPH

Applying a small amount of gel to the blades of a speculum before insertion won’t alter test results—but will reduce the patient’s discomfort.
Applying a small amount of gel to the blades of a speculum before insertion won’t alter test results—but will reduce the patient’s discomfort.

PRACTICE CHANGER

Put lubricating gel, not water, on the speculum every time you do a pelvic exam.1

STRENGTH OF ­RECOMMENDATION

B: Based on one good-quality, randomized controlled trial.

Hill DA, Lamvu G. Effect of lubricating gel on patient comfort during vaginal speculum examination. Obstet Gynecol. 2012;119 (2 pt 1):
227-231.

ILLUSTRATIVE CASE

A 24-year-old woman presents for an annual exam, including Pap smear and testing for sexually transmitted diseases. She tells you how painful her previous speculum exam was and how worried she is about having another. To ease her discomfort, should you apply lubricating gel or water to the speculum before vaginal insertion?

Physiology teaches us that vaginal entry requires lubrication. But traditional teaching has held that lubricating gel on a speculum can interfere with the results of a Pap smear and chlamydia tesing.2 Pelvic exams performed without lubricating gel on the speculum can cause significant discomfort—possibly bad enough to prevent some women from undergoing the recommended screening tests.3

Until now, we’ve only ­evaluated gel’s impact on test results

Studies comparing lubricating gel and water have conclusively shown that a small amount of gel, used on the outside of the speculum blades, does not interfere with either Pap testing or detection of Chlamydia trachomatis.4,5 One liquid-based cytology manufacturer, however, discourages the use of lubricants with “carbomers” or “carbopol polymers,” but states that water-based lubricants have not been shown to interfere with Pap smear results.6 No studies have evaluated lubricants from a patient perspective—until now.

STUDY SUMMARY

Lubricating gel eases discomfort

The study by Hill and Lamvu1 was a six-month, single-blind, randomized trial of women ages 18 to 50 who sought care at an Orlando obstetrics and gynecology department for conditions requiring vaginal speculum examination.1 The study excluded women who might have an altered perception of pain during speculum insertion: those who were menopausal, pregnant, or within six weeks’ postpartum; had dyspareunia, vaginitis, vulvar pain, or vulvar lesions; were undergoing a procedure; or had never had vaginal intercourse. Women who were not fluent in English were excluded, as well.

The study included 120 women who underwent computer-generated randomization into two groups with no marked differences in demographics. A single examiner performed all the speculum exams, using a standard protocol with a medium-size Graves speculum. The examiner applied 0.3 mL water-based lubricating gel to the speculum before insertion for the women in one group, and used 3 mL water for the other.

Immediately after the speculum was inserted and opened—before the examiner attempted to visualize the cervix—patients were given a visual analog scale and told to indicate the level of pain with insertion, using a scale of 0 (no pain) to 10 (the worst pain imaginable). The gel group had lower pain scores for speculum insertion compared with the water group (1.41 ± 1.55 vs 2.15 ± 1.93), a statistically significant difference of 0.74. Twenty of the 59 patients in the gel group (33.9%) rated their pain as 0, compared with six of 60 (10%) in the water group.

Although pain, rather than sampling quality, was the primary outcome of the study, the authors also reported that all of the women who underwent Pap screening (73) had adequate cytology.

WHAT’S NEW?

It’s time for lubrication to become standard practice

This trial is the first to study speculum lubrication from this patient-oriented outcome and to show that women experience less pain when lubricating gel, rather than water, is applied to the speculum. This knowledge, combined with previous studies showing that a small amount of water-based lubricating gel does not interfere with liquid-based cytology or chlamydia test results, should make the use of lubricating gel standard practice when performing speculum examinations.

CAVEATS

We see no downside

The exclusion criteria of this study were meant to eliminate women who had an altered perception of pain that could skew study results. Yet those who met the exclusion criteria may also benefit from a pelvic exam with gel lubrication. We see no harm in trying a small amount of lubricant when examining them, as well.

In addition, the study did not compare various types and sizes of specula. However, we see no reason why the benefit of a gel lubricant would be limited to the type of speculum used by the examiner.

Studies in emergency departments that have used visual analog scales to measure interventions that decrease pain have used a mean difference of 0.9 as “clinically meaningful.” 7,8 The 0.74 difference observed in this study does not rise to that level. However, one in three patients in the gel group marked 0 on the pain scale, indicating that they had no pain, compared with only one in 10 in the water group.

 

 

We believe that the higher proportion of women experiencing no pain and the mean difference of 0.74 on the pain scale (both statistically significant), combined with the lack of risk associated with the use of a water-based lubricant, makes this a clinically useful practice changer.

CHALLENGES TO IMPLEMENTATION

There aren’t any

Other than clinical inertia, we see no challenges to the implementation of this recommendation.

References

1. Hill DA, Lamvu G. Effect of lubricating gel
on patient comfort during vaginal specu-
lum examination. Obstet Gynecol. 2012;119 (2 pt 1):227-231.

2. Harmanli O, Jones KA. Using lubricant for speculum insertion. Obstet Gynecol. 2010; 116:415-417.

3. Hoyo C, Yarnall KSH, Skinner CS, et al. Pain predicts non-adherence to pap smear screening among middle-aged African American women. Prev Med. 2005;41:439-445.

4. Amies AE, Miller L, Lee S, et al. The effect of vaginal speculum lubrication on the rate of unsatisfactory cervical cytology diagnosis. Obstet Gynecol. 2002;100:889-892.

5. Griffith WF, Stuart GS, Gluck KL, et al. Vaginal speculum lubrication and its effects on cervical cytology and microbiology. Contraception. 2005;72:60-64.

6. Evantash E. Lubricant use during Pap test collection. Bedford, Mass: Hologic, Inc; 2009. www.thinprep.com/hcp/specimen_collection/common_questions.html. Ac-
cessed October 9, 2013.

7. Bijur PE, Silver W, Gallagher EJ. Reliability of the visual analog scale for measurement of acute pain. Acad Emerg Med. 2001;8:1153-1157.

8. Kelly AM. Does the clinically significant difference in visual analog scale pain scores vary with gender, age, or cause of pain? Acad Emerg Med. 1998;5:1086-1090.

Acknowledgement

The PURLs Surveillance System is supported in part by Grant Number UL 1RR 024999 from the National Center for Research Resources, a Clinical Translational Science Award to the University of Chicago. The content is solely the responsibility of the authors and does not necessarily represent the official views of the National Center for Research Resources or the National Institutes of Health.

Copyright © 2013. The Family Physicians Inquiries Network. All rights reserved.

Reprinted with permission from the Family Physicians Inquiries Network and The Journal of Family Practice. 2013;61(9):E1-E3.

PRACTICE CHANGER

Put lubricating gel, not water, on the speculum every time you do a pelvic exam.1

STRENGTH OF ­RECOMMENDATION

B: Based on one good-quality, randomized controlled trial.

Hill DA, Lamvu G. Effect of lubricating gel on patient comfort during vaginal speculum examination. Obstet Gynecol. 2012;119 (2 pt 1):
227-231.

ILLUSTRATIVE CASE

A 24-year-old woman presents for an annual exam, including Pap smear and testing for sexually transmitted diseases. She tells you how painful her previous speculum exam was and how worried she is about having another. To ease her discomfort, should you apply lubricating gel or water to the speculum before vaginal insertion?

Physiology teaches us that vaginal entry requires lubrication. But traditional teaching has held that lubricating gel on a speculum can interfere with the results of a Pap smear and chlamydia tesing.2 Pelvic exams performed without lubricating gel on the speculum can cause significant discomfort—possibly bad enough to prevent some women from undergoing the recommended screening tests.3

Until now, we’ve only ­evaluated gel’s impact on test results

Studies comparing lubricating gel and water have conclusively shown that a small amount of gel, used on the outside of the speculum blades, does not interfere with either Pap testing or detection of Chlamydia trachomatis.4,5 One liquid-based cytology manufacturer, however, discourages the use of lubricants with “carbomers” or “carbopol polymers,” but states that water-based lubricants have not been shown to interfere with Pap smear results.6 No studies have evaluated lubricants from a patient perspective—until now.

STUDY SUMMARY

Lubricating gel eases discomfort

The study by Hill and Lamvu1 was a six-month, single-blind, randomized trial of women ages 18 to 50 who sought care at an Orlando obstetrics and gynecology department for conditions requiring vaginal speculum examination.1 The study excluded women who might have an altered perception of pain during speculum insertion: those who were menopausal, pregnant, or within six weeks’ postpartum; had dyspareunia, vaginitis, vulvar pain, or vulvar lesions; were undergoing a procedure; or had never had vaginal intercourse. Women who were not fluent in English were excluded, as well.

The study included 120 women who underwent computer-generated randomization into two groups with no marked differences in demographics. A single examiner performed all the speculum exams, using a standard protocol with a medium-size Graves speculum. The examiner applied 0.3 mL water-based lubricating gel to the speculum before insertion for the women in one group, and used 3 mL water for the other.

Immediately after the speculum was inserted and opened—before the examiner attempted to visualize the cervix—patients were given a visual analog scale and told to indicate the level of pain with insertion, using a scale of 0 (no pain) to 10 (the worst pain imaginable). The gel group had lower pain scores for speculum insertion compared with the water group (1.41 ± 1.55 vs 2.15 ± 1.93), a statistically significant difference of 0.74. Twenty of the 59 patients in the gel group (33.9%) rated their pain as 0, compared with six of 60 (10%) in the water group.

Although pain, rather than sampling quality, was the primary outcome of the study, the authors also reported that all of the women who underwent Pap screening (73) had adequate cytology.

WHAT’S NEW?

It’s time for lubrication to become standard practice

This trial is the first to study speculum lubrication from this patient-oriented outcome and to show that women experience less pain when lubricating gel, rather than water, is applied to the speculum. This knowledge, combined with previous studies showing that a small amount of water-based lubricating gel does not interfere with liquid-based cytology or chlamydia test results, should make the use of lubricating gel standard practice when performing speculum examinations.

CAVEATS

We see no downside

The exclusion criteria of this study were meant to eliminate women who had an altered perception of pain that could skew study results. Yet those who met the exclusion criteria may also benefit from a pelvic exam with gel lubrication. We see no harm in trying a small amount of lubricant when examining them, as well.

In addition, the study did not compare various types and sizes of specula. However, we see no reason why the benefit of a gel lubricant would be limited to the type of speculum used by the examiner.

Studies in emergency departments that have used visual analog scales to measure interventions that decrease pain have used a mean difference of 0.9 as “clinically meaningful.” 7,8 The 0.74 difference observed in this study does not rise to that level. However, one in three patients in the gel group marked 0 on the pain scale, indicating that they had no pain, compared with only one in 10 in the water group.

 

 

We believe that the higher proportion of women experiencing no pain and the mean difference of 0.74 on the pain scale (both statistically significant), combined with the lack of risk associated with the use of a water-based lubricant, makes this a clinically useful practice changer.

CHALLENGES TO IMPLEMENTATION

There aren’t any

Other than clinical inertia, we see no challenges to the implementation of this recommendation.

References

1. Hill DA, Lamvu G. Effect of lubricating gel
on patient comfort during vaginal specu-
lum examination. Obstet Gynecol. 2012;119 (2 pt 1):227-231.

2. Harmanli O, Jones KA. Using lubricant for speculum insertion. Obstet Gynecol. 2010; 116:415-417.

3. Hoyo C, Yarnall KSH, Skinner CS, et al. Pain predicts non-adherence to pap smear screening among middle-aged African American women. Prev Med. 2005;41:439-445.

4. Amies AE, Miller L, Lee S, et al. The effect of vaginal speculum lubrication on the rate of unsatisfactory cervical cytology diagnosis. Obstet Gynecol. 2002;100:889-892.

5. Griffith WF, Stuart GS, Gluck KL, et al. Vaginal speculum lubrication and its effects on cervical cytology and microbiology. Contraception. 2005;72:60-64.

6. Evantash E. Lubricant use during Pap test collection. Bedford, Mass: Hologic, Inc; 2009. www.thinprep.com/hcp/specimen_collection/common_questions.html. Ac-
cessed October 9, 2013.

7. Bijur PE, Silver W, Gallagher EJ. Reliability of the visual analog scale for measurement of acute pain. Acad Emerg Med. 2001;8:1153-1157.

8. Kelly AM. Does the clinically significant difference in visual analog scale pain scores vary with gender, age, or cause of pain? Acad Emerg Med. 1998;5:1086-1090.

Acknowledgement

The PURLs Surveillance System is supported in part by Grant Number UL 1RR 024999 from the National Center for Research Resources, a Clinical Translational Science Award to the University of Chicago. The content is solely the responsibility of the authors and does not necessarily represent the official views of the National Center for Research Resources or the National Institutes of Health.

Copyright © 2013. The Family Physicians Inquiries Network. All rights reserved.

Reprinted with permission from the Family Physicians Inquiries Network and The Journal of Family Practice. 2013;61(9):E1-E3.

Issue
Clinician Reviews - 23(11)
Issue
Clinician Reviews - 23(11)
Page Number
28
Page Number
28
Publications
Publications
Topics
Article Type
Display Headline
Easing the Discomfort of a Speculum Exam
Display Headline
Easing the Discomfort of a Speculum Exam
Legacy Keywords
Speculum, exam, comfort, gel, lubrication, pelvic exam, pap smear, annual,
Legacy Keywords
Speculum, exam, comfort, gel, lubrication, pelvic exam, pap smear, annual,
Sections
Article Source

PURLs Copyright

Inside the Article

This Asthma Treatment Has a Lasting Side Effect in Children

Article Type
Changed
Tue, 12/13/2016 - 12:08
Display Headline
This Asthma Treatment Has a Lasting Side Effect in Children
A new study finds that when children with asthma use inhaled corticosteroids, the effect on growth may not be temporary, as once thought.

Practice changer

Before prescribing inhaled corticosteroids (ICS) for a child with asthma, tell the patient—and parents—that their use could lead to a small but permanent effect on adult height.1

STRENGTH OF ­RECOMMENDATIONS

B: Based on one prospective study.

Illustrative case

A 10-year-old boy is brought in by his father for asthma follow-up. The child uses an albuterol inhaler but has had increased coughing and wheezing recently. You are ready to step up his asthma therapy to include ICS. But the patient’s father questions this, noting that he recently read that steroids may reduce a child’s growth. How should you respond?

Inhaled corticosteroids are a mainstay in the treatment of asthma ranging from mild persistent to severe. Standards of care for asthma treatment involve a stepwise approach, with ICS added if symptoms are not controlled with short-acting beta antagonists alone.2 In addition, monotherapy with ICS is more effective for controlling symptoms than leukotriene inhibitors or other controller medications, while also decreasing hospitalizations and nocturnal awakenings and improving quality of life—with few side effects.3

What we know about ICS
and children’s growth

One adverse effect of ICS, however, is that of “decreased linear growth velocity”4—ie, slowing the rate at which children grow. Until recently, children were thought to “catch up” later in life, either by growing for a longer period of time than they would have had they not taken ICS or by growing at an increased velocity after ICS medications are discontinued.4-6

Study summary
The effect on growth is small, but long lasting

Kelly et al conducted a prospective observational cohort study that followed 943 participants in the Childhood Asthma Management Program (CAMP) in the years after the randomized controlled trial (RCT) ended.

A double-blind, placebo-controlled RCT, CAMP studied the linear growth of 1,041 children with mild to moderate persistent asthma who were divided into three treatment groups: One group received 200 g inhaled budesonide twice daily; a second group received 8 mg inhaled nedocromil twice daily; and a third group received placebo. Albuterol was used symptomatically by all three groups.7 The children ranged in age from 5 to 13 years at the start of the study; 98 patients—split evenly among the three treatment arms—were lost to follow-up.

During the four to six years of the CAMP trial, the budesonide group received a mean total of 636 mg ICS, whereas the nedocromil and placebo groups received an average of 88.5 and 109.4 mg ICS, respectively. After the RCT ended, all participants had standardized asthma treatment, receiving mean adjusted total doses of ICS of 381 mg for the budesonide group, 347.9 mg for the nedocromil group, and 355 mg for the placebo group.

Patients’ height was measured every six months for the next 4.5 years, and once or twice a year thereafter until they reached adult height (at a mean age of 24.9 ± 2.7 years).

ICS users were a half-inch shorter

Long-term ICS use was linked to a lower adult height. The adjusted mean height was 171.1 cm for the budesonide group versus 172.3 cm for those on placebo, a difference of 1.2 cm, or 0.47 inch; the mean adult height in the nedocromil group (172.1 cm) was similar to that of the placebo group (−0.2 cm).

The lower adult height in the ICS group did not vary significantly based on sex, age at trial entry, race, or duration of asthma prior to trial entry; however, dose was a key factor. A larger daily dose of budesonide—particularly in the first two years of the RCT—was associated with a lower adult height (about −0.1 cm for each g/kg in that two-year timeframe). This was consistent with results from studies that examined other types of ICS (beclomethasone, fluticasone, and mometasone).8-11

The study also showed that growth velocity was reduced in the first two years of assigned treatment with budesonide, and this was primarily among prepubertal participants. After the initial two-year slowing in growth rate, the children resumed growing at normal speeds.

What’s new?

Now we know:
Children don’t “catch up”

Retrospective studies have reported that children taking ICS for mild persistent to moderate asthma would have an initial slowing in growth velocity but then “catch up” by growing for a longer period of time.3-5 This is the first prospective study with good follow-up to show that ICS use affects long-term growth and adult height. While the effect is not large, some children and their families might be concerned about it.

Caveats
ICS use was atypical

The randomized controlled portion of the study used a prescribed dose of budesonide without regard to symptoms. This is not the typical pattern of ICS use. In addition, compliance with ICS varies significantly.12 Because the effect on adult height appears to be dose dependent, however, we think the results of this study are valid.

In addition, there was a placebo control group (and big differences in exposure to ICS) only for the duration of the RCT. During the subsequent study, all patients received equivalent doses of ICS. This means that the variation in mean adult height achieved can be primarily ascribed to participants’ use of ICS during the 4- to 6-year CAMP trial. Of note, the effect of ICS was greatest in prepubertal participants, so there may be a diminished effect as teens approach their final height.

 

 

The study did not look at the effect of ICS use in patients with severe asthma—the group most likely to use ICS. However, the benefits of ICS for those with severe asthma likely outweigh any negative effects on adult height.

Challenges to implementation
What to tell patients

The message we convey to patients (and parents) about ICS use is a nuanced one. We can stress that ICS remain very important in the treatment of asthma and, while it appears that their use causes a slight decrease in adult height, most children with persistent asthma benefit from ICS.

References

1. Kelly HW, Sternberg AL, Lescher R, et al; CAMP Research Group. Effect of inhaled glucocorticoids in childhood on adult height. 
N Engl J Med. 2012;367:904-912.

2. National Institutes of Health National Heart, Lung and Blood Institute. Expert Panel Report 3: Guidelines for the Diagnosis and Management of Asthma. National Asthma Education and Prevention Program, 2007. www.nhlbi.nih.gov/guidelines/asthma/asthgdln.pdf. Accessed August 15, 2013.

3. Chauhan BF, Ducharme FM. Anti-leukotriene agents compared to inhaled corticosteroids in the management of recurrent and/ or chronic asthma in adults and children. Cochrane Database Syst Rev. 2012;(5): CD002314.

4. Agertoft L, Pedersen S. Effect of long-term treatment with budesonide on adult height in children with asthma. N Engl J Med. 2000; 343:1064-1069.

5. Van Bever HP, Desager KN, Lijssens N, et al. Does treatment of asthmatic children with inhaled corticosteroids affect their adult height? Pediatr Pulmonol. 1999;27:369-375.

6. Silverstein MD, Yunginger JW, Reed CE, et al. Attained adult height after childhood asthma: effect of glucocorticoid therapy. J Allergy Clin Immunol. 1997;99:466-474.

7. The Childhood Asthma Management Program Research Group. Long-term effects of budesonide or nedocromil in children with asthma. N Engl J Med. 2000;343:1054-1063.

8. Tinkelman DG, Reed CE, Nelson HS, et al. Aerosol beclomethasone dipropionate compared with theophylline as primary treatment of chronic, mild to moderately severe asthma in children. Pediatrics. 1993;92:64-77.

9. Verberne AA, Frost C, Roorda RJ, et al. One year treatment with salmeterol compared with beclomethasone in children with asthma. Am J Respir Crit Care Med. 1997;156:688-695.

10. Allen DB, Bronsky EA, LaForce CF, et al. Growth in asthmatic children treated with fluticasone propionate. J Pediatr. 1998;132: 472-477.

11. Skoner DP, Meltzer EO, Milgrom H, et al. Effects of inhaled mometasone furoate on growth velocity and adrenal function: a placebo-controlled trial in children 4-9 years old with mild persistent asthma. J Asthma. 2011;48:848-859.

12. Cochrane MG, Bala MV, Downs KE, et al. Inhaled corticosteroids for asthma therapy: patient compliance, devices, and inhalation technique. Chest. 2000;117:542-550.

ACKNOWLEDGEMENT

The PURLs Surveillance System is supported in part by Grant Number UL 1RR 024999 from the National Center for Research Resources, a Clinical Translational Science Award to the University of Chicago. The content is solely the responsibility of the authors and does not necessarily represent the official views of the National Center for Research Resources or the National Institutes of Health.

Copyright © 2013. The Family Physicians Inquiries Network. All rights reserved.

Reprinted with permission from the Family Physicians Inquiries Network and The Journal of Family Practice. 2013;62(9):500-502.

Author and Disclosure Information

Tanner Nissly, DO, Shailendra Prasad, MBBS, MPH

Issue
Clinician Reviews - 23(10)
Publications
Topics
Page Number
19-20
Legacy Keywords
Asthma, treatment, side-effects, inhaled corticosteroids, ICS, height, pediatrics, children
Sections
Author and Disclosure Information

Tanner Nissly, DO, Shailendra Prasad, MBBS, MPH

Author and Disclosure Information

Tanner Nissly, DO, Shailendra Prasad, MBBS, MPH

A new study finds that when children with asthma use inhaled corticosteroids, the effect on growth may not be temporary, as once thought.
A new study finds that when children with asthma use inhaled corticosteroids, the effect on growth may not be temporary, as once thought.

Practice changer

Before prescribing inhaled corticosteroids (ICS) for a child with asthma, tell the patient—and parents—that their use could lead to a small but permanent effect on adult height.1

STRENGTH OF ­RECOMMENDATIONS

B: Based on one prospective study.

Illustrative case

A 10-year-old boy is brought in by his father for asthma follow-up. The child uses an albuterol inhaler but has had increased coughing and wheezing recently. You are ready to step up his asthma therapy to include ICS. But the patient’s father questions this, noting that he recently read that steroids may reduce a child’s growth. How should you respond?

Inhaled corticosteroids are a mainstay in the treatment of asthma ranging from mild persistent to severe. Standards of care for asthma treatment involve a stepwise approach, with ICS added if symptoms are not controlled with short-acting beta antagonists alone.2 In addition, monotherapy with ICS is more effective for controlling symptoms than leukotriene inhibitors or other controller medications, while also decreasing hospitalizations and nocturnal awakenings and improving quality of life—with few side effects.3

What we know about ICS
and children’s growth

One adverse effect of ICS, however, is that of “decreased linear growth velocity”4—ie, slowing the rate at which children grow. Until recently, children were thought to “catch up” later in life, either by growing for a longer period of time than they would have had they not taken ICS or by growing at an increased velocity after ICS medications are discontinued.4-6

Study summary
The effect on growth is small, but long lasting

Kelly et al conducted a prospective observational cohort study that followed 943 participants in the Childhood Asthma Management Program (CAMP) in the years after the randomized controlled trial (RCT) ended.

A double-blind, placebo-controlled RCT, CAMP studied the linear growth of 1,041 children with mild to moderate persistent asthma who were divided into three treatment groups: One group received 200 g inhaled budesonide twice daily; a second group received 8 mg inhaled nedocromil twice daily; and a third group received placebo. Albuterol was used symptomatically by all three groups.7 The children ranged in age from 5 to 13 years at the start of the study; 98 patients—split evenly among the three treatment arms—were lost to follow-up.

During the four to six years of the CAMP trial, the budesonide group received a mean total of 636 mg ICS, whereas the nedocromil and placebo groups received an average of 88.5 and 109.4 mg ICS, respectively. After the RCT ended, all participants had standardized asthma treatment, receiving mean adjusted total doses of ICS of 381 mg for the budesonide group, 347.9 mg for the nedocromil group, and 355 mg for the placebo group.

Patients’ height was measured every six months for the next 4.5 years, and once or twice a year thereafter until they reached adult height (at a mean age of 24.9 ± 2.7 years).

ICS users were a half-inch shorter

Long-term ICS use was linked to a lower adult height. The adjusted mean height was 171.1 cm for the budesonide group versus 172.3 cm for those on placebo, a difference of 1.2 cm, or 0.47 inch; the mean adult height in the nedocromil group (172.1 cm) was similar to that of the placebo group (−0.2 cm).

The lower adult height in the ICS group did not vary significantly based on sex, age at trial entry, race, or duration of asthma prior to trial entry; however, dose was a key factor. A larger daily dose of budesonide—particularly in the first two years of the RCT—was associated with a lower adult height (about −0.1 cm for each g/kg in that two-year timeframe). This was consistent with results from studies that examined other types of ICS (beclomethasone, fluticasone, and mometasone).8-11

The study also showed that growth velocity was reduced in the first two years of assigned treatment with budesonide, and this was primarily among prepubertal participants. After the initial two-year slowing in growth rate, the children resumed growing at normal speeds.

What’s new?

Now we know:
Children don’t “catch up”

Retrospective studies have reported that children taking ICS for mild persistent to moderate asthma would have an initial slowing in growth velocity but then “catch up” by growing for a longer period of time.3-5 This is the first prospective study with good follow-up to show that ICS use affects long-term growth and adult height. While the effect is not large, some children and their families might be concerned about it.

Caveats
ICS use was atypical

The randomized controlled portion of the study used a prescribed dose of budesonide without regard to symptoms. This is not the typical pattern of ICS use. In addition, compliance with ICS varies significantly.12 Because the effect on adult height appears to be dose dependent, however, we think the results of this study are valid.

In addition, there was a placebo control group (and big differences in exposure to ICS) only for the duration of the RCT. During the subsequent study, all patients received equivalent doses of ICS. This means that the variation in mean adult height achieved can be primarily ascribed to participants’ use of ICS during the 4- to 6-year CAMP trial. Of note, the effect of ICS was greatest in prepubertal participants, so there may be a diminished effect as teens approach their final height.

 

 

The study did not look at the effect of ICS use in patients with severe asthma—the group most likely to use ICS. However, the benefits of ICS for those with severe asthma likely outweigh any negative effects on adult height.

Challenges to implementation
What to tell patients

The message we convey to patients (and parents) about ICS use is a nuanced one. We can stress that ICS remain very important in the treatment of asthma and, while it appears that their use causes a slight decrease in adult height, most children with persistent asthma benefit from ICS.

References

1. Kelly HW, Sternberg AL, Lescher R, et al; CAMP Research Group. Effect of inhaled glucocorticoids in childhood on adult height. 
N Engl J Med. 2012;367:904-912.

2. National Institutes of Health National Heart, Lung and Blood Institute. Expert Panel Report 3: Guidelines for the Diagnosis and Management of Asthma. National Asthma Education and Prevention Program, 2007. www.nhlbi.nih.gov/guidelines/asthma/asthgdln.pdf. Accessed August 15, 2013.

3. Chauhan BF, Ducharme FM. Anti-leukotriene agents compared to inhaled corticosteroids in the management of recurrent and/ or chronic asthma in adults and children. Cochrane Database Syst Rev. 2012;(5): CD002314.

4. Agertoft L, Pedersen S. Effect of long-term treatment with budesonide on adult height in children with asthma. N Engl J Med. 2000; 343:1064-1069.

5. Van Bever HP, Desager KN, Lijssens N, et al. Does treatment of asthmatic children with inhaled corticosteroids affect their adult height? Pediatr Pulmonol. 1999;27:369-375.

6. Silverstein MD, Yunginger JW, Reed CE, et al. Attained adult height after childhood asthma: effect of glucocorticoid therapy. J Allergy Clin Immunol. 1997;99:466-474.

7. The Childhood Asthma Management Program Research Group. Long-term effects of budesonide or nedocromil in children with asthma. N Engl J Med. 2000;343:1054-1063.

8. Tinkelman DG, Reed CE, Nelson HS, et al. Aerosol beclomethasone dipropionate compared with theophylline as primary treatment of chronic, mild to moderately severe asthma in children. Pediatrics. 1993;92:64-77.

9. Verberne AA, Frost C, Roorda RJ, et al. One year treatment with salmeterol compared with beclomethasone in children with asthma. Am J Respir Crit Care Med. 1997;156:688-695.

10. Allen DB, Bronsky EA, LaForce CF, et al. Growth in asthmatic children treated with fluticasone propionate. J Pediatr. 1998;132: 472-477.

11. Skoner DP, Meltzer EO, Milgrom H, et al. Effects of inhaled mometasone furoate on growth velocity and adrenal function: a placebo-controlled trial in children 4-9 years old with mild persistent asthma. J Asthma. 2011;48:848-859.

12. Cochrane MG, Bala MV, Downs KE, et al. Inhaled corticosteroids for asthma therapy: patient compliance, devices, and inhalation technique. Chest. 2000;117:542-550.

ACKNOWLEDGEMENT

The PURLs Surveillance System is supported in part by Grant Number UL 1RR 024999 from the National Center for Research Resources, a Clinical Translational Science Award to the University of Chicago. The content is solely the responsibility of the authors and does not necessarily represent the official views of the National Center for Research Resources or the National Institutes of Health.

Copyright © 2013. The Family Physicians Inquiries Network. All rights reserved.

Reprinted with permission from the Family Physicians Inquiries Network and The Journal of Family Practice. 2013;62(9):500-502.

Practice changer

Before prescribing inhaled corticosteroids (ICS) for a child with asthma, tell the patient—and parents—that their use could lead to a small but permanent effect on adult height.1

STRENGTH OF ­RECOMMENDATIONS

B: Based on one prospective study.

Illustrative case

A 10-year-old boy is brought in by his father for asthma follow-up. The child uses an albuterol inhaler but has had increased coughing and wheezing recently. You are ready to step up his asthma therapy to include ICS. But the patient’s father questions this, noting that he recently read that steroids may reduce a child’s growth. How should you respond?

Inhaled corticosteroids are a mainstay in the treatment of asthma ranging from mild persistent to severe. Standards of care for asthma treatment involve a stepwise approach, with ICS added if symptoms are not controlled with short-acting beta antagonists alone.2 In addition, monotherapy with ICS is more effective for controlling symptoms than leukotriene inhibitors or other controller medications, while also decreasing hospitalizations and nocturnal awakenings and improving quality of life—with few side effects.3

What we know about ICS
and children’s growth

One adverse effect of ICS, however, is that of “decreased linear growth velocity”4—ie, slowing the rate at which children grow. Until recently, children were thought to “catch up” later in life, either by growing for a longer period of time than they would have had they not taken ICS or by growing at an increased velocity after ICS medications are discontinued.4-6

Study summary
The effect on growth is small, but long lasting

Kelly et al conducted a prospective observational cohort study that followed 943 participants in the Childhood Asthma Management Program (CAMP) in the years after the randomized controlled trial (RCT) ended.

A double-blind, placebo-controlled RCT, CAMP studied the linear growth of 1,041 children with mild to moderate persistent asthma who were divided into three treatment groups: One group received 200 g inhaled budesonide twice daily; a second group received 8 mg inhaled nedocromil twice daily; and a third group received placebo. Albuterol was used symptomatically by all three groups.7 The children ranged in age from 5 to 13 years at the start of the study; 98 patients—split evenly among the three treatment arms—were lost to follow-up.

During the four to six years of the CAMP trial, the budesonide group received a mean total of 636 mg ICS, whereas the nedocromil and placebo groups received an average of 88.5 and 109.4 mg ICS, respectively. After the RCT ended, all participants had standardized asthma treatment, receiving mean adjusted total doses of ICS of 381 mg for the budesonide group, 347.9 mg for the nedocromil group, and 355 mg for the placebo group.

Patients’ height was measured every six months for the next 4.5 years, and once or twice a year thereafter until they reached adult height (at a mean age of 24.9 ± 2.7 years).

ICS users were a half-inch shorter

Long-term ICS use was linked to a lower adult height. The adjusted mean height was 171.1 cm for the budesonide group versus 172.3 cm for those on placebo, a difference of 1.2 cm, or 0.47 inch; the mean adult height in the nedocromil group (172.1 cm) was similar to that of the placebo group (−0.2 cm).

The lower adult height in the ICS group did not vary significantly based on sex, age at trial entry, race, or duration of asthma prior to trial entry; however, dose was a key factor. A larger daily dose of budesonide—particularly in the first two years of the RCT—was associated with a lower adult height (about −0.1 cm for each g/kg in that two-year timeframe). This was consistent with results from studies that examined other types of ICS (beclomethasone, fluticasone, and mometasone).8-11

The study also showed that growth velocity was reduced in the first two years of assigned treatment with budesonide, and this was primarily among prepubertal participants. After the initial two-year slowing in growth rate, the children resumed growing at normal speeds.

What’s new?

Now we know:
Children don’t “catch up”

Retrospective studies have reported that children taking ICS for mild persistent to moderate asthma would have an initial slowing in growth velocity but then “catch up” by growing for a longer period of time.3-5 This is the first prospective study with good follow-up to show that ICS use affects long-term growth and adult height. While the effect is not large, some children and their families might be concerned about it.

Caveats
ICS use was atypical

The randomized controlled portion of the study used a prescribed dose of budesonide without regard to symptoms. This is not the typical pattern of ICS use. In addition, compliance with ICS varies significantly.12 Because the effect on adult height appears to be dose dependent, however, we think the results of this study are valid.

In addition, there was a placebo control group (and big differences in exposure to ICS) only for the duration of the RCT. During the subsequent study, all patients received equivalent doses of ICS. This means that the variation in mean adult height achieved can be primarily ascribed to participants’ use of ICS during the 4- to 6-year CAMP trial. Of note, the effect of ICS was greatest in prepubertal participants, so there may be a diminished effect as teens approach their final height.

 

 

The study did not look at the effect of ICS use in patients with severe asthma—the group most likely to use ICS. However, the benefits of ICS for those with severe asthma likely outweigh any negative effects on adult height.

Challenges to implementation
What to tell patients

The message we convey to patients (and parents) about ICS use is a nuanced one. We can stress that ICS remain very important in the treatment of asthma and, while it appears that their use causes a slight decrease in adult height, most children with persistent asthma benefit from ICS.

References

1. Kelly HW, Sternberg AL, Lescher R, et al; CAMP Research Group. Effect of inhaled glucocorticoids in childhood on adult height. 
N Engl J Med. 2012;367:904-912.

2. National Institutes of Health National Heart, Lung and Blood Institute. Expert Panel Report 3: Guidelines for the Diagnosis and Management of Asthma. National Asthma Education and Prevention Program, 2007. www.nhlbi.nih.gov/guidelines/asthma/asthgdln.pdf. Accessed August 15, 2013.

3. Chauhan BF, Ducharme FM. Anti-leukotriene agents compared to inhaled corticosteroids in the management of recurrent and/ or chronic asthma in adults and children. Cochrane Database Syst Rev. 2012;(5): CD002314.

4. Agertoft L, Pedersen S. Effect of long-term treatment with budesonide on adult height in children with asthma. N Engl J Med. 2000; 343:1064-1069.

5. Van Bever HP, Desager KN, Lijssens N, et al. Does treatment of asthmatic children with inhaled corticosteroids affect their adult height? Pediatr Pulmonol. 1999;27:369-375.

6. Silverstein MD, Yunginger JW, Reed CE, et al. Attained adult height after childhood asthma: effect of glucocorticoid therapy. J Allergy Clin Immunol. 1997;99:466-474.

7. The Childhood Asthma Management Program Research Group. Long-term effects of budesonide or nedocromil in children with asthma. N Engl J Med. 2000;343:1054-1063.

8. Tinkelman DG, Reed CE, Nelson HS, et al. Aerosol beclomethasone dipropionate compared with theophylline as primary treatment of chronic, mild to moderately severe asthma in children. Pediatrics. 1993;92:64-77.

9. Verberne AA, Frost C, Roorda RJ, et al. One year treatment with salmeterol compared with beclomethasone in children with asthma. Am J Respir Crit Care Med. 1997;156:688-695.

10. Allen DB, Bronsky EA, LaForce CF, et al. Growth in asthmatic children treated with fluticasone propionate. J Pediatr. 1998;132: 472-477.

11. Skoner DP, Meltzer EO, Milgrom H, et al. Effects of inhaled mometasone furoate on growth velocity and adrenal function: a placebo-controlled trial in children 4-9 years old with mild persistent asthma. J Asthma. 2011;48:848-859.

12. Cochrane MG, Bala MV, Downs KE, et al. Inhaled corticosteroids for asthma therapy: patient compliance, devices, and inhalation technique. Chest. 2000;117:542-550.

ACKNOWLEDGEMENT

The PURLs Surveillance System is supported in part by Grant Number UL 1RR 024999 from the National Center for Research Resources, a Clinical Translational Science Award to the University of Chicago. The content is solely the responsibility of the authors and does not necessarily represent the official views of the National Center for Research Resources or the National Institutes of Health.

Copyright © 2013. The Family Physicians Inquiries Network. All rights reserved.

Reprinted with permission from the Family Physicians Inquiries Network and The Journal of Family Practice. 2013;62(9):500-502.

Issue
Clinician Reviews - 23(10)
Issue
Clinician Reviews - 23(10)
Page Number
19-20
Page Number
19-20
Publications
Publications
Topics
Article Type
Display Headline
This Asthma Treatment Has a Lasting Side Effect in Children
Display Headline
This Asthma Treatment Has a Lasting Side Effect in Children
Legacy Keywords
Asthma, treatment, side-effects, inhaled corticosteroids, ICS, height, pediatrics, children
Legacy Keywords
Asthma, treatment, side-effects, inhaled corticosteroids, ICS, height, pediatrics, children
Sections
Article Source

PURLs Copyright

Inside the Article

This asthma treatment has a lasting side effect in children

Article Type
Changed
Tue, 07/14/2020 - 15:12
Display Headline
This asthma treatment has a lasting side effect in children
Practice changer

Before prescribing inhaled corticosteroids (ICS) for a child with asthma, tell the patient—and parents—that their use could lead to a small but permanent effect on adult height.1

STRENGTH OF RECOMMENDATIONS

B: Based on one prospective study.

Kelly HW, Sternberg AL, Lescher R, et al; CAMP Research Group. Effect of inhaled glucocorticoids in childhood on adult height. N Engl J Med. 2012;367:904-912.

 

Illustrative case

A 10-year-old boy is brought in by his father for asthma follow-up. The child uses an albuterol inhaler, but has had increased coughing and wheezing recently. You are ready to step up his asthma therapy to include ICS. But the patient’s father questions this, noting that he recently read that steroids may reduce a child’s growth. How should you respond?

Inhaled corticosteroids (ICS) are a mainstay in the treatment of asthma ranging from mild persistent to severe. Standards of care for asthma treatment involve a stepwise approach, with ICS added if symptoms are not controlled with short-acting beta antagonists alone.2 In addition, monotherapy with ICS is more effective for controlling symptoms than leukotriene inhibitors or other controller medications, while also decreasing hospitalizations and nocturnal awakenings and improving quality of life—with few side effects.3

What we know about ICS and children’s growth
One adverse effect of ICS, however, is that of “decreased linear growth velocity”4—ie, slowing the rate at which children grow. Until recently, children were thought to “catch up” later in life, either by growing for a longer period of time than they would had they not taken ICS or by growing at an increased velocity after ICS medications are discontinued.4-6

Study summary: The effect on growth is small, but long-lasting

Kelly et al conducted a prospective observational cohort study that followed 943 (90.7%) participants in the Childhood Asthma Management Program (CAMP) in the years after the randomized controlled trial (RCT) ended.

A double-blind, placebo-controlled RCT, CAMP studied the linear growth of 1041 children with mild-to-moderate persistent asthma who were divided into 3 treatment groups: One group received 200 mcg inhaled budesonide twice daily; a second group received 8 mg inhaled nedocromil twice daily; and a third group received placebo. Albuterol was used symptomatically by all 3 groups.7 The children ranged in age from 5 to 13 years at the start of the study; 98 patients—split evenly among the 3 treatment arms—were lost to follow-up.

During the 4 to 6 years of the CAMP trial, the budesonide group received a mean total of 636 mg ICS, whereas the nedocromil and placebo groups received an average of 88.5 and 109.4 mg ICS, respectively. After the RCT ended, all participants had standardized asthma treatment, receiving mean adjusted total doses of ICS of 381 mg for the budesonide group, 347.9 mg for the nedocromil group, and 355 mg for the placebo group.

Patients’ height was measured every 6 months for the next 4.5 years, and once or twice a year thereafter until they reached adult height (at a mean age of 24.9±2.7 years).

ICS users were a half inch shorter
Long-term ICS use was linked to a lower adult height. The adjusted mean height was 171.1 cm for the budesonide group vs 172.3 cm for those on placebo, a difference of 1.2 cm, or 0.47 inch (95% confidence interval [CI], −1.9 to −0.5; P=.001); the mean adult height in the nedocromil group (172.1 cm) was similar to that of the placebo group (−0.2 cm; 95% CI, −0.9 to 0.5; P=.61).

The lower adult height in the ICS group did not vary significantly based on sex, age at trial entry, race, or duration of asthma prior to trial entry; however, dose was a key factor. A larger daily dose of budesonide—particularly in the first 2 years of the RCT—was associated with a lower adult height (about −0.1 cm for each mcg/kg in that 2-year time frame). This was consistent with results from studies that looked at other types of ICS (beclomethasone, fluticasone, and mometasone).8-11

The study also showed that growth velocity was reduced in the first 2 years of assigned treatment with budesonide, and this was primarily among prepubertal participants. After the initial 2-year slowing in growth rate, the children resumed growing at normal speeds.

 

 

 

What’s new: Now we know: Children don’t “catch up"

Retrospective studies have reported that children on ICS for mild persistent to moderate asthma would have an initial slowing in growth velocity but then “catch up” by growing for a longer period of time.3-5 This is the first prospective study with good follow-up to show that ICS use affects long-term growth and adult height. While the effect is not large, some children and their families might be concerned about it.

Caveats: ICS use was atypical

The randomized controlled portion of the study used a prescribed dose of budesonide without regard to symptoms. This is not the typical pattern of ICS use. In addition, compliance with ICS varies significantly.12 Because the effect on adult height appears to be dose dependent, however, we think the results of this study are valid.

In addition, there was a placebo control group (and big differences in exposure to ICS) only for the duration of the RCT. During the subsequent study, all patients received equivalent doses of ICS. This means that the variation in mean adult height achieved can be primarily ascribed to participants’ use of ICS during the 4- to 6-year CAMP trial. Of note, the effect of ICS was greatest in prepubertal participants, so there may be a diminished effect as teens approach their final height.

The study did not look at the effect of ICS use in patients with severe asthma—the group most likely to use ICS. However, the benefits of ICS for those with severe asthma likely outweigh any negative effects on adult height.

Challenges to implementation: What to tell patients

The message we convey to patients (and parents) about ICS use is a nuanced one. We can stress that ICS remain very important in the treatment of asthma and, while it appears that their use causes a slight decrease in adult height, most children with persistent asthma benefit from ICS.

Acknowledgement

The PURLs Surveillance System is supported in part by Grant Number UL1RR024999 from the National Center for Research Resources, a Clinical Translational Science Award to the University of Chicago. The content is solely the responsibility of the authors and does not necessarily represent the official views of the National Center for Research Resources or the National Institutes of Health.

Files
References

1. Kelly HW, Sternberg AL, Lescher R, et al; CAMP Research Group. Effect of inhaled glucocorticoids in childhood on adult height. N Engl J Med. 2012;367:904-912.

2. Expert Panel Report 3: Guidelines for the Diagnosis and Management of Asthma. National Institutes of Health National Heart, Lung and Blood Institute: National Asthma Education and Prevention Program, 2007. Available at: http://www.nhlbi. nih.gov/guidelines/asthma/asthgdln.pdf. Accessed August 15, 2013.

3. Chauhan BF, Ducharme FM. Anti-leukotriene agents compared to inhaled corticosteroids in the management of recurrent and/ or chronic asthma in adults and children. Cochrane Database Syst Rev. 2012;(5):CD002314.

4. Agertoft L, Pedersen S. Effect of long- term treatment with budesonide on adult height in children with asthma. N Engl J Med. 2000;343:1064-1069.

5. Van Bever HP, Desager KN, Lijssens N, et al. Does treatment of asthmatic children with inhaled corticosteroids affect their adult height? Pediatr Pulmonol. 1999;27:369-375.

6. Silverstein MD, Yunginger JW, Reed CE, et al. Attained adult height after childhood asthma: effect of glucocorticoid therapy. J Allergy Clin Immunol. 1997;99:466-474.

7. The Childhood Asthma Management Program Research Group. Long-term effects of budesonide or nedocromil in children with asthma. N Engl J Med. 2000;343:1054-1063.

8. Tinkelman DG, Reed CE, Nelson HS, et al. Aerosol beclomethasone dipropionate compared with theophylline as primary treatment of chronic, mild to moderately severe asthma in children. Pediatrics. 1993;92:64-77.

9. Verberne AA, Frost C, Roorda RJ, et al. One year treatment with salmeterol compared with beclomethasone in children with asthma. Am J Respir Crit Care Med. 1997;156:688-695.

10. Allen DB, Bronsky EA, LaForce CF, et al. Growth in asthmatic children treated with fluticasone propionate. J Pediatr 1998;132: 472-477.

11. Skoner DP, Meltzer EO, Milgrom H, et al. Effects of inhaled mometasone furoate on growth velocity and adrenal function: a placebo-controlled trial in children 4-9 years old with mild persistent asthma. J Asthma. 2011;48:848-859.

12. Cochrane MG, Bala MV, Downs KE, et al. Inhaled corticosteroids for asthma therapy: patient compliance, devices, and inhalation technique. Chest. 2000;117:542-550.

Article PDF
Author and Disclosure Information

Tanner Nissly, DO;
Shailendra Prasad, MBBS, MPH

Department of Family Medicine and Community Health, University of Minnesota, Minneapolis

PURLs Editor
James Stevermer, MD, MSPH
Department of Family Medicine, University of Missouri at Columbia

Issue
The Journal of Family Practice - 62(9)
Publications
Topics
Page Number
500-502
Legacy Keywords
Tanner Nissly; DO; Shailendra Prasad; MBBS; MPH; asthma; PURLs; inhaled corticosteroids; ICS; Childhood Asthma Management Program; CAMP;
Sections
Files
Files
Author and Disclosure Information

Tanner Nissly, DO;
Shailendra Prasad, MBBS, MPH

Department of Family Medicine and Community Health, University of Minnesota, Minneapolis

PURLs Editor
James Stevermer, MD, MSPH
Department of Family Medicine, University of Missouri at Columbia

Author and Disclosure Information

Tanner Nissly, DO;
Shailendra Prasad, MBBS, MPH

Department of Family Medicine and Community Health, University of Minnesota, Minneapolis

PURLs Editor
James Stevermer, MD, MSPH
Department of Family Medicine, University of Missouri at Columbia

Article PDF
Article PDF
Practice changer

Before prescribing inhaled corticosteroids (ICS) for a child with asthma, tell the patient—and parents—that their use could lead to a small but permanent effect on adult height.1

STRENGTH OF RECOMMENDATIONS

B: Based on one prospective study.

Kelly HW, Sternberg AL, Lescher R, et al; CAMP Research Group. Effect of inhaled glucocorticoids in childhood on adult height. N Engl J Med. 2012;367:904-912.

 

Illustrative case

A 10-year-old boy is brought in by his father for asthma follow-up. The child uses an albuterol inhaler, but has had increased coughing and wheezing recently. You are ready to step up his asthma therapy to include ICS. But the patient’s father questions this, noting that he recently read that steroids may reduce a child’s growth. How should you respond?

Inhaled corticosteroids (ICS) are a mainstay in the treatment of asthma ranging from mild persistent to severe. Standards of care for asthma treatment involve a stepwise approach, with ICS added if symptoms are not controlled with short-acting beta antagonists alone.2 In addition, monotherapy with ICS is more effective for controlling symptoms than leukotriene inhibitors or other controller medications, while also decreasing hospitalizations and nocturnal awakenings and improving quality of life—with few side effects.3

What we know about ICS and children’s growth
One adverse effect of ICS, however, is that of “decreased linear growth velocity”4—ie, slowing the rate at which children grow. Until recently, children were thought to “catch up” later in life, either by growing for a longer period of time than they would had they not taken ICS or by growing at an increased velocity after ICS medications are discontinued.4-6

Study summary: The effect on growth is small, but long-lasting

Kelly et al conducted a prospective observational cohort study that followed 943 (90.7%) participants in the Childhood Asthma Management Program (CAMP) in the years after the randomized controlled trial (RCT) ended.

A double-blind, placebo-controlled RCT, CAMP studied the linear growth of 1041 children with mild-to-moderate persistent asthma who were divided into 3 treatment groups: One group received 200 mcg inhaled budesonide twice daily; a second group received 8 mg inhaled nedocromil twice daily; and a third group received placebo. Albuterol was used symptomatically by all 3 groups.7 The children ranged in age from 5 to 13 years at the start of the study; 98 patients—split evenly among the 3 treatment arms—were lost to follow-up.

During the 4 to 6 years of the CAMP trial, the budesonide group received a mean total of 636 mg ICS, whereas the nedocromil and placebo groups received an average of 88.5 and 109.4 mg ICS, respectively. After the RCT ended, all participants had standardized asthma treatment, receiving mean adjusted total doses of ICS of 381 mg for the budesonide group, 347.9 mg for the nedocromil group, and 355 mg for the placebo group.

Patients’ height was measured every 6 months for the next 4.5 years, and once or twice a year thereafter until they reached adult height (at a mean age of 24.9±2.7 years).

ICS users were a half inch shorter
Long-term ICS use was linked to a lower adult height. The adjusted mean height was 171.1 cm for the budesonide group vs 172.3 cm for those on placebo, a difference of 1.2 cm, or 0.47 inch (95% confidence interval [CI], −1.9 to −0.5; P=.001); the mean adult height in the nedocromil group (172.1 cm) was similar to that of the placebo group (−0.2 cm; 95% CI, −0.9 to 0.5; P=.61).

The lower adult height in the ICS group did not vary significantly based on sex, age at trial entry, race, or duration of asthma prior to trial entry; however, dose was a key factor. A larger daily dose of budesonide—particularly in the first 2 years of the RCT—was associated with a lower adult height (about −0.1 cm for each mcg/kg in that 2-year time frame). This was consistent with results from studies that looked at other types of ICS (beclomethasone, fluticasone, and mometasone).8-11

The study also showed that growth velocity was reduced in the first 2 years of assigned treatment with budesonide, and this was primarily among prepubertal participants. After the initial 2-year slowing in growth rate, the children resumed growing at normal speeds.

 

 

 

What’s new: Now we know: Children don’t “catch up"

Retrospective studies have reported that children on ICS for mild persistent to moderate asthma would have an initial slowing in growth velocity but then “catch up” by growing for a longer period of time.3-5 This is the first prospective study with good follow-up to show that ICS use affects long-term growth and adult height. While the effect is not large, some children and their families might be concerned about it.

Caveats: ICS use was atypical

The randomized controlled portion of the study used a prescribed dose of budesonide without regard to symptoms. This is not the typical pattern of ICS use. In addition, compliance with ICS varies significantly.12 Because the effect on adult height appears to be dose dependent, however, we think the results of this study are valid.

In addition, there was a placebo control group (and big differences in exposure to ICS) only for the duration of the RCT. During the subsequent study, all patients received equivalent doses of ICS. This means that the variation in mean adult height achieved can be primarily ascribed to participants’ use of ICS during the 4- to 6-year CAMP trial. Of note, the effect of ICS was greatest in prepubertal participants, so there may be a diminished effect as teens approach their final height.

The study did not look at the effect of ICS use in patients with severe asthma—the group most likely to use ICS. However, the benefits of ICS for those with severe asthma likely outweigh any negative effects on adult height.

Challenges to implementation: What to tell patients

The message we convey to patients (and parents) about ICS use is a nuanced one. We can stress that ICS remain very important in the treatment of asthma and, while it appears that their use causes a slight decrease in adult height, most children with persistent asthma benefit from ICS.

Acknowledgement

The PURLs Surveillance System is supported in part by Grant Number UL1RR024999 from the National Center for Research Resources, a Clinical Translational Science Award to the University of Chicago. The content is solely the responsibility of the authors and does not necessarily represent the official views of the National Center for Research Resources or the National Institutes of Health.

Practice changer

Before prescribing inhaled corticosteroids (ICS) for a child with asthma, tell the patient—and parents—that their use could lead to a small but permanent effect on adult height.1

STRENGTH OF RECOMMENDATIONS

B: Based on one prospective study.

Kelly HW, Sternberg AL, Lescher R, et al; CAMP Research Group. Effect of inhaled glucocorticoids in childhood on adult height. N Engl J Med. 2012;367:904-912.

 

Illustrative case

A 10-year-old boy is brought in by his father for asthma follow-up. The child uses an albuterol inhaler, but has had increased coughing and wheezing recently. You are ready to step up his asthma therapy to include ICS. But the patient’s father questions this, noting that he recently read that steroids may reduce a child’s growth. How should you respond?

Inhaled corticosteroids (ICS) are a mainstay in the treatment of asthma ranging from mild persistent to severe. Standards of care for asthma treatment involve a stepwise approach, with ICS added if symptoms are not controlled with short-acting beta antagonists alone.2 In addition, monotherapy with ICS is more effective for controlling symptoms than leukotriene inhibitors or other controller medications, while also decreasing hospitalizations and nocturnal awakenings and improving quality of life—with few side effects.3

What we know about ICS and children’s growth
One adverse effect of ICS, however, is that of “decreased linear growth velocity”4—ie, slowing the rate at which children grow. Until recently, children were thought to “catch up” later in life, either by growing for a longer period of time than they would had they not taken ICS or by growing at an increased velocity after ICS medications are discontinued.4-6

Study summary: The effect on growth is small, but long-lasting

Kelly et al conducted a prospective observational cohort study that followed 943 (90.7%) participants in the Childhood Asthma Management Program (CAMP) in the years after the randomized controlled trial (RCT) ended.

A double-blind, placebo-controlled RCT, CAMP studied the linear growth of 1041 children with mild-to-moderate persistent asthma who were divided into 3 treatment groups: One group received 200 mcg inhaled budesonide twice daily; a second group received 8 mg inhaled nedocromil twice daily; and a third group received placebo. Albuterol was used symptomatically by all 3 groups.7 The children ranged in age from 5 to 13 years at the start of the study; 98 patients—split evenly among the 3 treatment arms—were lost to follow-up.

During the 4 to 6 years of the CAMP trial, the budesonide group received a mean total of 636 mg ICS, whereas the nedocromil and placebo groups received an average of 88.5 and 109.4 mg ICS, respectively. After the RCT ended, all participants had standardized asthma treatment, receiving mean adjusted total doses of ICS of 381 mg for the budesonide group, 347.9 mg for the nedocromil group, and 355 mg for the placebo group.

Patients’ height was measured every 6 months for the next 4.5 years, and once or twice a year thereafter until they reached adult height (at a mean age of 24.9±2.7 years).

ICS users were a half inch shorter
Long-term ICS use was linked to a lower adult height. The adjusted mean height was 171.1 cm for the budesonide group vs 172.3 cm for those on placebo, a difference of 1.2 cm, or 0.47 inch (95% confidence interval [CI], −1.9 to −0.5; P=.001); the mean adult height in the nedocromil group (172.1 cm) was similar to that of the placebo group (−0.2 cm; 95% CI, −0.9 to 0.5; P=.61).

The lower adult height in the ICS group did not vary significantly based on sex, age at trial entry, race, or duration of asthma prior to trial entry; however, dose was a key factor. A larger daily dose of budesonide—particularly in the first 2 years of the RCT—was associated with a lower adult height (about −0.1 cm for each mcg/kg in that 2-year time frame). This was consistent with results from studies that looked at other types of ICS (beclomethasone, fluticasone, and mometasone).8-11

The study also showed that growth velocity was reduced in the first 2 years of assigned treatment with budesonide, and this was primarily among prepubertal participants. After the initial 2-year slowing in growth rate, the children resumed growing at normal speeds.

 

 

 

What’s new: Now we know: Children don’t “catch up"

Retrospective studies have reported that children on ICS for mild persistent to moderate asthma would have an initial slowing in growth velocity but then “catch up” by growing for a longer period of time.3-5 This is the first prospective study with good follow-up to show that ICS use affects long-term growth and adult height. While the effect is not large, some children and their families might be concerned about it.

Caveats: ICS use was atypical

The randomized controlled portion of the study used a prescribed dose of budesonide without regard to symptoms. This is not the typical pattern of ICS use. In addition, compliance with ICS varies significantly.12 Because the effect on adult height appears to be dose dependent, however, we think the results of this study are valid.

In addition, there was a placebo control group (and big differences in exposure to ICS) only for the duration of the RCT. During the subsequent study, all patients received equivalent doses of ICS. This means that the variation in mean adult height achieved can be primarily ascribed to participants’ use of ICS during the 4- to 6-year CAMP trial. Of note, the effect of ICS was greatest in prepubertal participants, so there may be a diminished effect as teens approach their final height.

The study did not look at the effect of ICS use in patients with severe asthma—the group most likely to use ICS. However, the benefits of ICS for those with severe asthma likely outweigh any negative effects on adult height.

Challenges to implementation: What to tell patients

The message we convey to patients (and parents) about ICS use is a nuanced one. We can stress that ICS remain very important in the treatment of asthma and, while it appears that their use causes a slight decrease in adult height, most children with persistent asthma benefit from ICS.

Acknowledgement

The PURLs Surveillance System is supported in part by Grant Number UL1RR024999 from the National Center for Research Resources, a Clinical Translational Science Award to the University of Chicago. The content is solely the responsibility of the authors and does not necessarily represent the official views of the National Center for Research Resources or the National Institutes of Health.

References

1. Kelly HW, Sternberg AL, Lescher R, et al; CAMP Research Group. Effect of inhaled glucocorticoids in childhood on adult height. N Engl J Med. 2012;367:904-912.

2. Expert Panel Report 3: Guidelines for the Diagnosis and Management of Asthma. National Institutes of Health National Heart, Lung and Blood Institute: National Asthma Education and Prevention Program, 2007. Available at: http://www.nhlbi. nih.gov/guidelines/asthma/asthgdln.pdf. Accessed August 15, 2013.

3. Chauhan BF, Ducharme FM. Anti-leukotriene agents compared to inhaled corticosteroids in the management of recurrent and/ or chronic asthma in adults and children. Cochrane Database Syst Rev. 2012;(5):CD002314.

4. Agertoft L, Pedersen S. Effect of long- term treatment with budesonide on adult height in children with asthma. N Engl J Med. 2000;343:1064-1069.

5. Van Bever HP, Desager KN, Lijssens N, et al. Does treatment of asthmatic children with inhaled corticosteroids affect their adult height? Pediatr Pulmonol. 1999;27:369-375.

6. Silverstein MD, Yunginger JW, Reed CE, et al. Attained adult height after childhood asthma: effect of glucocorticoid therapy. J Allergy Clin Immunol. 1997;99:466-474.

7. The Childhood Asthma Management Program Research Group. Long-term effects of budesonide or nedocromil in children with asthma. N Engl J Med. 2000;343:1054-1063.

8. Tinkelman DG, Reed CE, Nelson HS, et al. Aerosol beclomethasone dipropionate compared with theophylline as primary treatment of chronic, mild to moderately severe asthma in children. Pediatrics. 1993;92:64-77.

9. Verberne AA, Frost C, Roorda RJ, et al. One year treatment with salmeterol compared with beclomethasone in children with asthma. Am J Respir Crit Care Med. 1997;156:688-695.

10. Allen DB, Bronsky EA, LaForce CF, et al. Growth in asthmatic children treated with fluticasone propionate. J Pediatr 1998;132: 472-477.

11. Skoner DP, Meltzer EO, Milgrom H, et al. Effects of inhaled mometasone furoate on growth velocity and adrenal function: a placebo-controlled trial in children 4-9 years old with mild persistent asthma. J Asthma. 2011;48:848-859.

12. Cochrane MG, Bala MV, Downs KE, et al. Inhaled corticosteroids for asthma therapy: patient compliance, devices, and inhalation technique. Chest. 2000;117:542-550.

References

1. Kelly HW, Sternberg AL, Lescher R, et al; CAMP Research Group. Effect of inhaled glucocorticoids in childhood on adult height. N Engl J Med. 2012;367:904-912.

2. Expert Panel Report 3: Guidelines for the Diagnosis and Management of Asthma. National Institutes of Health National Heart, Lung and Blood Institute: National Asthma Education and Prevention Program, 2007. Available at: http://www.nhlbi. nih.gov/guidelines/asthma/asthgdln.pdf. Accessed August 15, 2013.

3. Chauhan BF, Ducharme FM. Anti-leukotriene agents compared to inhaled corticosteroids in the management of recurrent and/ or chronic asthma in adults and children. Cochrane Database Syst Rev. 2012;(5):CD002314.

4. Agertoft L, Pedersen S. Effect of long- term treatment with budesonide on adult height in children with asthma. N Engl J Med. 2000;343:1064-1069.

5. Van Bever HP, Desager KN, Lijssens N, et al. Does treatment of asthmatic children with inhaled corticosteroids affect their adult height? Pediatr Pulmonol. 1999;27:369-375.

6. Silverstein MD, Yunginger JW, Reed CE, et al. Attained adult height after childhood asthma: effect of glucocorticoid therapy. J Allergy Clin Immunol. 1997;99:466-474.

7. The Childhood Asthma Management Program Research Group. Long-term effects of budesonide or nedocromil in children with asthma. N Engl J Med. 2000;343:1054-1063.

8. Tinkelman DG, Reed CE, Nelson HS, et al. Aerosol beclomethasone dipropionate compared with theophylline as primary treatment of chronic, mild to moderately severe asthma in children. Pediatrics. 1993;92:64-77.

9. Verberne AA, Frost C, Roorda RJ, et al. One year treatment with salmeterol compared with beclomethasone in children with asthma. Am J Respir Crit Care Med. 1997;156:688-695.

10. Allen DB, Bronsky EA, LaForce CF, et al. Growth in asthmatic children treated with fluticasone propionate. J Pediatr 1998;132: 472-477.

11. Skoner DP, Meltzer EO, Milgrom H, et al. Effects of inhaled mometasone furoate on growth velocity and adrenal function: a placebo-controlled trial in children 4-9 years old with mild persistent asthma. J Asthma. 2011;48:848-859.

12. Cochrane MG, Bala MV, Downs KE, et al. Inhaled corticosteroids for asthma therapy: patient compliance, devices, and inhalation technique. Chest. 2000;117:542-550.

Issue
The Journal of Family Practice - 62(9)
Issue
The Journal of Family Practice - 62(9)
Page Number
500-502
Page Number
500-502
Publications
Publications
Topics
Article Type
Display Headline
This asthma treatment has a lasting side effect in children
Display Headline
This asthma treatment has a lasting side effect in children
Legacy Keywords
Tanner Nissly; DO; Shailendra Prasad; MBBS; MPH; asthma; PURLs; inhaled corticosteroids; ICS; Childhood Asthma Management Program; CAMP;
Legacy Keywords
Tanner Nissly; DO; Shailendra Prasad; MBBS; MPH; asthma; PURLs; inhaled corticosteroids; ICS; Childhood Asthma Management Program; CAMP;
Sections
PURLs Copyright

Copyright © 2013 Family Physicians Inquiries Network. All rights reserved.

Disallow All Ads
Alternative CME
Use ProPublica
Hide sidebar & use full width
render the right sidebar.
Conference Recap Checkbox
Not Conference Recap
Clinical Edge
Display the Slideshow in this Article
Article PDF Media
Media Files

Corticosteroids for a Sore Throat?

Article Type
Changed
Tue, 12/13/2016 - 12:08
Display Headline
Corticosteroids for a Sore Throat?
One dose of a steroid can alleviate the pain—and has the potential to decrease unnecessary use of antibiotics.


PRACTICE CHANGER
Consider prescribing a single dose of corticosteroids for patients with sore throat; this has been found to provide quick pain relief and resolution of symptoms.1

STRENGTH OF RECOMMENDATION
A: Based on a meta-analysis of randomized controlled trials (RCTs) in ambulatory care settings.1

ILLUSTRATIVE CASE
A 28-year-old woman comes to your clinic because she’s had a severe sore throat and low-grade fever for the past two days. She has no associated cough. Examination reveals erythematous posterior oropharynx with exudate. A rapid strep test is negative. The patient says the sore throat is very painful and asks for medication to make it better. What should you prescribe?

Most sore throats—particularly in adults—are viral and self-limiting.2,3 Group A β-hemolytic Streptococcus infections account for just 10% of sore throats in adults and 15% to 30% in children.4 Yet US physicians have been found to prescribe antibiotics for more than half of patients who present with sore throat.5-7

Do patients want antibiotics, or simply pain relief?
Antibiotics produce only a modest reduction in symptoms of pharyngitis (fever and throat soreness), presumably in patients with bacterial infections, and increase the risk for adverse events.5,6 Research suggests that patients who request antibiotics for sore throat may be seeking primarily pain relief.8 Thus, a treatment that is more effective in alleviating symptoms of a sore throat would likely contribute to a decrease in unnecessary use of antibiotics.

A short course of corticosteroids has been used successfully and shown to be safe for conditions such as acute sinusitis, croup, and asthma.9-11 Could the anti-inflammatory effects of corticosteroids reduce pain in patients with sore throat, as well? A 2010 systematic review suggested that was the case.12 Cochrane reviewers recently took another look.1

STUDY SUMMARY
Steroids bring speedier pain relief
This meta-analysis included eight RCTs (the same eight trials used in the systematic review9) that compared corticosteroids with placebo for the symptomatic treatment of exudative or severe sore throat.1 Sore throat was defined as clinical evidence of pharyngitis and/or tonsillitis or the clinical syndrome of painful throat and odynophagia.

Five studies were conducted in the United States, and one each in Canada, Turkey, and Israel. Five studies focused on adults (n = 413); the other three studied children (n = 393). Overall, 47% of participants had exudative sore throat, and 44% were positive for group A β-hemolytic Streptococcus.

In all eight RCTs, antibiotics were given to those in both the treatment and placebo groups. In addition, all participants were allowed to use traditional analgesia (either acetaminophen or NSAIDs). Corticosteroids (oral dexamethasone, oral prednisone, or intramuscular [IM] dexamethasone) were used as an adjunctive treatment in all the RCTs.

Primary outcomes varied between studies. Four of the eight RCTs included the proportion of patients with improvement or complete resolution of symptoms within 24 to 48 hours. Mean time to onset of pain relief was the primary outcome in five of the eight studies. Some of the secondary outcomes in the individual trials included relapse rates, adverse events, and days missed from school or work.

Overall, patients who received corticosteroids were three times more likely to report complete resolution of symptoms at 24 hours (relative risk = 3.2) and had a reduced mean time to onset of pain relief of about six hours. The number needed to treat to prevent one patient from experiencing pain at 24 hours was < 4.

Adverse events were reported in only one of the trials (n = 125): Five patients (three in the steroid group and two receiving placebo) were hospitalized for fluid rehydration, and three patients (one in the steroid group and two receiving placebo) developed peritonsillar abscess.12 Three RCTs did not find any significant difference in days missed from school or work, and four trials reported no difference in recurrence of symptoms. One of the trials found that 16% of the patients in the placebo group returned to seek additional care, while none in the steroid group did.13

WHAT’S NEW
Steroids haven’t been tested as standalone treatment

Steroids are not currently recommended for routine use to treat symptoms of sore throat. This Cochrane review found that patients with severe or exudative sore throat benefit from pain reduction with corticosteroids, used as an adjunct to antibiotics and other analgesics without increased risk for harm. Nonetheless, the use of steroids in this patient population would address a practical concern of those seeking symptom relief and has the potential to decrease unnecessary use of antibiotics.

CAVEATS
Questions about effects on antibiotic use, heterogeneity

The studies in this meta-analysis did not assess whether the use of corticosteroids would reduce unnecessary use of antibiotics, so we cannot conclude that this would be the case. Because the effect was similar in all subgroups analyzed, however, it is reasonable to expect that reduced antibiotic use could be a positive effect. The main documented benefit was resolution of pain, an important patient-centered outcome that justifies consideration of treating painful pharyngitis with corticosteroids.

 

 

Corticosteroids have an immunosuppressant effect and carry the theoretical risk for exacerbation of an existing infection. That did not occur in these studies. Nor has it occurred when short courses of corticosteroids are used for other illnesses, such as croup, infectious mononucleosis, asthma, contact dermatitis, and COPD.14 Thus, this theoretical risk is not a barrier to implementation.

It is important to note that single and multiple doses of corticosteroids and oral and IM routes were effective, with only minimal differences in results.

CHALLENGES TO 
IMPLEMENTATION
Determining the severity
Acetaminophen and NSAIDs are used for pain relief in sore throat and have been shown to be effective—but may be inadequate for severe pain.15 There are no head-to-head trials that have compared steroids to NSAIDs or acetaminophen in this clinical scenario. So the challenge for clinicians will be to decide when pharyngitis is severe enough to justify the use of corticosteroids, rather than simple analgesics alone.  

REFERENCES
1. Hayward G, Thompson M, Perera R, et al. Corticosteroids as stand-alone or add-on treatment for sore throat. Cochrane Database Syst Rev. 2012;(10):CDC008268.

2. Cherry DK, Woodwell DA. National Ambulatory Medical Care Survey: 2000 summary. Adv Data. 2002;328:1-32.

3. Bisno AL. Acute pharyngitis. N Engl J Med. 2001;344:205-211.

4. Del Mar CB, Glasziou PP, Sprinks AB. Antibiotics for sore throat. Cochrane Database Syst Rev. 2006;(4):CD000023.

5. Linder JA, Stafford RS. Antibiotic treatment of adults with sore throat by community primary care physicians: a national survey, 1989-1999. JAMA. 2001;286:1181-1186.

6. Linder JA, Bates DW, Lee GM, et al. Antibiotic treatment of children with sore throat. JAMA. 2005;294:2315-2322.

7. Hong SY, Taur Y, Jordan MR. Antimicrobial prescribing in the USA for adult acute pharyngitis in relation to treatment guidelines. J Eval Clin Pract. 2011;17:1176-1183.

8. van Driel ML, De Sutter A, Deveugele M, et al Are sore throat patients who hope for antibiotics actually asking for pain relief? Ann Fam Med. 2006;4:494-499.

9. Venekamp RP, Thompson MJ, Hayward G, et al. Systemic corticosteroids for acute sinusitis. Cochrane Database Syst Rev. 2011;(12): CD008115.

10. Russell KF, Liang Y, O’Gorman K, et al. Glucocorticoids for croup. Cochrane Database Syst Rev. 2011;(1):CD001955.

11. Rowe BH, Spooner C, Ducharme F, et al. Early emergency department treatment of acute asthma with systemic corticosteroids. Cochrane Database Syst Rev. 2001;(1):CD002178.

12. Korb K, Scherer M, Cenot J. Steroids as adjuvant therapy for acute pharyngitis in ambulatory patients: a systematic review. Ann Fam Med. 2010;8:58-63.

13. Olympia RP, Khine H, Avner JR. Effectiveness of oral dexamethasone in the treatment of moderate to severe pharyngitis in children. Arch Pediatr Adolesc Med. 2005;159:278-282.

14. Manson SC, Brown RE, Cerulli A, et al. The cumulative burden of oral corticosteroid side effects and the economic implications of steroid use. Respir Med. 2009;103:975-994.

15. Wei JL, Kasperbauer JL, Weaver AL, et al. Efficacy of single-dose dexamethasone as adjuvant therapy for acute pharyngitis. Laryngoscope. 2002;112:87-93.

Acknowledgement
The PURLs Surveillance System was supported in part by Grant Number UL1RR024999 from the National Center for Research Resources, a Clinical Translational Science Award to the University of Chicago. The content is solely the responsibility of the authors and does not necessarily represent the official views of the National Center for Research Resources or the National Institutes of Health.

Copyright © 2013. The Family Physicians Inquiries Network. All rights reserved.

Reprinted with permission from the Family Physicians Inquiries Network and The Journal of Family Practice. 2013;62(7):372-374.

Author and Disclosure Information

Keri Bergeson, MD, Nina Rogers, MD, Shailendra Prasad, MBBS, MPH

Issue
Clinician Reviews - 23(8)
Publications
Topics
Page Number
18
Legacy Keywords
corticosteroids, sore throat, oropharynx, strep, Streptococcus, pharyngitis, dexamethasone, prednisone
Sections
Author and Disclosure Information

Keri Bergeson, MD, Nina Rogers, MD, Shailendra Prasad, MBBS, MPH

Author and Disclosure Information

Keri Bergeson, MD, Nina Rogers, MD, Shailendra Prasad, MBBS, MPH

One dose of a steroid can alleviate the pain—and has the potential to decrease unnecessary use of antibiotics.

One dose of a steroid can alleviate the pain—and has the potential to decrease unnecessary use of antibiotics.


PRACTICE CHANGER
Consider prescribing a single dose of corticosteroids for patients with sore throat; this has been found to provide quick pain relief and resolution of symptoms.1

STRENGTH OF RECOMMENDATION
A: Based on a meta-analysis of randomized controlled trials (RCTs) in ambulatory care settings.1

ILLUSTRATIVE CASE
A 28-year-old woman comes to your clinic because she’s had a severe sore throat and low-grade fever for the past two days. She has no associated cough. Examination reveals erythematous posterior oropharynx with exudate. A rapid strep test is negative. The patient says the sore throat is very painful and asks for medication to make it better. What should you prescribe?

Most sore throats—particularly in adults—are viral and self-limiting.2,3 Group A β-hemolytic Streptococcus infections account for just 10% of sore throats in adults and 15% to 30% in children.4 Yet US physicians have been found to prescribe antibiotics for more than half of patients who present with sore throat.5-7

Do patients want antibiotics, or simply pain relief?
Antibiotics produce only a modest reduction in symptoms of pharyngitis (fever and throat soreness), presumably in patients with bacterial infections, and increase the risk for adverse events.5,6 Research suggests that patients who request antibiotics for sore throat may be seeking primarily pain relief.8 Thus, a treatment that is more effective in alleviating symptoms of a sore throat would likely contribute to a decrease in unnecessary use of antibiotics.

A short course of corticosteroids has been used successfully and shown to be safe for conditions such as acute sinusitis, croup, and asthma.9-11 Could the anti-inflammatory effects of corticosteroids reduce pain in patients with sore throat, as well? A 2010 systematic review suggested that was the case.12 Cochrane reviewers recently took another look.1

STUDY SUMMARY
Steroids bring speedier pain relief
This meta-analysis included eight RCTs (the same eight trials used in the systematic review9) that compared corticosteroids with placebo for the symptomatic treatment of exudative or severe sore throat.1 Sore throat was defined as clinical evidence of pharyngitis and/or tonsillitis or the clinical syndrome of painful throat and odynophagia.

Five studies were conducted in the United States, and one each in Canada, Turkey, and Israel. Five studies focused on adults (n = 413); the other three studied children (n = 393). Overall, 47% of participants had exudative sore throat, and 44% were positive for group A β-hemolytic Streptococcus.

In all eight RCTs, antibiotics were given to those in both the treatment and placebo groups. In addition, all participants were allowed to use traditional analgesia (either acetaminophen or NSAIDs). Corticosteroids (oral dexamethasone, oral prednisone, or intramuscular [IM] dexamethasone) were used as an adjunctive treatment in all the RCTs.

Primary outcomes varied between studies. Four of the eight RCTs included the proportion of patients with improvement or complete resolution of symptoms within 24 to 48 hours. Mean time to onset of pain relief was the primary outcome in five of the eight studies. Some of the secondary outcomes in the individual trials included relapse rates, adverse events, and days missed from school or work.

Overall, patients who received corticosteroids were three times more likely to report complete resolution of symptoms at 24 hours (relative risk = 3.2) and had a reduced mean time to onset of pain relief of about six hours. The number needed to treat to prevent one patient from experiencing pain at 24 hours was < 4.

Adverse events were reported in only one of the trials (n = 125): Five patients (three in the steroid group and two receiving placebo) were hospitalized for fluid rehydration, and three patients (one in the steroid group and two receiving placebo) developed peritonsillar abscess.12 Three RCTs did not find any significant difference in days missed from school or work, and four trials reported no difference in recurrence of symptoms. One of the trials found that 16% of the patients in the placebo group returned to seek additional care, while none in the steroid group did.13

WHAT’S NEW
Steroids haven’t been tested as standalone treatment

Steroids are not currently recommended for routine use to treat symptoms of sore throat. This Cochrane review found that patients with severe or exudative sore throat benefit from pain reduction with corticosteroids, used as an adjunct to antibiotics and other analgesics without increased risk for harm. Nonetheless, the use of steroids in this patient population would address a practical concern of those seeking symptom relief and has the potential to decrease unnecessary use of antibiotics.

CAVEATS
Questions about effects on antibiotic use, heterogeneity

The studies in this meta-analysis did not assess whether the use of corticosteroids would reduce unnecessary use of antibiotics, so we cannot conclude that this would be the case. Because the effect was similar in all subgroups analyzed, however, it is reasonable to expect that reduced antibiotic use could be a positive effect. The main documented benefit was resolution of pain, an important patient-centered outcome that justifies consideration of treating painful pharyngitis with corticosteroids.

 

 

Corticosteroids have an immunosuppressant effect and carry the theoretical risk for exacerbation of an existing infection. That did not occur in these studies. Nor has it occurred when short courses of corticosteroids are used for other illnesses, such as croup, infectious mononucleosis, asthma, contact dermatitis, and COPD.14 Thus, this theoretical risk is not a barrier to implementation.

It is important to note that single and multiple doses of corticosteroids and oral and IM routes were effective, with only minimal differences in results.

CHALLENGES TO 
IMPLEMENTATION
Determining the severity
Acetaminophen and NSAIDs are used for pain relief in sore throat and have been shown to be effective—but may be inadequate for severe pain.15 There are no head-to-head trials that have compared steroids to NSAIDs or acetaminophen in this clinical scenario. So the challenge for clinicians will be to decide when pharyngitis is severe enough to justify the use of corticosteroids, rather than simple analgesics alone.  

REFERENCES
1. Hayward G, Thompson M, Perera R, et al. Corticosteroids as stand-alone or add-on treatment for sore throat. Cochrane Database Syst Rev. 2012;(10):CDC008268.

2. Cherry DK, Woodwell DA. National Ambulatory Medical Care Survey: 2000 summary. Adv Data. 2002;328:1-32.

3. Bisno AL. Acute pharyngitis. N Engl J Med. 2001;344:205-211.

4. Del Mar CB, Glasziou PP, Sprinks AB. Antibiotics for sore throat. Cochrane Database Syst Rev. 2006;(4):CD000023.

5. Linder JA, Stafford RS. Antibiotic treatment of adults with sore throat by community primary care physicians: a national survey, 1989-1999. JAMA. 2001;286:1181-1186.

6. Linder JA, Bates DW, Lee GM, et al. Antibiotic treatment of children with sore throat. JAMA. 2005;294:2315-2322.

7. Hong SY, Taur Y, Jordan MR. Antimicrobial prescribing in the USA for adult acute pharyngitis in relation to treatment guidelines. J Eval Clin Pract. 2011;17:1176-1183.

8. van Driel ML, De Sutter A, Deveugele M, et al Are sore throat patients who hope for antibiotics actually asking for pain relief? Ann Fam Med. 2006;4:494-499.

9. Venekamp RP, Thompson MJ, Hayward G, et al. Systemic corticosteroids for acute sinusitis. Cochrane Database Syst Rev. 2011;(12): CD008115.

10. Russell KF, Liang Y, O’Gorman K, et al. Glucocorticoids for croup. Cochrane Database Syst Rev. 2011;(1):CD001955.

11. Rowe BH, Spooner C, Ducharme F, et al. Early emergency department treatment of acute asthma with systemic corticosteroids. Cochrane Database Syst Rev. 2001;(1):CD002178.

12. Korb K, Scherer M, Cenot J. Steroids as adjuvant therapy for acute pharyngitis in ambulatory patients: a systematic review. Ann Fam Med. 2010;8:58-63.

13. Olympia RP, Khine H, Avner JR. Effectiveness of oral dexamethasone in the treatment of moderate to severe pharyngitis in children. Arch Pediatr Adolesc Med. 2005;159:278-282.

14. Manson SC, Brown RE, Cerulli A, et al. The cumulative burden of oral corticosteroid side effects and the economic implications of steroid use. Respir Med. 2009;103:975-994.

15. Wei JL, Kasperbauer JL, Weaver AL, et al. Efficacy of single-dose dexamethasone as adjuvant therapy for acute pharyngitis. Laryngoscope. 2002;112:87-93.

Acknowledgement
The PURLs Surveillance System was supported in part by Grant Number UL1RR024999 from the National Center for Research Resources, a Clinical Translational Science Award to the University of Chicago. The content is solely the responsibility of the authors and does not necessarily represent the official views of the National Center for Research Resources or the National Institutes of Health.

Copyright © 2013. The Family Physicians Inquiries Network. All rights reserved.

Reprinted with permission from the Family Physicians Inquiries Network and The Journal of Family Practice. 2013;62(7):372-374.

PRACTICE CHANGER
Consider prescribing a single dose of corticosteroids for patients with sore throat; this has been found to provide quick pain relief and resolution of symptoms.1

STRENGTH OF RECOMMENDATION
A: Based on a meta-analysis of randomized controlled trials (RCTs) in ambulatory care settings.1

ILLUSTRATIVE CASE
A 28-year-old woman comes to your clinic because she’s had a severe sore throat and low-grade fever for the past two days. She has no associated cough. Examination reveals erythematous posterior oropharynx with exudate. A rapid strep test is negative. The patient says the sore throat is very painful and asks for medication to make it better. What should you prescribe?

Most sore throats—particularly in adults—are viral and self-limiting.2,3 Group A β-hemolytic Streptococcus infections account for just 10% of sore throats in adults and 15% to 30% in children.4 Yet US physicians have been found to prescribe antibiotics for more than half of patients who present with sore throat.5-7

Do patients want antibiotics, or simply pain relief?
Antibiotics produce only a modest reduction in symptoms of pharyngitis (fever and throat soreness), presumably in patients with bacterial infections, and increase the risk for adverse events.5,6 Research suggests that patients who request antibiotics for sore throat may be seeking primarily pain relief.8 Thus, a treatment that is more effective in alleviating symptoms of a sore throat would likely contribute to a decrease in unnecessary use of antibiotics.

A short course of corticosteroids has been used successfully and shown to be safe for conditions such as acute sinusitis, croup, and asthma.9-11 Could the anti-inflammatory effects of corticosteroids reduce pain in patients with sore throat, as well? A 2010 systematic review suggested that was the case.12 Cochrane reviewers recently took another look.1

STUDY SUMMARY
Steroids bring speedier pain relief
This meta-analysis included eight RCTs (the same eight trials used in the systematic review9) that compared corticosteroids with placebo for the symptomatic treatment of exudative or severe sore throat.1 Sore throat was defined as clinical evidence of pharyngitis and/or tonsillitis or the clinical syndrome of painful throat and odynophagia.

Five studies were conducted in the United States, and one each in Canada, Turkey, and Israel. Five studies focused on adults (n = 413); the other three studied children (n = 393). Overall, 47% of participants had exudative sore throat, and 44% were positive for group A β-hemolytic Streptococcus.

In all eight RCTs, antibiotics were given to those in both the treatment and placebo groups. In addition, all participants were allowed to use traditional analgesia (either acetaminophen or NSAIDs). Corticosteroids (oral dexamethasone, oral prednisone, or intramuscular [IM] dexamethasone) were used as an adjunctive treatment in all the RCTs.

Primary outcomes varied between studies. Four of the eight RCTs included the proportion of patients with improvement or complete resolution of symptoms within 24 to 48 hours. Mean time to onset of pain relief was the primary outcome in five of the eight studies. Some of the secondary outcomes in the individual trials included relapse rates, adverse events, and days missed from school or work.

Overall, patients who received corticosteroids were three times more likely to report complete resolution of symptoms at 24 hours (relative risk = 3.2) and had a reduced mean time to onset of pain relief of about six hours. The number needed to treat to prevent one patient from experiencing pain at 24 hours was < 4.

Adverse events were reported in only one of the trials (n = 125): Five patients (three in the steroid group and two receiving placebo) were hospitalized for fluid rehydration, and three patients (one in the steroid group and two receiving placebo) developed peritonsillar abscess.12 Three RCTs did not find any significant difference in days missed from school or work, and four trials reported no difference in recurrence of symptoms. One of the trials found that 16% of the patients in the placebo group returned to seek additional care, while none in the steroid group did.13

WHAT’S NEW
Steroids haven’t been tested as standalone treatment

Steroids are not currently recommended for routine use to treat symptoms of sore throat. This Cochrane review found that patients with severe or exudative sore throat benefit from pain reduction with corticosteroids, used as an adjunct to antibiotics and other analgesics without increased risk for harm. Nonetheless, the use of steroids in this patient population would address a practical concern of those seeking symptom relief and has the potential to decrease unnecessary use of antibiotics.

CAVEATS
Questions about effects on antibiotic use, heterogeneity

The studies in this meta-analysis did not assess whether the use of corticosteroids would reduce unnecessary use of antibiotics, so we cannot conclude that this would be the case. Because the effect was similar in all subgroups analyzed, however, it is reasonable to expect that reduced antibiotic use could be a positive effect. The main documented benefit was resolution of pain, an important patient-centered outcome that justifies consideration of treating painful pharyngitis with corticosteroids.

 

 

Corticosteroids have an immunosuppressant effect and carry the theoretical risk for exacerbation of an existing infection. That did not occur in these studies. Nor has it occurred when short courses of corticosteroids are used for other illnesses, such as croup, infectious mononucleosis, asthma, contact dermatitis, and COPD.14 Thus, this theoretical risk is not a barrier to implementation.

It is important to note that single and multiple doses of corticosteroids and oral and IM routes were effective, with only minimal differences in results.

CHALLENGES TO 
IMPLEMENTATION
Determining the severity
Acetaminophen and NSAIDs are used for pain relief in sore throat and have been shown to be effective—but may be inadequate for severe pain.15 There are no head-to-head trials that have compared steroids to NSAIDs or acetaminophen in this clinical scenario. So the challenge for clinicians will be to decide when pharyngitis is severe enough to justify the use of corticosteroids, rather than simple analgesics alone.  

REFERENCES
1. Hayward G, Thompson M, Perera R, et al. Corticosteroids as stand-alone or add-on treatment for sore throat. Cochrane Database Syst Rev. 2012;(10):CDC008268.

2. Cherry DK, Woodwell DA. National Ambulatory Medical Care Survey: 2000 summary. Adv Data. 2002;328:1-32.

3. Bisno AL. Acute pharyngitis. N Engl J Med. 2001;344:205-211.

4. Del Mar CB, Glasziou PP, Sprinks AB. Antibiotics for sore throat. Cochrane Database Syst Rev. 2006;(4):CD000023.

5. Linder JA, Stafford RS. Antibiotic treatment of adults with sore throat by community primary care physicians: a national survey, 1989-1999. JAMA. 2001;286:1181-1186.

6. Linder JA, Bates DW, Lee GM, et al. Antibiotic treatment of children with sore throat. JAMA. 2005;294:2315-2322.

7. Hong SY, Taur Y, Jordan MR. Antimicrobial prescribing in the USA for adult acute pharyngitis in relation to treatment guidelines. J Eval Clin Pract. 2011;17:1176-1183.

8. van Driel ML, De Sutter A, Deveugele M, et al Are sore throat patients who hope for antibiotics actually asking for pain relief? Ann Fam Med. 2006;4:494-499.

9. Venekamp RP, Thompson MJ, Hayward G, et al. Systemic corticosteroids for acute sinusitis. Cochrane Database Syst Rev. 2011;(12): CD008115.

10. Russell KF, Liang Y, O’Gorman K, et al. Glucocorticoids for croup. Cochrane Database Syst Rev. 2011;(1):CD001955.

11. Rowe BH, Spooner C, Ducharme F, et al. Early emergency department treatment of acute asthma with systemic corticosteroids. Cochrane Database Syst Rev. 2001;(1):CD002178.

12. Korb K, Scherer M, Cenot J. Steroids as adjuvant therapy for acute pharyngitis in ambulatory patients: a systematic review. Ann Fam Med. 2010;8:58-63.

13. Olympia RP, Khine H, Avner JR. Effectiveness of oral dexamethasone in the treatment of moderate to severe pharyngitis in children. Arch Pediatr Adolesc Med. 2005;159:278-282.

14. Manson SC, Brown RE, Cerulli A, et al. The cumulative burden of oral corticosteroid side effects and the economic implications of steroid use. Respir Med. 2009;103:975-994.

15. Wei JL, Kasperbauer JL, Weaver AL, et al. Efficacy of single-dose dexamethasone as adjuvant therapy for acute pharyngitis. Laryngoscope. 2002;112:87-93.

Acknowledgement
The PURLs Surveillance System was supported in part by Grant Number UL1RR024999 from the National Center for Research Resources, a Clinical Translational Science Award to the University of Chicago. The content is solely the responsibility of the authors and does not necessarily represent the official views of the National Center for Research Resources or the National Institutes of Health.

Copyright © 2013. The Family Physicians Inquiries Network. All rights reserved.

Reprinted with permission from the Family Physicians Inquiries Network and The Journal of Family Practice. 2013;62(7):372-374.

Issue
Clinician Reviews - 23(8)
Issue
Clinician Reviews - 23(8)
Page Number
18
Page Number
18
Publications
Publications
Topics
Article Type
Display Headline
Corticosteroids for a Sore Throat?
Display Headline
Corticosteroids for a Sore Throat?
Legacy Keywords
corticosteroids, sore throat, oropharynx, strep, Streptococcus, pharyngitis, dexamethasone, prednisone
Legacy Keywords
corticosteroids, sore throat, oropharynx, strep, Streptococcus, pharyngitis, dexamethasone, prednisone
Sections
Article Source

PURLs Copyright

Inside the Article

Is this pregnancy viable?

Article Type
Changed
Tue, 07/14/2020 - 14:52
Display Headline
Is this pregnancy viable?
PRACTICE CHANGER

Measure serum progesterone levels of women with bleeding or pain and inconclusive ultrasound in early pregnancy to rule out viability, potentially eliminating the need for serial b-hormone human chorionic gonadotropin (b-hCG) testing.1

STRENGTH OF RECOMMENDATION
A: Based on a systematic review and meta-analysis of 26 diagnostic accuracy studies.

Verhaegen J, Gallos ID, van Mello NM, et al. Accuracy of a single progesterone test to predict early pregnancy outcome in women with pain or bleeding: meta-analysis of cohort studies. BMJ. 2012;345:e6077.

 

Illustrative case

A 20-year-old woman with an estimated gestational age of 7 weeks comes to your clinic because of vaginal bleeding, which started 4 hours ago. A transvaginal ultrasound is inconclusive for an intrauterine pregnancy. Should you obtain a serum progesterone measurement?

Between 21% and 27% of pregnant women experience vaginal bleeding in their first trimester.2,3 This leads to concern, both for patients and physicians, as it can be the first sign of a miscarriage or an ectopic pregnancy. A longitudinal population-based Swedish study of women who had ever been pregnant found that one in 4 had experienced an early pregnancy failure. Overall, about 12% of clinically recognized pregnancies ended in miscarriage.4

Our ability to predict early pregnancy loss is limited

Tools used by clinicians to evaluate vaginal bleeding or pain in the first trimester include transvaginal ultrasound (TVUS) and serial serum β-hCG measurements.5 Even when combined with risk factors for pregnancy loss (serum levels of estradiol, inhibin A, and inhibin B; maternal age; smoking; past history of spontaneous miscarriage; and vaginal bleeding), TVUS is not accurate at predicting early pregnancy loss.6,7

A suboptimal rise in β-hCG (<66%) after 48 hours has historically been used to indicate possible miscarriage or ectopic pregnancy,1but studies have found similarly low rates of increase in some viable pregnancies, as well.8,9 And β-hCG measurements need to be done on more than one occasion, making this an inconvenient means of predicting miscarriage.

Moreover, β-hCG levels vary based on gestational age, leaving family physicians with no solid diagnostic rule regarding the appropriate level of rise in a viable pregnancy.10 Thus, there is a need for a test that complements TVUS and β-hCG to increase diagnostic accuracy in predicting nonviable pregnancies.

Can serum progesterone testing fill the gap?
Serum progesterone measurement is a noninvasive predictive tool, with low values associated with miscarriage and ectopic pregnancy and higher levels with a viable pregnancy.10   Studies have found that serum progesterone combined with β-hCG measurements has the highest reliability in predicting nonviable pregnancy, with a diagnostic accuracy of 85.7% (sensitivity, 88.1%; specificity, 84.3%). This compares with a diagnostic accuracy of 72.5% (sensitivity, 76.1%; specificity, 70.4%) for a single progesterone test alone, and 74.8% (sensitivity, 64.1%; specificity, 81.4%) for β-hCG alone.10-12 The data are from older studies, including a meta-analysis, that did not include the use of TVUS.10-12 But TVUS is now in widespread use and included in the systematic review and meta-analysis this PURL addresses.

Study summary:

Progesterone test is predictive—when combined with ultrasound
Verhaegen et al performed a comprehensive literature search to identify studies in which a single serum progesterone measurement was used to predict the viability of pregnancy vs miscarriage or ectopic pregnancy. They included studies of women with spontaneous pregnancy of <14 weeks. Trials of women who had conceived after ovulation induction or in vitro fertilization or received progesterone supplementation were excluded.

Twenty-six cohort studies met the inclusion criteria. These included 7 mostly high-quality studies, with a total of 2379 women with pain or bleeding and inconclusive TVUS, and 19 intermediate-quality studies (n=7057) of women who had pain or bleeding but no ultrasound.

Five of the 7 studies in women with symptoms and inconclusive TVUS had a similar progesterone test cutoff value (3.2-6 ng/mL). In these 5 studies (n=1998), the progesterone test predicted a nonviable pregnancy with a pooled sensitivity of 74.6% (95% CI, 50.6%-89.4%) and specificity of 98.4% (95% CI, 90.9%-99.7%), a positive likelihood ratio of 45 (7.1- 289) and a negative likelihood ratio of 0.26 (0.12-0.57). When progesterone was below the cutoff value, the probability of a nonviable pregnancy increased to 99.2%. In women with pain or bleeding but no ultrasound, a single progesterone test is less accurate in ruling out a viable pregnancy.

 

 

 

What's new

This test can end days of anxious waiting

This meta-analysis provides strong evidence that a single progesterone measurement is useful in predicting nonviable pregnancies in women with pain or bleeding when TVUS is inconclusive. In such patients, a low serum progesterone is highly predictive of a nonviable pregnancy.1 This finding enables the physician to counsel the woman immediately on the likely pregnancy loss, without waiting days for serial β-hCG results.

Caveats

Progesterone is a poor predictor of ectopic pregnancy

An important caveat to our recommendation is that a single serum progesterone test has a poor predictive value for ectopic pregnancy and should not be used for this purpose. A combination of TVUS and serial β-hCG remains the optimal strategy for diagnosing ectopic pregnancy.13

It is important to note that there is no universally accepted definition of a low serum progesterone level: This meta-analysis included studies with a cutoff value of 3.2 to 6 ng/mL in women who had had a previous ultrasound. What’s more, these studies did not evaluate the predictive value of a serum progesterone test combined with β-hCG measurements.

Challenges to implementation

There are none
We do not see any challenges to the implementation of this recommendation.

ACKNOWLEDGEMENT 

The PURLs Surveillance System was supported in part by Grant Number UL1RR024999 from the National Center for Research Resources, a Clinical Translational Science Award to the University of Chicago. The content is solely the responsibility of the authors and does not necessarily represent the official views of the National Center of Research Resources or the National Institutes of Health.

Files
References

1. Verhaegen J, Gallos ID, van Mello NM, et al. Accuracy of single progesterone test to predict early pregnancy outcome in women with pain or bleeding: meta-analysis of cohort studies. BMJ. 2012;345:e6077.

2. Hasan R, Baird DD, Herring AH, et al. Patterns and predictors of vaginal bleeding in the first trimester of pregnancy. Ann Epidemiol. 2010;20:524-531.

3. Everett C. Incidence and outcome of bleeding before the 20th week of pregnancy: prospective study from general practice. BMJ. 1997;315:32-34.

4. Blohm F, Friden B, Milsom I. A prospective longitudinal population-based study of clinical miscarriage in an urban Swedish population. BJOG. 2008;115:176-183.
Chen BA, Creinin MD. Contemporary management of early pregnancy failure. Clin Obstet Gynecol. 2007;50:67-88.

5. Jauniaux E, Johns J, Burton GJ. The role of ultrasound imaging in diagnosing and investigating early pregnancy failure. Ultrasound Obstet Gynecol. 2005;25:613–624.

6. Gagnon A, Wilson RD, Audibert F, et al. Obstetrical complications associated with abnormal maternal serum markers analytes.J Obstet Gynaecol Can. 2008;30:918-949.

7. Barnhart KT, Sammel MD, Rinaudo PF, et al. Symptomatic patients with an early viable intrauterine pregnancy; hCG curves redefined. Obstet Gynecol. 2004;104:50-54.

8. Morse CB, Sammel MD, Shaunik A, et al. Performance of human chorionic gonadotropin curves in women at risk for ectopic pregnancy: exceptions to the rules. Fertil Steril. 2012;97:101e2-106.e2.

9. Mol BW, Lijmer JG, Ankum WM, et al. The accuracy of single serum progesterone measurement in the diagnosis of ectopic pregnancy: a meta-analysis. Hum Reprod. 1998;13:3220-3227.

10. Duan L, Yan D, Zeng W, et al. Predictive power of progesterone combined with beta human chorionic gonadotropin measurements in the outcome of threatened miscarriage. Arch Gynecol Obstet. 2011;283:431-4355.

11. Phipps MG, Hogan JW, Peipert JF, et al. Progesterone, inhibin, and hCG multiple marker strategy to differentiate viable from nonviable pregnancies. Obstet Gynecol. 2000;95:227-231.

12. American College of Obstetricians and Gynecologists. Practice Bulletin no. 94: Medical management of ectopic pregnancy. Obstet Gynecol. 2008;111:1479-1485.

Author and Disclosure Information

Andrew H. Slattengren, DO; Shailendra Prasad, MBBS, MPH; Sonia Oyola, MD
North Memorial Family Medicine Residency, University of Minnesota, Minneapolis (Drs. Slattengren and Prasad); Department of Family Medicine, The University of Chicago (Dr. Oyola)

PURLs Editor
Anne Mounsey, MD
University of North Carolina at Chapel Hill

Issue
The Journal of Family Practice - 62(06)
Publications
Topics
Page Number
305-305, 316
Legacy Keywords
Andrew H. Slattengren; DO; Shailendra Prasad; MBBS; MPH; Sonia Oyola; MD; PURLs; pregnancy; serum progesterone; β-hCG; TVUS; progesterone test
Sections
Files
Files
Author and Disclosure Information

Andrew H. Slattengren, DO; Shailendra Prasad, MBBS, MPH; Sonia Oyola, MD
North Memorial Family Medicine Residency, University of Minnesota, Minneapolis (Drs. Slattengren and Prasad); Department of Family Medicine, The University of Chicago (Dr. Oyola)

PURLs Editor
Anne Mounsey, MD
University of North Carolina at Chapel Hill

Author and Disclosure Information

Andrew H. Slattengren, DO; Shailendra Prasad, MBBS, MPH; Sonia Oyola, MD
North Memorial Family Medicine Residency, University of Minnesota, Minneapolis (Drs. Slattengren and Prasad); Department of Family Medicine, The University of Chicago (Dr. Oyola)

PURLs Editor
Anne Mounsey, MD
University of North Carolina at Chapel Hill

PRACTICE CHANGER

Measure serum progesterone levels of women with bleeding or pain and inconclusive ultrasound in early pregnancy to rule out viability, potentially eliminating the need for serial b-hormone human chorionic gonadotropin (b-hCG) testing.1

STRENGTH OF RECOMMENDATION
A: Based on a systematic review and meta-analysis of 26 diagnostic accuracy studies.

Verhaegen J, Gallos ID, van Mello NM, et al. Accuracy of a single progesterone test to predict early pregnancy outcome in women with pain or bleeding: meta-analysis of cohort studies. BMJ. 2012;345:e6077.

 

Illustrative case

A 20-year-old woman with an estimated gestational age of 7 weeks comes to your clinic because of vaginal bleeding, which started 4 hours ago. A transvaginal ultrasound is inconclusive for an intrauterine pregnancy. Should you obtain a serum progesterone measurement?

Between 21% and 27% of pregnant women experience vaginal bleeding in their first trimester.2,3 This leads to concern, both for patients and physicians, as it can be the first sign of a miscarriage or an ectopic pregnancy. A longitudinal population-based Swedish study of women who had ever been pregnant found that one in 4 had experienced an early pregnancy failure. Overall, about 12% of clinically recognized pregnancies ended in miscarriage.4

Our ability to predict early pregnancy loss is limited

Tools used by clinicians to evaluate vaginal bleeding or pain in the first trimester include transvaginal ultrasound (TVUS) and serial serum β-hCG measurements.5 Even when combined with risk factors for pregnancy loss (serum levels of estradiol, inhibin A, and inhibin B; maternal age; smoking; past history of spontaneous miscarriage; and vaginal bleeding), TVUS is not accurate at predicting early pregnancy loss.6,7

A suboptimal rise in β-hCG (<66%) after 48 hours has historically been used to indicate possible miscarriage or ectopic pregnancy,1but studies have found similarly low rates of increase in some viable pregnancies, as well.8,9 And β-hCG measurements need to be done on more than one occasion, making this an inconvenient means of predicting miscarriage.

Moreover, β-hCG levels vary based on gestational age, leaving family physicians with no solid diagnostic rule regarding the appropriate level of rise in a viable pregnancy.10 Thus, there is a need for a test that complements TVUS and β-hCG to increase diagnostic accuracy in predicting nonviable pregnancies.

Can serum progesterone testing fill the gap?
Serum progesterone measurement is a noninvasive predictive tool, with low values associated with miscarriage and ectopic pregnancy and higher levels with a viable pregnancy.10   Studies have found that serum progesterone combined with β-hCG measurements has the highest reliability in predicting nonviable pregnancy, with a diagnostic accuracy of 85.7% (sensitivity, 88.1%; specificity, 84.3%). This compares with a diagnostic accuracy of 72.5% (sensitivity, 76.1%; specificity, 70.4%) for a single progesterone test alone, and 74.8% (sensitivity, 64.1%; specificity, 81.4%) for β-hCG alone.10-12 The data are from older studies, including a meta-analysis, that did not include the use of TVUS.10-12 But TVUS is now in widespread use and included in the systematic review and meta-analysis this PURL addresses.

Study summary:

Progesterone test is predictive—when combined with ultrasound
Verhaegen et al performed a comprehensive literature search to identify studies in which a single serum progesterone measurement was used to predict the viability of pregnancy vs miscarriage or ectopic pregnancy. They included studies of women with spontaneous pregnancy of <14 weeks. Trials of women who had conceived after ovulation induction or in vitro fertilization or received progesterone supplementation were excluded.

Twenty-six cohort studies met the inclusion criteria. These included 7 mostly high-quality studies, with a total of 2379 women with pain or bleeding and inconclusive TVUS, and 19 intermediate-quality studies (n=7057) of women who had pain or bleeding but no ultrasound.

Five of the 7 studies in women with symptoms and inconclusive TVUS had a similar progesterone test cutoff value (3.2-6 ng/mL). In these 5 studies (n=1998), the progesterone test predicted a nonviable pregnancy with a pooled sensitivity of 74.6% (95% CI, 50.6%-89.4%) and specificity of 98.4% (95% CI, 90.9%-99.7%), a positive likelihood ratio of 45 (7.1- 289) and a negative likelihood ratio of 0.26 (0.12-0.57). When progesterone was below the cutoff value, the probability of a nonviable pregnancy increased to 99.2%. In women with pain or bleeding but no ultrasound, a single progesterone test is less accurate in ruling out a viable pregnancy.

 

 

 

What's new

This test can end days of anxious waiting

This meta-analysis provides strong evidence that a single progesterone measurement is useful in predicting nonviable pregnancies in women with pain or bleeding when TVUS is inconclusive. In such patients, a low serum progesterone is highly predictive of a nonviable pregnancy.1 This finding enables the physician to counsel the woman immediately on the likely pregnancy loss, without waiting days for serial β-hCG results.

Caveats

Progesterone is a poor predictor of ectopic pregnancy

An important caveat to our recommendation is that a single serum progesterone test has a poor predictive value for ectopic pregnancy and should not be used for this purpose. A combination of TVUS and serial β-hCG remains the optimal strategy for diagnosing ectopic pregnancy.13

It is important to note that there is no universally accepted definition of a low serum progesterone level: This meta-analysis included studies with a cutoff value of 3.2 to 6 ng/mL in women who had had a previous ultrasound. What’s more, these studies did not evaluate the predictive value of a serum progesterone test combined with β-hCG measurements.

Challenges to implementation

There are none
We do not see any challenges to the implementation of this recommendation.

ACKNOWLEDGEMENT 

The PURLs Surveillance System was supported in part by Grant Number UL1RR024999 from the National Center for Research Resources, a Clinical Translational Science Award to the University of Chicago. The content is solely the responsibility of the authors and does not necessarily represent the official views of the National Center of Research Resources or the National Institutes of Health.

PRACTICE CHANGER

Measure serum progesterone levels of women with bleeding or pain and inconclusive ultrasound in early pregnancy to rule out viability, potentially eliminating the need for serial b-hormone human chorionic gonadotropin (b-hCG) testing.1

STRENGTH OF RECOMMENDATION
A: Based on a systematic review and meta-analysis of 26 diagnostic accuracy studies.

Verhaegen J, Gallos ID, van Mello NM, et al. Accuracy of a single progesterone test to predict early pregnancy outcome in women with pain or bleeding: meta-analysis of cohort studies. BMJ. 2012;345:e6077.

 

Illustrative case

A 20-year-old woman with an estimated gestational age of 7 weeks comes to your clinic because of vaginal bleeding, which started 4 hours ago. A transvaginal ultrasound is inconclusive for an intrauterine pregnancy. Should you obtain a serum progesterone measurement?

Between 21% and 27% of pregnant women experience vaginal bleeding in their first trimester.2,3 This leads to concern, both for patients and physicians, as it can be the first sign of a miscarriage or an ectopic pregnancy. A longitudinal population-based Swedish study of women who had ever been pregnant found that one in 4 had experienced an early pregnancy failure. Overall, about 12% of clinically recognized pregnancies ended in miscarriage.4

Our ability to predict early pregnancy loss is limited

Tools used by clinicians to evaluate vaginal bleeding or pain in the first trimester include transvaginal ultrasound (TVUS) and serial serum β-hCG measurements.5 Even when combined with risk factors for pregnancy loss (serum levels of estradiol, inhibin A, and inhibin B; maternal age; smoking; past history of spontaneous miscarriage; and vaginal bleeding), TVUS is not accurate at predicting early pregnancy loss.6,7

A suboptimal rise in β-hCG (<66%) after 48 hours has historically been used to indicate possible miscarriage or ectopic pregnancy,1but studies have found similarly low rates of increase in some viable pregnancies, as well.8,9 And β-hCG measurements need to be done on more than one occasion, making this an inconvenient means of predicting miscarriage.

Moreover, β-hCG levels vary based on gestational age, leaving family physicians with no solid diagnostic rule regarding the appropriate level of rise in a viable pregnancy.10 Thus, there is a need for a test that complements TVUS and β-hCG to increase diagnostic accuracy in predicting nonviable pregnancies.

Can serum progesterone testing fill the gap?
Serum progesterone measurement is a noninvasive predictive tool, with low values associated with miscarriage and ectopic pregnancy and higher levels with a viable pregnancy.10   Studies have found that serum progesterone combined with β-hCG measurements has the highest reliability in predicting nonviable pregnancy, with a diagnostic accuracy of 85.7% (sensitivity, 88.1%; specificity, 84.3%). This compares with a diagnostic accuracy of 72.5% (sensitivity, 76.1%; specificity, 70.4%) for a single progesterone test alone, and 74.8% (sensitivity, 64.1%; specificity, 81.4%) for β-hCG alone.10-12 The data are from older studies, including a meta-analysis, that did not include the use of TVUS.10-12 But TVUS is now in widespread use and included in the systematic review and meta-analysis this PURL addresses.

Study summary:

Progesterone test is predictive—when combined with ultrasound
Verhaegen et al performed a comprehensive literature search to identify studies in which a single serum progesterone measurement was used to predict the viability of pregnancy vs miscarriage or ectopic pregnancy. They included studies of women with spontaneous pregnancy of <14 weeks. Trials of women who had conceived after ovulation induction or in vitro fertilization or received progesterone supplementation were excluded.

Twenty-six cohort studies met the inclusion criteria. These included 7 mostly high-quality studies, with a total of 2379 women with pain or bleeding and inconclusive TVUS, and 19 intermediate-quality studies (n=7057) of women who had pain or bleeding but no ultrasound.

Five of the 7 studies in women with symptoms and inconclusive TVUS had a similar progesterone test cutoff value (3.2-6 ng/mL). In these 5 studies (n=1998), the progesterone test predicted a nonviable pregnancy with a pooled sensitivity of 74.6% (95% CI, 50.6%-89.4%) and specificity of 98.4% (95% CI, 90.9%-99.7%), a positive likelihood ratio of 45 (7.1- 289) and a negative likelihood ratio of 0.26 (0.12-0.57). When progesterone was below the cutoff value, the probability of a nonviable pregnancy increased to 99.2%. In women with pain or bleeding but no ultrasound, a single progesterone test is less accurate in ruling out a viable pregnancy.

 

 

 

What's new

This test can end days of anxious waiting

This meta-analysis provides strong evidence that a single progesterone measurement is useful in predicting nonviable pregnancies in women with pain or bleeding when TVUS is inconclusive. In such patients, a low serum progesterone is highly predictive of a nonviable pregnancy.1 This finding enables the physician to counsel the woman immediately on the likely pregnancy loss, without waiting days for serial β-hCG results.

Caveats

Progesterone is a poor predictor of ectopic pregnancy

An important caveat to our recommendation is that a single serum progesterone test has a poor predictive value for ectopic pregnancy and should not be used for this purpose. A combination of TVUS and serial β-hCG remains the optimal strategy for diagnosing ectopic pregnancy.13

It is important to note that there is no universally accepted definition of a low serum progesterone level: This meta-analysis included studies with a cutoff value of 3.2 to 6 ng/mL in women who had had a previous ultrasound. What’s more, these studies did not evaluate the predictive value of a serum progesterone test combined with β-hCG measurements.

Challenges to implementation

There are none
We do not see any challenges to the implementation of this recommendation.

ACKNOWLEDGEMENT 

The PURLs Surveillance System was supported in part by Grant Number UL1RR024999 from the National Center for Research Resources, a Clinical Translational Science Award to the University of Chicago. The content is solely the responsibility of the authors and does not necessarily represent the official views of the National Center of Research Resources or the National Institutes of Health.

References

1. Verhaegen J, Gallos ID, van Mello NM, et al. Accuracy of single progesterone test to predict early pregnancy outcome in women with pain or bleeding: meta-analysis of cohort studies. BMJ. 2012;345:e6077.

2. Hasan R, Baird DD, Herring AH, et al. Patterns and predictors of vaginal bleeding in the first trimester of pregnancy. Ann Epidemiol. 2010;20:524-531.

3. Everett C. Incidence and outcome of bleeding before the 20th week of pregnancy: prospective study from general practice. BMJ. 1997;315:32-34.

4. Blohm F, Friden B, Milsom I. A prospective longitudinal population-based study of clinical miscarriage in an urban Swedish population. BJOG. 2008;115:176-183.
Chen BA, Creinin MD. Contemporary management of early pregnancy failure. Clin Obstet Gynecol. 2007;50:67-88.

5. Jauniaux E, Johns J, Burton GJ. The role of ultrasound imaging in diagnosing and investigating early pregnancy failure. Ultrasound Obstet Gynecol. 2005;25:613–624.

6. Gagnon A, Wilson RD, Audibert F, et al. Obstetrical complications associated with abnormal maternal serum markers analytes.J Obstet Gynaecol Can. 2008;30:918-949.

7. Barnhart KT, Sammel MD, Rinaudo PF, et al. Symptomatic patients with an early viable intrauterine pregnancy; hCG curves redefined. Obstet Gynecol. 2004;104:50-54.

8. Morse CB, Sammel MD, Shaunik A, et al. Performance of human chorionic gonadotropin curves in women at risk for ectopic pregnancy: exceptions to the rules. Fertil Steril. 2012;97:101e2-106.e2.

9. Mol BW, Lijmer JG, Ankum WM, et al. The accuracy of single serum progesterone measurement in the diagnosis of ectopic pregnancy: a meta-analysis. Hum Reprod. 1998;13:3220-3227.

10. Duan L, Yan D, Zeng W, et al. Predictive power of progesterone combined with beta human chorionic gonadotropin measurements in the outcome of threatened miscarriage. Arch Gynecol Obstet. 2011;283:431-4355.

11. Phipps MG, Hogan JW, Peipert JF, et al. Progesterone, inhibin, and hCG multiple marker strategy to differentiate viable from nonviable pregnancies. Obstet Gynecol. 2000;95:227-231.

12. American College of Obstetricians and Gynecologists. Practice Bulletin no. 94: Medical management of ectopic pregnancy. Obstet Gynecol. 2008;111:1479-1485.

References

1. Verhaegen J, Gallos ID, van Mello NM, et al. Accuracy of single progesterone test to predict early pregnancy outcome in women with pain or bleeding: meta-analysis of cohort studies. BMJ. 2012;345:e6077.

2. Hasan R, Baird DD, Herring AH, et al. Patterns and predictors of vaginal bleeding in the first trimester of pregnancy. Ann Epidemiol. 2010;20:524-531.

3. Everett C. Incidence and outcome of bleeding before the 20th week of pregnancy: prospective study from general practice. BMJ. 1997;315:32-34.

4. Blohm F, Friden B, Milsom I. A prospective longitudinal population-based study of clinical miscarriage in an urban Swedish population. BJOG. 2008;115:176-183.
Chen BA, Creinin MD. Contemporary management of early pregnancy failure. Clin Obstet Gynecol. 2007;50:67-88.

5. Jauniaux E, Johns J, Burton GJ. The role of ultrasound imaging in diagnosing and investigating early pregnancy failure. Ultrasound Obstet Gynecol. 2005;25:613–624.

6. Gagnon A, Wilson RD, Audibert F, et al. Obstetrical complications associated with abnormal maternal serum markers analytes.J Obstet Gynaecol Can. 2008;30:918-949.

7. Barnhart KT, Sammel MD, Rinaudo PF, et al. Symptomatic patients with an early viable intrauterine pregnancy; hCG curves redefined. Obstet Gynecol. 2004;104:50-54.

8. Morse CB, Sammel MD, Shaunik A, et al. Performance of human chorionic gonadotropin curves in women at risk for ectopic pregnancy: exceptions to the rules. Fertil Steril. 2012;97:101e2-106.e2.

9. Mol BW, Lijmer JG, Ankum WM, et al. The accuracy of single serum progesterone measurement in the diagnosis of ectopic pregnancy: a meta-analysis. Hum Reprod. 1998;13:3220-3227.

10. Duan L, Yan D, Zeng W, et al. Predictive power of progesterone combined with beta human chorionic gonadotropin measurements in the outcome of threatened miscarriage. Arch Gynecol Obstet. 2011;283:431-4355.

11. Phipps MG, Hogan JW, Peipert JF, et al. Progesterone, inhibin, and hCG multiple marker strategy to differentiate viable from nonviable pregnancies. Obstet Gynecol. 2000;95:227-231.

12. American College of Obstetricians and Gynecologists. Practice Bulletin no. 94: Medical management of ectopic pregnancy. Obstet Gynecol. 2008;111:1479-1485.

Issue
The Journal of Family Practice - 62(06)
Issue
The Journal of Family Practice - 62(06)
Page Number
305-305, 316
Page Number
305-305, 316
Publications
Publications
Topics
Article Type
Display Headline
Is this pregnancy viable?
Display Headline
Is this pregnancy viable?
Legacy Keywords
Andrew H. Slattengren; DO; Shailendra Prasad; MBBS; MPH; Sonia Oyola; MD; PURLs; pregnancy; serum progesterone; β-hCG; TVUS; progesterone test
Legacy Keywords
Andrew H. Slattengren; DO; Shailendra Prasad; MBBS; MPH; Sonia Oyola; MD; PURLs; pregnancy; serum progesterone; β-hCG; TVUS; progesterone test
Sections
PURLs Copyright

Copyright © 2013. The Family Physicians Inquiries Network. All rights reserved.

Disallow All Ads
Alternative CME
Use ProPublica
Hide sidebar & use full width
render the right sidebar.
Conference Recap Checkbox
Not Conference Recap
Clinical Edge
Display the Slideshow in this Article
Media Files

Corticosteroids for a sore throat?

Article Type
Changed
Tue, 07/14/2020 - 14:42
Display Headline
Corticosteroids for a sore throat?

PRACTICE CHANGER

Consider prescribing a single dose of corticosteroids for patients with sore throat, which has been found to bring quicker pain relief and resolution of symptoms.1

STRENGTH OF RECOMMENDATION

A: Based on a meta-analysis of randomized controlled trials (RCTs) in ambulatory care settings.

Hayward G, Thompson M, Perera R, et al. Corticosteroids as standalone or add-on treatment for sore throat. Cochrane Database Syst Rev. 2012;(10):CDC008268.

 

Illustrative case

A 28-year-old woman comes to your clinic because she’s had a severe sore throat and low-grade fever for the past 2 days. She has no associated cough. Examination reveals erythematous posterior oropharynx with exudate. A rapid strep test is negative. The patient says the sore throat is very painful and asks for medication to make it better. What should you prescribe?

Most sore throats—particularly in adults—are viral and self-limiting.2,3 Group A B-hemolytic Streptococcusinfections account for just 10% of sore throats in adults and 15% to 30% in children.4 Yet US physicians have been found to prescribe antibiotics for more than half of patients who present with sore throat.5-7

Do patients want antibiotics, or simply pain relief?Antibiotics produce only a modest reduction in symptoms of pharyngitis (fever and throat soreness), presumably in patients with bacterial infections, and increase the risk of adverse events.5,6 Research suggests that patients who request antibiotics for sore throat may primarily be seeking pain relief.8 Thus, a treatment that’s more effective in alleviating symptoms of a sore throat would likely contribute to a decrease in unnecessary use of antibiotics.

A short course of corticosteroids has been used successfully and shown to be safe for conditions such as acute sinusitis, croup, and asthma.9-11 Could the anti-inflammatory effects of corticosteroids reduce pain in patients with sore throat, as well? A 2010 systematic review suggested that was the case.12 Cochrane reviewers recently took another look.1

Study summary: Steroids bring speedier pain relief

This meta-analysis included 8 RCTs (the same 8 trials used in the systematic review9) that compared corticosteroids with placebo for the symptomatic treatment of exudative or severe sore throat.1 Sore throat was defined as clinical evidence of pharyngitis and/or tonsillitis or the clinical syndrome of painful throat and odynophagia.

Five studies were conducted in the United States, and one each in Canada, Turkey, and Israel. Five studies focused on adults (N=413); the other 3 studied children (N=393). Overall, 47% of participants had exudative sore throat, and 44% were positive for group A B-hemolytic Streptococcus.

In all 8 RCTs, antibiotics were given to those in both the treatment and placebo groups. In addition, all participants were allowed to use traditional analgesia— either acetaminophen or nonsteroidal anti-inflammatory drugs. Corticosteroids (oral dexamethasone, oral prednisone, or intramuscular [IM] dexamethasone) were used as an adjunctive treatment in all the RCTs.

Primary outcomes varied among the studies. Four of the 8 RCTs included the proportion of patients with improvement or complete resolution of symptoms within 24 to 48 hours. Mean time to onset of pain relief was the primary outcome in 5 of the 8 studies. Some of the secondary outcomes in the individual trials included relapse rates, adverse events, and days missed from school or work.

Overall, patients who received corticosteroids were 3 times more likely to report complete resolution of symptoms at 24 hours (relative risk=3.2; 95% confidence interval, 2.0-5.1; P<.001) and had a reduced mean time to onset of pain relief of about 6 hours. The number needed to treat to prevent one patient from experiencing pain at 24 hours was <4.

Adverse events were reported in only one of the trials (N=125): Five patients (3 in the steroid group and 2 on placebo) were hospitalized for fluid rehydration, and 3 patients (one in the steroid group and 2 on placebo) developed peritonsillar abscess.12 Three RCTs did not find any significant difference in days missed from school or work, and 4 trials reported no difference in recurrence of symptoms. One of the trials found that 16% of the patients in the placebo group returned to seek additional care, while none in the steroid group did.13

What's new: Steroids haven't been tested as standalone treatment

Steroids are not currently recommended for routine use to treat symptoms of sore throat. This Cochrane review found that patients with severe or exudative sore throat benefit from pain reduction with corticosteroids, used as an adjunct to antibiotics and other analgesics without increased risk of harm. Nonetheless, the use of steroids in this patient population would address a practical concern of those seeking symptom relief and has the potential to decrease unnecessary antibiotic use.

 

 

 

Caveats: Questions about effects on antibiotic use, heterogeneity remain

The studies in this meta-analysis did not assess whether the use corticosteroids would reduce unnecessary use of antibiotics, so we cannot conclude that this would be the case. Because the effect was similar in all sub-groups analyzed, however, it is reasonable to expect that reduced antibiotic use could be a positive effect. The main documented benefit was resolution of pain, an important patient-centered outcome that justifies consideration of treating painful pharyngitis with corticosteroids.                                                                                                     

Corticosteroids have an immunosuppressant effect and carry the FAST TRACK
Research suggests that patients who request antibiotics for a sore throat may be seeking pain relief.
 theoretical risk of exacerbating an existing infection. That did not occur in these studies. Nor has it occurred when used for short courses in other illnesses such as croup, infectious mononucleosis, asthma, contact dermatitis, and chronic obstructive pulmonary disease.14 Thus, this theoretical risk is not a barrier to implementation.

It is important to note that single and multiple doses of corticosteroids and oral and IM routes were effective, with only minimal differences in results.  

Challenges to implementation: Determining the severity

Acetaminophen and NSAIDs are used for pain relief in sore throat, and have been shown to be effective—but may be inadequate for severe pain.15 There are no head-to-head trials that have compared steroids to NSAIDs or acetaminophen in this clinical scenario.  So the challenge for clinicians will be to decide when pharyngitis is severe enough to justify the use of corticosteroids, rather than simple analgesics alone.

Acknowledgement

The PURLs Surveillance System was supported in part by Grant number ul1rr024999 from the national center for research resources, a clinical Translational Science award to the university of chicago. The content is solely the responsibility of the authors and does not necessarily represent the official views of the national center for research resources or the national institutes of health.

Files
References

1.    Hayward G, Thompson M, Perera R, et al. Corticosteroids as stand-alone or add-on treatment for sore throat. Cochrane
Database Syst Rev
. 2012;(10):CDC008268.

2.    Cherry DK, Woodwell DA. National Ambulatory Medical Care Survey: 2000 summary. Adv Data. 2002;328:1-32.

3.    Bisno AL. Acute pharyngitis. N Engl J Med . 2001;344:205-211.

4.    Del Mar CB, Glasziou PP, Sprinks AB. Antibiotics for sore throat. Cochrane Database Syst Rev. 2006;(4):CD000023.

5.    Linder JA, Stafford RS. Antibiotic treatment of adults with sore throat by community primary care physicians: a national survey, 1989-1999. JAMA. 2001;286:1181-1186.

6.    Linder JA, Bates DW, Lee GM, et al. Antibiotic treatment of children with sore throat. JAMA. 2005;294:2315-2322.

7.    Hong SY, Taur Y, Jordan MR. Antimicrobial prescribing in the USA for adult acute pharyngitis in relation to treatment guidelines.
J Eval Clin Pract. 2011;17: 1176-1183.

8.    van Driel ML, De Sutter A, Deveugele M, et al Are sore throat patients who hope for antibiotics actually asking for pain relief? Ann Fam Med. 2006;4:494-499.

9.    Venekamp RP, Thompson MJ, Hayward G, et al. Systemic corticosteroids for acute sinusitis. Cochrane Database Syst Rev. 2011;(12):CD008115.

10.  Russell KF, Liang Y, O’Gorman K, et al. Glucocorticoids for croup. Cochrane Database Syst Rev. 2011;(1):CD001955.

11.  Rowe BH, Spooner C, Ducharme F, et al. Early emergency department treatment of acute asthma with systemic corticosteroids. Cochrane Database Syst Rev. 2001;(1):CD002178.

12.  Korb K, Scherer M, Cenot J. Steroids as adjuvant therapy for acute pharyngitis in ambulatory patients: a systematic review. Ann Fam Med. 2010;8:58-63.

13.  Olympia RP, Khine H, Avner JR. Effectiveness of oral dexamethasone in the treatment of moderate to severe pharyngitis in children. Arch Pediatr Adolesc Med. 2005;159:278-282.

14.  Manson SC, Brown RE, Cerulli A, et al. The cumulative burden of oral corticosteroid side effects and the economic implications of steroid use. Respir Med. 2009;103:975-994.

15.  Wei JL, Kasperbauer JL, Weaver AL, et al. Efficacy of single-dose dexamethasone as adjuvant therapy for acute pharyngitis. Laryngoscope. 2002;112:87-93.

Article PDF
Author and Disclosure Information

Keri Bergeson, MD
North Memorial Family Medicine Residency Program, University of Minnesota

Nina Rogers, MD
Department of Family Medicine, The University of Chicago

Shailendra Prasad, MBBS, MPH
North Memorial Family Medicine Residency Program, University of Minnesota

PURLs EDITOR
Bernard Ewigman, MD, MSPH
Department of Family Medicine, The University of Chicago

Issue
The Journal of Family Practice - 62(7)
Publications
Topics
Page Number
372-374
Legacy Keywords
Keri Bergeson; MD; Nina Rogers; MD; Shailendra Prasad; MBBS; MPH; sore throat; corticosteroids; PURLs; steroids; NSAIDs
Sections
Files
Files
Author and Disclosure Information

Keri Bergeson, MD
North Memorial Family Medicine Residency Program, University of Minnesota

Nina Rogers, MD
Department of Family Medicine, The University of Chicago

Shailendra Prasad, MBBS, MPH
North Memorial Family Medicine Residency Program, University of Minnesota

PURLs EDITOR
Bernard Ewigman, MD, MSPH
Department of Family Medicine, The University of Chicago

Author and Disclosure Information

Keri Bergeson, MD
North Memorial Family Medicine Residency Program, University of Minnesota

Nina Rogers, MD
Department of Family Medicine, The University of Chicago

Shailendra Prasad, MBBS, MPH
North Memorial Family Medicine Residency Program, University of Minnesota

PURLs EDITOR
Bernard Ewigman, MD, MSPH
Department of Family Medicine, The University of Chicago

Article PDF
Article PDF

PRACTICE CHANGER

Consider prescribing a single dose of corticosteroids for patients with sore throat, which has been found to bring quicker pain relief and resolution of symptoms.1

STRENGTH OF RECOMMENDATION

A: Based on a meta-analysis of randomized controlled trials (RCTs) in ambulatory care settings.

Hayward G, Thompson M, Perera R, et al. Corticosteroids as standalone or add-on treatment for sore throat. Cochrane Database Syst Rev. 2012;(10):CDC008268.

 

Illustrative case

A 28-year-old woman comes to your clinic because she’s had a severe sore throat and low-grade fever for the past 2 days. She has no associated cough. Examination reveals erythematous posterior oropharynx with exudate. A rapid strep test is negative. The patient says the sore throat is very painful and asks for medication to make it better. What should you prescribe?

Most sore throats—particularly in adults—are viral and self-limiting.2,3 Group A B-hemolytic Streptococcusinfections account for just 10% of sore throats in adults and 15% to 30% in children.4 Yet US physicians have been found to prescribe antibiotics for more than half of patients who present with sore throat.5-7

Do patients want antibiotics, or simply pain relief?Antibiotics produce only a modest reduction in symptoms of pharyngitis (fever and throat soreness), presumably in patients with bacterial infections, and increase the risk of adverse events.5,6 Research suggests that patients who request antibiotics for sore throat may primarily be seeking pain relief.8 Thus, a treatment that’s more effective in alleviating symptoms of a sore throat would likely contribute to a decrease in unnecessary use of antibiotics.

A short course of corticosteroids has been used successfully and shown to be safe for conditions such as acute sinusitis, croup, and asthma.9-11 Could the anti-inflammatory effects of corticosteroids reduce pain in patients with sore throat, as well? A 2010 systematic review suggested that was the case.12 Cochrane reviewers recently took another look.1

Study summary: Steroids bring speedier pain relief

This meta-analysis included 8 RCTs (the same 8 trials used in the systematic review9) that compared corticosteroids with placebo for the symptomatic treatment of exudative or severe sore throat.1 Sore throat was defined as clinical evidence of pharyngitis and/or tonsillitis or the clinical syndrome of painful throat and odynophagia.

Five studies were conducted in the United States, and one each in Canada, Turkey, and Israel. Five studies focused on adults (N=413); the other 3 studied children (N=393). Overall, 47% of participants had exudative sore throat, and 44% were positive for group A B-hemolytic Streptococcus.

In all 8 RCTs, antibiotics were given to those in both the treatment and placebo groups. In addition, all participants were allowed to use traditional analgesia— either acetaminophen or nonsteroidal anti-inflammatory drugs. Corticosteroids (oral dexamethasone, oral prednisone, or intramuscular [IM] dexamethasone) were used as an adjunctive treatment in all the RCTs.

Primary outcomes varied among the studies. Four of the 8 RCTs included the proportion of patients with improvement or complete resolution of symptoms within 24 to 48 hours. Mean time to onset of pain relief was the primary outcome in 5 of the 8 studies. Some of the secondary outcomes in the individual trials included relapse rates, adverse events, and days missed from school or work.

Overall, patients who received corticosteroids were 3 times more likely to report complete resolution of symptoms at 24 hours (relative risk=3.2; 95% confidence interval, 2.0-5.1; P<.001) and had a reduced mean time to onset of pain relief of about 6 hours. The number needed to treat to prevent one patient from experiencing pain at 24 hours was <4.

Adverse events were reported in only one of the trials (N=125): Five patients (3 in the steroid group and 2 on placebo) were hospitalized for fluid rehydration, and 3 patients (one in the steroid group and 2 on placebo) developed peritonsillar abscess.12 Three RCTs did not find any significant difference in days missed from school or work, and 4 trials reported no difference in recurrence of symptoms. One of the trials found that 16% of the patients in the placebo group returned to seek additional care, while none in the steroid group did.13

What's new: Steroids haven't been tested as standalone treatment

Steroids are not currently recommended for routine use to treat symptoms of sore throat. This Cochrane review found that patients with severe or exudative sore throat benefit from pain reduction with corticosteroids, used as an adjunct to antibiotics and other analgesics without increased risk of harm. Nonetheless, the use of steroids in this patient population would address a practical concern of those seeking symptom relief and has the potential to decrease unnecessary antibiotic use.

 

 

 

Caveats: Questions about effects on antibiotic use, heterogeneity remain

The studies in this meta-analysis did not assess whether the use corticosteroids would reduce unnecessary use of antibiotics, so we cannot conclude that this would be the case. Because the effect was similar in all sub-groups analyzed, however, it is reasonable to expect that reduced antibiotic use could be a positive effect. The main documented benefit was resolution of pain, an important patient-centered outcome that justifies consideration of treating painful pharyngitis with corticosteroids.                                                                                                     

Corticosteroids have an immunosuppressant effect and carry the FAST TRACK
Research suggests that patients who request antibiotics for a sore throat may be seeking pain relief.
 theoretical risk of exacerbating an existing infection. That did not occur in these studies. Nor has it occurred when used for short courses in other illnesses such as croup, infectious mononucleosis, asthma, contact dermatitis, and chronic obstructive pulmonary disease.14 Thus, this theoretical risk is not a barrier to implementation.

It is important to note that single and multiple doses of corticosteroids and oral and IM routes were effective, with only minimal differences in results.  

Challenges to implementation: Determining the severity

Acetaminophen and NSAIDs are used for pain relief in sore throat, and have been shown to be effective—but may be inadequate for severe pain.15 There are no head-to-head trials that have compared steroids to NSAIDs or acetaminophen in this clinical scenario.  So the challenge for clinicians will be to decide when pharyngitis is severe enough to justify the use of corticosteroids, rather than simple analgesics alone.

Acknowledgement

The PURLs Surveillance System was supported in part by Grant number ul1rr024999 from the national center for research resources, a clinical Translational Science award to the university of chicago. The content is solely the responsibility of the authors and does not necessarily represent the official views of the national center for research resources or the national institutes of health.

PRACTICE CHANGER

Consider prescribing a single dose of corticosteroids for patients with sore throat, which has been found to bring quicker pain relief and resolution of symptoms.1

STRENGTH OF RECOMMENDATION

A: Based on a meta-analysis of randomized controlled trials (RCTs) in ambulatory care settings.

Hayward G, Thompson M, Perera R, et al. Corticosteroids as standalone or add-on treatment for sore throat. Cochrane Database Syst Rev. 2012;(10):CDC008268.

 

Illustrative case

A 28-year-old woman comes to your clinic because she’s had a severe sore throat and low-grade fever for the past 2 days. She has no associated cough. Examination reveals erythematous posterior oropharynx with exudate. A rapid strep test is negative. The patient says the sore throat is very painful and asks for medication to make it better. What should you prescribe?

Most sore throats—particularly in adults—are viral and self-limiting.2,3 Group A B-hemolytic Streptococcusinfections account for just 10% of sore throats in adults and 15% to 30% in children.4 Yet US physicians have been found to prescribe antibiotics for more than half of patients who present with sore throat.5-7

Do patients want antibiotics, or simply pain relief?Antibiotics produce only a modest reduction in symptoms of pharyngitis (fever and throat soreness), presumably in patients with bacterial infections, and increase the risk of adverse events.5,6 Research suggests that patients who request antibiotics for sore throat may primarily be seeking pain relief.8 Thus, a treatment that’s more effective in alleviating symptoms of a sore throat would likely contribute to a decrease in unnecessary use of antibiotics.

A short course of corticosteroids has been used successfully and shown to be safe for conditions such as acute sinusitis, croup, and asthma.9-11 Could the anti-inflammatory effects of corticosteroids reduce pain in patients with sore throat, as well? A 2010 systematic review suggested that was the case.12 Cochrane reviewers recently took another look.1

Study summary: Steroids bring speedier pain relief

This meta-analysis included 8 RCTs (the same 8 trials used in the systematic review9) that compared corticosteroids with placebo for the symptomatic treatment of exudative or severe sore throat.1 Sore throat was defined as clinical evidence of pharyngitis and/or tonsillitis or the clinical syndrome of painful throat and odynophagia.

Five studies were conducted in the United States, and one each in Canada, Turkey, and Israel. Five studies focused on adults (N=413); the other 3 studied children (N=393). Overall, 47% of participants had exudative sore throat, and 44% were positive for group A B-hemolytic Streptococcus.

In all 8 RCTs, antibiotics were given to those in both the treatment and placebo groups. In addition, all participants were allowed to use traditional analgesia— either acetaminophen or nonsteroidal anti-inflammatory drugs. Corticosteroids (oral dexamethasone, oral prednisone, or intramuscular [IM] dexamethasone) were used as an adjunctive treatment in all the RCTs.

Primary outcomes varied among the studies. Four of the 8 RCTs included the proportion of patients with improvement or complete resolution of symptoms within 24 to 48 hours. Mean time to onset of pain relief was the primary outcome in 5 of the 8 studies. Some of the secondary outcomes in the individual trials included relapse rates, adverse events, and days missed from school or work.

Overall, patients who received corticosteroids were 3 times more likely to report complete resolution of symptoms at 24 hours (relative risk=3.2; 95% confidence interval, 2.0-5.1; P<.001) and had a reduced mean time to onset of pain relief of about 6 hours. The number needed to treat to prevent one patient from experiencing pain at 24 hours was <4.

Adverse events were reported in only one of the trials (N=125): Five patients (3 in the steroid group and 2 on placebo) were hospitalized for fluid rehydration, and 3 patients (one in the steroid group and 2 on placebo) developed peritonsillar abscess.12 Three RCTs did not find any significant difference in days missed from school or work, and 4 trials reported no difference in recurrence of symptoms. One of the trials found that 16% of the patients in the placebo group returned to seek additional care, while none in the steroid group did.13

What's new: Steroids haven't been tested as standalone treatment

Steroids are not currently recommended for routine use to treat symptoms of sore throat. This Cochrane review found that patients with severe or exudative sore throat benefit from pain reduction with corticosteroids, used as an adjunct to antibiotics and other analgesics without increased risk of harm. Nonetheless, the use of steroids in this patient population would address a practical concern of those seeking symptom relief and has the potential to decrease unnecessary antibiotic use.

 

 

 

Caveats: Questions about effects on antibiotic use, heterogeneity remain

The studies in this meta-analysis did not assess whether the use corticosteroids would reduce unnecessary use of antibiotics, so we cannot conclude that this would be the case. Because the effect was similar in all sub-groups analyzed, however, it is reasonable to expect that reduced antibiotic use could be a positive effect. The main documented benefit was resolution of pain, an important patient-centered outcome that justifies consideration of treating painful pharyngitis with corticosteroids.                                                                                                     

Corticosteroids have an immunosuppressant effect and carry the FAST TRACK
Research suggests that patients who request antibiotics for a sore throat may be seeking pain relief.
 theoretical risk of exacerbating an existing infection. That did not occur in these studies. Nor has it occurred when used for short courses in other illnesses such as croup, infectious mononucleosis, asthma, contact dermatitis, and chronic obstructive pulmonary disease.14 Thus, this theoretical risk is not a barrier to implementation.

It is important to note that single and multiple doses of corticosteroids and oral and IM routes were effective, with only minimal differences in results.  

Challenges to implementation: Determining the severity

Acetaminophen and NSAIDs are used for pain relief in sore throat, and have been shown to be effective—but may be inadequate for severe pain.15 There are no head-to-head trials that have compared steroids to NSAIDs or acetaminophen in this clinical scenario.  So the challenge for clinicians will be to decide when pharyngitis is severe enough to justify the use of corticosteroids, rather than simple analgesics alone.

Acknowledgement

The PURLs Surveillance System was supported in part by Grant number ul1rr024999 from the national center for research resources, a clinical Translational Science award to the university of chicago. The content is solely the responsibility of the authors and does not necessarily represent the official views of the national center for research resources or the national institutes of health.

References

1.    Hayward G, Thompson M, Perera R, et al. Corticosteroids as stand-alone or add-on treatment for sore throat. Cochrane
Database Syst Rev
. 2012;(10):CDC008268.

2.    Cherry DK, Woodwell DA. National Ambulatory Medical Care Survey: 2000 summary. Adv Data. 2002;328:1-32.

3.    Bisno AL. Acute pharyngitis. N Engl J Med . 2001;344:205-211.

4.    Del Mar CB, Glasziou PP, Sprinks AB. Antibiotics for sore throat. Cochrane Database Syst Rev. 2006;(4):CD000023.

5.    Linder JA, Stafford RS. Antibiotic treatment of adults with sore throat by community primary care physicians: a national survey, 1989-1999. JAMA. 2001;286:1181-1186.

6.    Linder JA, Bates DW, Lee GM, et al. Antibiotic treatment of children with sore throat. JAMA. 2005;294:2315-2322.

7.    Hong SY, Taur Y, Jordan MR. Antimicrobial prescribing in the USA for adult acute pharyngitis in relation to treatment guidelines.
J Eval Clin Pract. 2011;17: 1176-1183.

8.    van Driel ML, De Sutter A, Deveugele M, et al Are sore throat patients who hope for antibiotics actually asking for pain relief? Ann Fam Med. 2006;4:494-499.

9.    Venekamp RP, Thompson MJ, Hayward G, et al. Systemic corticosteroids for acute sinusitis. Cochrane Database Syst Rev. 2011;(12):CD008115.

10.  Russell KF, Liang Y, O’Gorman K, et al. Glucocorticoids for croup. Cochrane Database Syst Rev. 2011;(1):CD001955.

11.  Rowe BH, Spooner C, Ducharme F, et al. Early emergency department treatment of acute asthma with systemic corticosteroids. Cochrane Database Syst Rev. 2001;(1):CD002178.

12.  Korb K, Scherer M, Cenot J. Steroids as adjuvant therapy for acute pharyngitis in ambulatory patients: a systematic review. Ann Fam Med. 2010;8:58-63.

13.  Olympia RP, Khine H, Avner JR. Effectiveness of oral dexamethasone in the treatment of moderate to severe pharyngitis in children. Arch Pediatr Adolesc Med. 2005;159:278-282.

14.  Manson SC, Brown RE, Cerulli A, et al. The cumulative burden of oral corticosteroid side effects and the economic implications of steroid use. Respir Med. 2009;103:975-994.

15.  Wei JL, Kasperbauer JL, Weaver AL, et al. Efficacy of single-dose dexamethasone as adjuvant therapy for acute pharyngitis. Laryngoscope. 2002;112:87-93.

References

1.    Hayward G, Thompson M, Perera R, et al. Corticosteroids as stand-alone or add-on treatment for sore throat. Cochrane
Database Syst Rev
. 2012;(10):CDC008268.

2.    Cherry DK, Woodwell DA. National Ambulatory Medical Care Survey: 2000 summary. Adv Data. 2002;328:1-32.

3.    Bisno AL. Acute pharyngitis. N Engl J Med . 2001;344:205-211.

4.    Del Mar CB, Glasziou PP, Sprinks AB. Antibiotics for sore throat. Cochrane Database Syst Rev. 2006;(4):CD000023.

5.    Linder JA, Stafford RS. Antibiotic treatment of adults with sore throat by community primary care physicians: a national survey, 1989-1999. JAMA. 2001;286:1181-1186.

6.    Linder JA, Bates DW, Lee GM, et al. Antibiotic treatment of children with sore throat. JAMA. 2005;294:2315-2322.

7.    Hong SY, Taur Y, Jordan MR. Antimicrobial prescribing in the USA for adult acute pharyngitis in relation to treatment guidelines.
J Eval Clin Pract. 2011;17: 1176-1183.

8.    van Driel ML, De Sutter A, Deveugele M, et al Are sore throat patients who hope for antibiotics actually asking for pain relief? Ann Fam Med. 2006;4:494-499.

9.    Venekamp RP, Thompson MJ, Hayward G, et al. Systemic corticosteroids for acute sinusitis. Cochrane Database Syst Rev. 2011;(12):CD008115.

10.  Russell KF, Liang Y, O’Gorman K, et al. Glucocorticoids for croup. Cochrane Database Syst Rev. 2011;(1):CD001955.

11.  Rowe BH, Spooner C, Ducharme F, et al. Early emergency department treatment of acute asthma with systemic corticosteroids. Cochrane Database Syst Rev. 2001;(1):CD002178.

12.  Korb K, Scherer M, Cenot J. Steroids as adjuvant therapy for acute pharyngitis in ambulatory patients: a systematic review. Ann Fam Med. 2010;8:58-63.

13.  Olympia RP, Khine H, Avner JR. Effectiveness of oral dexamethasone in the treatment of moderate to severe pharyngitis in children. Arch Pediatr Adolesc Med. 2005;159:278-282.

14.  Manson SC, Brown RE, Cerulli A, et al. The cumulative burden of oral corticosteroid side effects and the economic implications of steroid use. Respir Med. 2009;103:975-994.

15.  Wei JL, Kasperbauer JL, Weaver AL, et al. Efficacy of single-dose dexamethasone as adjuvant therapy for acute pharyngitis. Laryngoscope. 2002;112:87-93.

Issue
The Journal of Family Practice - 62(7)
Issue
The Journal of Family Practice - 62(7)
Page Number
372-374
Page Number
372-374
Publications
Publications
Topics
Article Type
Display Headline
Corticosteroids for a sore throat?
Display Headline
Corticosteroids for a sore throat?
Legacy Keywords
Keri Bergeson; MD; Nina Rogers; MD; Shailendra Prasad; MBBS; MPH; sore throat; corticosteroids; PURLs; steroids; NSAIDs
Legacy Keywords
Keri Bergeson; MD; Nina Rogers; MD; Shailendra Prasad; MBBS; MPH; sore throat; corticosteroids; PURLs; steroids; NSAIDs
Sections
PURLs Copyright

Copyright © 2013. The Family Physicians Inquiries Network. All Rights Reserved.

Disallow All Ads
Alternative CME
Use ProPublica
Hide sidebar & use full width
render the right sidebar.
Conference Recap Checkbox
Not Conference Recap
Clinical Edge
Display the Slideshow in this Article
Article PDF Media
Media Files

Patient overusing antianxiety meds? Say so (in a letter)

Article Type
Changed
Mon, 01/14/2019 - 11:35
Display Headline
Patient overusing antianxiety meds? Say so (in a letter)

PRACTICE CHANGER

Express your concern about long-term use of benzodiazepines in a letter—a simple intervention that patients often respond to by reducing or eliminating their use of the drug.1

STRENGTH OF RECOMMENDATION

A: Based on a well-done meta-analysis with few clinical trials.

Mugunthan K, McGuire T, Glasziou P. Minimal interventions to decrease long-term use of benzodiazepines in primary care: a systematic review and meta-analysis. Br J Gen Pract. 2011;61:e573-e578.

ILLUSTRATIVE CASE

A 65-year-old patient has been taking lorazepam for insomnia for more than a year. You are concerned about her ongoing use of the benzodiazepine and want to wean her from the medication. What strategies can you use to decrease, or eliminate, her use of the drug?

Benzodiazepines are commonly used medications, with an estimated 12-month prevalence of use of 8.6% in the United States.2 While short-term use of these antianxiety medications can be effective, long-term use (defined as regular use for >3 months) is associated with significant risk.

Abuse linked to chronic use
Prescription drug abuse has recently become the nation’s leading cause of accidental death, overtaking motor vehicle accidents.3 And tranquilizers, including benzodiazepines, are the second most abused prescription medication, after pain relievers.4 In addition to dependence and withdrawal, chronic use of benzodiazepines is associated with daytime somnolence, blunted reflexes, memory loss, cognitive impairment, and an increased risk of falls and fractures—particularly in older patients.5

Reducing long-term use of benzodiazepines in a primary care setting is important but challenging. Until recently, most of the successful strategies reported were resource intensive and required multiple office visits.6

STUDY SUMMARY: Brief interventions are often effective

This study was a meta-analysis of randomized controlled trials in which “minimal interventions” were compared with usual care for their effectiveness in reducing or eliminating benzodiazepine use in primary care patients. A minimal intervention was defined as a letter, self-help information, or short consultation with a primary care provider. In each case, the message to the patient included (a) an expression of concern about the patient’s long-term use of the medication, (b) information about the potential adverse effects of the medication, and (c) advice on how to gradually reduce or stop using it.

Three studies met the inclusion criteria for randomization, blinding, and analysis by intention-to-treat.7-9 All 3 (n=615) had a 6-month follow-up period, a higher proportion of women (>60%), and participants with a mean age >60 years. Few patients were lost to follow-up; withdrawal rates were low and similar in all 3 studies. Each study compared a letter with usual care; 2 of the 3 had a third arm that included both a letter and a short consultation.

Pooled results from the studies showed twice the reduction in benzodiazepine use in the intervention groups compared with the control groups (risk ratio [RR]=2.04; 95% confidence interval [CI], 1.5-2.8; P< .001). The RR for cessation of benzodiazepine use was 2.3 (95% CI, 1.3-4.2; P= .003). The number needed to treat for a reduction or cessation of use was 12. The studies reported benzodiazepine reduction rates of 20% to 35% in the intervention groups vs 6% to 15% in the usual care groups.7-9 There appeared to be no additional benefit to adding the brief consultation compared with the letter alone.

WHAT’S NEW?: This strategy is easy to implement

While many methods to reduce benzodiazepine use have been studied, most involved levels of skill and resources that are not feasible for widespread use. This study found that a letter, stating the risks of continued use of the medication and providing a weaning schedule and tips for handling withdrawal, can be effective in reducing chronic use in a small but significant part of the population.

CAVEATS: Effects of withdrawal went unaddressed

The study did not adequately address the adverse effects of withdrawal from benzodiazepines, with one of the studies reporting significantly worse qualitative (but not quantitative) withdrawal symptoms at 6 months.7 This is of particular concern, as withdrawal symptoms are associated with the potential for relapse and concomitant abuse of other drugs and alcohol. We recommend that primary care physicians screen for substance abuse prior to the intervention and arrange for adequate follow-up.

All 3 studies in the meta-analysis lasted 6 months; no longer-term outcomes were reported. In addition, the study did not yield enough information to identify patients who would be most likely to respond to this brief intervention.

CHALLENGES TO IMPLEMENTATION: Determining which patients to target

 

 

Identifying patients who are appropriate candidates for this brief intervention and providing adequate monitoring for adverse effects of withdrawal are the main challenges of this practice changer. Nonetheless, chronic benzodiazepine use is of considerable concern, and we believe that this is a useful, and manageable intervention.

Acknowledgement

The PURLs Surveillance System is supported in part by Grant Number UL1RR024999 from the National Center for Research Resources, a Clinical Translational Science Award to the University of Chicago. The content is solely the responsibility of the authors and does not necessarily represent the official views of the National Center for Research Resources or the National Institutes of Health.

References

1. Mugunthan K, McGuire T, Glasziou P. Minimal interventions to decrease long-term use of benzodiazepines in primary care: a systematic review and meta-analysis. Br J Gen Pract. 2011;61:e573-e578.

2. Tyrer PJ. Benzodiazepines on trial. Br Med J. 1984;288:1101-1102.

3. Centers for Disease Control and Prevention. Deaths: Leading causes for 2008. June 6, 2012. Available at: http://www.cdc.gov/nchs/data/nvsr/nvsr60/nvsr60_06.pdf. Accessed October 10, 2012.

4. National Institute on Drug Abuse. Topics in brief: Prescription drug abuse. Available at: http://www.drugabuse.gov/publications/topics-in-brief/prescription-drug-abuse. Accessed October 11, 2012.

5. Morin CM, Bastien C, Guay B, et al. Randomized clinical trail of supervised tapering and cognitive behavior therapy to facilitate benzodiazepine discontinuation in older adults with chronic insomnia. Am J Psychiatry. 2004;161:332-342.

6. Oude Voshaar RC, Couvee JE, van Balkorn AJ, et al. Strategies for discontinuing long-term benzodiazepine use-meta-analysis. Br J Psychiatr. 2006;189:213-220.

7. Bashir K, King M, Ashworth M. Controlled evaluation of brief intervention by general practitioners to reduce chronic use of benzodiazepines. Br J Gen Pract. 1994;44:408-412.

8. Cormack MA, Sweeney KG, Hughes-Jones H, et al. Evaluation of an easy, cost-effective strategy to cut benzodiazepine use in general practice. Br J Gen Pract. 1994;44:5-8

9. Heather NA, Bowie A, Ashton H, et al. Randomized controlled trial of two brief interventions against long-term benzodiazepine use: outcome of intervention. Addict Res Theory. 2004;12:141-145.

Article PDF
Author and Disclosure Information

Robert Levy, MD
North Memorial, Family Medicine Residency, University of Minnesota, Minneapolis

Shailendra Prasad, MBBS, MPH
North Memorial, Family Medicine Residency, University of Minnesota, Minneapolis

PURLs EDITOR
James J. Stevermer, MD, MSPH

University of Missouri-Columbia

Issue
The Journal of Family Practice - 61(11)
Publications
Topics
Page Number
671-672
Legacy Keywords
Robert Levy;MD; Shailendra Prasad;MBBS;MPH; benzodiazepines; minimal interventions; inclusion criteria; primary care patients; short consultation; letter; withdrawal rates
Sections
Author and Disclosure Information

Robert Levy, MD
North Memorial, Family Medicine Residency, University of Minnesota, Minneapolis

Shailendra Prasad, MBBS, MPH
North Memorial, Family Medicine Residency, University of Minnesota, Minneapolis

PURLs EDITOR
James J. Stevermer, MD, MSPH

University of Missouri-Columbia

Author and Disclosure Information

Robert Levy, MD
North Memorial, Family Medicine Residency, University of Minnesota, Minneapolis

Shailendra Prasad, MBBS, MPH
North Memorial, Family Medicine Residency, University of Minnesota, Minneapolis

PURLs EDITOR
James J. Stevermer, MD, MSPH

University of Missouri-Columbia

Article PDF
Article PDF

PRACTICE CHANGER

Express your concern about long-term use of benzodiazepines in a letter—a simple intervention that patients often respond to by reducing or eliminating their use of the drug.1

STRENGTH OF RECOMMENDATION

A: Based on a well-done meta-analysis with few clinical trials.

Mugunthan K, McGuire T, Glasziou P. Minimal interventions to decrease long-term use of benzodiazepines in primary care: a systematic review and meta-analysis. Br J Gen Pract. 2011;61:e573-e578.

ILLUSTRATIVE CASE

A 65-year-old patient has been taking lorazepam for insomnia for more than a year. You are concerned about her ongoing use of the benzodiazepine and want to wean her from the medication. What strategies can you use to decrease, or eliminate, her use of the drug?

Benzodiazepines are commonly used medications, with an estimated 12-month prevalence of use of 8.6% in the United States.2 While short-term use of these antianxiety medications can be effective, long-term use (defined as regular use for >3 months) is associated with significant risk.

Abuse linked to chronic use
Prescription drug abuse has recently become the nation’s leading cause of accidental death, overtaking motor vehicle accidents.3 And tranquilizers, including benzodiazepines, are the second most abused prescription medication, after pain relievers.4 In addition to dependence and withdrawal, chronic use of benzodiazepines is associated with daytime somnolence, blunted reflexes, memory loss, cognitive impairment, and an increased risk of falls and fractures—particularly in older patients.5

Reducing long-term use of benzodiazepines in a primary care setting is important but challenging. Until recently, most of the successful strategies reported were resource intensive and required multiple office visits.6

STUDY SUMMARY: Brief interventions are often effective

This study was a meta-analysis of randomized controlled trials in which “minimal interventions” were compared with usual care for their effectiveness in reducing or eliminating benzodiazepine use in primary care patients. A minimal intervention was defined as a letter, self-help information, or short consultation with a primary care provider. In each case, the message to the patient included (a) an expression of concern about the patient’s long-term use of the medication, (b) information about the potential adverse effects of the medication, and (c) advice on how to gradually reduce or stop using it.

Three studies met the inclusion criteria for randomization, blinding, and analysis by intention-to-treat.7-9 All 3 (n=615) had a 6-month follow-up period, a higher proportion of women (>60%), and participants with a mean age >60 years. Few patients were lost to follow-up; withdrawal rates were low and similar in all 3 studies. Each study compared a letter with usual care; 2 of the 3 had a third arm that included both a letter and a short consultation.

Pooled results from the studies showed twice the reduction in benzodiazepine use in the intervention groups compared with the control groups (risk ratio [RR]=2.04; 95% confidence interval [CI], 1.5-2.8; P< .001). The RR for cessation of benzodiazepine use was 2.3 (95% CI, 1.3-4.2; P= .003). The number needed to treat for a reduction or cessation of use was 12. The studies reported benzodiazepine reduction rates of 20% to 35% in the intervention groups vs 6% to 15% in the usual care groups.7-9 There appeared to be no additional benefit to adding the brief consultation compared with the letter alone.

WHAT’S NEW?: This strategy is easy to implement

While many methods to reduce benzodiazepine use have been studied, most involved levels of skill and resources that are not feasible for widespread use. This study found that a letter, stating the risks of continued use of the medication and providing a weaning schedule and tips for handling withdrawal, can be effective in reducing chronic use in a small but significant part of the population.

CAVEATS: Effects of withdrawal went unaddressed

The study did not adequately address the adverse effects of withdrawal from benzodiazepines, with one of the studies reporting significantly worse qualitative (but not quantitative) withdrawal symptoms at 6 months.7 This is of particular concern, as withdrawal symptoms are associated with the potential for relapse and concomitant abuse of other drugs and alcohol. We recommend that primary care physicians screen for substance abuse prior to the intervention and arrange for adequate follow-up.

All 3 studies in the meta-analysis lasted 6 months; no longer-term outcomes were reported. In addition, the study did not yield enough information to identify patients who would be most likely to respond to this brief intervention.

CHALLENGES TO IMPLEMENTATION: Determining which patients to target

 

 

Identifying patients who are appropriate candidates for this brief intervention and providing adequate monitoring for adverse effects of withdrawal are the main challenges of this practice changer. Nonetheless, chronic benzodiazepine use is of considerable concern, and we believe that this is a useful, and manageable intervention.

Acknowledgement

The PURLs Surveillance System is supported in part by Grant Number UL1RR024999 from the National Center for Research Resources, a Clinical Translational Science Award to the University of Chicago. The content is solely the responsibility of the authors and does not necessarily represent the official views of the National Center for Research Resources or the National Institutes of Health.

PRACTICE CHANGER

Express your concern about long-term use of benzodiazepines in a letter—a simple intervention that patients often respond to by reducing or eliminating their use of the drug.1

STRENGTH OF RECOMMENDATION

A: Based on a well-done meta-analysis with few clinical trials.

Mugunthan K, McGuire T, Glasziou P. Minimal interventions to decrease long-term use of benzodiazepines in primary care: a systematic review and meta-analysis. Br J Gen Pract. 2011;61:e573-e578.

ILLUSTRATIVE CASE

A 65-year-old patient has been taking lorazepam for insomnia for more than a year. You are concerned about her ongoing use of the benzodiazepine and want to wean her from the medication. What strategies can you use to decrease, or eliminate, her use of the drug?

Benzodiazepines are commonly used medications, with an estimated 12-month prevalence of use of 8.6% in the United States.2 While short-term use of these antianxiety medications can be effective, long-term use (defined as regular use for >3 months) is associated with significant risk.

Abuse linked to chronic use
Prescription drug abuse has recently become the nation’s leading cause of accidental death, overtaking motor vehicle accidents.3 And tranquilizers, including benzodiazepines, are the second most abused prescription medication, after pain relievers.4 In addition to dependence and withdrawal, chronic use of benzodiazepines is associated with daytime somnolence, blunted reflexes, memory loss, cognitive impairment, and an increased risk of falls and fractures—particularly in older patients.5

Reducing long-term use of benzodiazepines in a primary care setting is important but challenging. Until recently, most of the successful strategies reported were resource intensive and required multiple office visits.6

STUDY SUMMARY: Brief interventions are often effective

This study was a meta-analysis of randomized controlled trials in which “minimal interventions” were compared with usual care for their effectiveness in reducing or eliminating benzodiazepine use in primary care patients. A minimal intervention was defined as a letter, self-help information, or short consultation with a primary care provider. In each case, the message to the patient included (a) an expression of concern about the patient’s long-term use of the medication, (b) information about the potential adverse effects of the medication, and (c) advice on how to gradually reduce or stop using it.

Three studies met the inclusion criteria for randomization, blinding, and analysis by intention-to-treat.7-9 All 3 (n=615) had a 6-month follow-up period, a higher proportion of women (>60%), and participants with a mean age >60 years. Few patients were lost to follow-up; withdrawal rates were low and similar in all 3 studies. Each study compared a letter with usual care; 2 of the 3 had a third arm that included both a letter and a short consultation.

Pooled results from the studies showed twice the reduction in benzodiazepine use in the intervention groups compared with the control groups (risk ratio [RR]=2.04; 95% confidence interval [CI], 1.5-2.8; P< .001). The RR for cessation of benzodiazepine use was 2.3 (95% CI, 1.3-4.2; P= .003). The number needed to treat for a reduction or cessation of use was 12. The studies reported benzodiazepine reduction rates of 20% to 35% in the intervention groups vs 6% to 15% in the usual care groups.7-9 There appeared to be no additional benefit to adding the brief consultation compared with the letter alone.

WHAT’S NEW?: This strategy is easy to implement

While many methods to reduce benzodiazepine use have been studied, most involved levels of skill and resources that are not feasible for widespread use. This study found that a letter, stating the risks of continued use of the medication and providing a weaning schedule and tips for handling withdrawal, can be effective in reducing chronic use in a small but significant part of the population.

CAVEATS: Effects of withdrawal went unaddressed

The study did not adequately address the adverse effects of withdrawal from benzodiazepines, with one of the studies reporting significantly worse qualitative (but not quantitative) withdrawal symptoms at 6 months.7 This is of particular concern, as withdrawal symptoms are associated with the potential for relapse and concomitant abuse of other drugs and alcohol. We recommend that primary care physicians screen for substance abuse prior to the intervention and arrange for adequate follow-up.

All 3 studies in the meta-analysis lasted 6 months; no longer-term outcomes were reported. In addition, the study did not yield enough information to identify patients who would be most likely to respond to this brief intervention.

CHALLENGES TO IMPLEMENTATION: Determining which patients to target

 

 

Identifying patients who are appropriate candidates for this brief intervention and providing adequate monitoring for adverse effects of withdrawal are the main challenges of this practice changer. Nonetheless, chronic benzodiazepine use is of considerable concern, and we believe that this is a useful, and manageable intervention.

Acknowledgement

The PURLs Surveillance System is supported in part by Grant Number UL1RR024999 from the National Center for Research Resources, a Clinical Translational Science Award to the University of Chicago. The content is solely the responsibility of the authors and does not necessarily represent the official views of the National Center for Research Resources or the National Institutes of Health.

References

1. Mugunthan K, McGuire T, Glasziou P. Minimal interventions to decrease long-term use of benzodiazepines in primary care: a systematic review and meta-analysis. Br J Gen Pract. 2011;61:e573-e578.

2. Tyrer PJ. Benzodiazepines on trial. Br Med J. 1984;288:1101-1102.

3. Centers for Disease Control and Prevention. Deaths: Leading causes for 2008. June 6, 2012. Available at: http://www.cdc.gov/nchs/data/nvsr/nvsr60/nvsr60_06.pdf. Accessed October 10, 2012.

4. National Institute on Drug Abuse. Topics in brief: Prescription drug abuse. Available at: http://www.drugabuse.gov/publications/topics-in-brief/prescription-drug-abuse. Accessed October 11, 2012.

5. Morin CM, Bastien C, Guay B, et al. Randomized clinical trail of supervised tapering and cognitive behavior therapy to facilitate benzodiazepine discontinuation in older adults with chronic insomnia. Am J Psychiatry. 2004;161:332-342.

6. Oude Voshaar RC, Couvee JE, van Balkorn AJ, et al. Strategies for discontinuing long-term benzodiazepine use-meta-analysis. Br J Psychiatr. 2006;189:213-220.

7. Bashir K, King M, Ashworth M. Controlled evaluation of brief intervention by general practitioners to reduce chronic use of benzodiazepines. Br J Gen Pract. 1994;44:408-412.

8. Cormack MA, Sweeney KG, Hughes-Jones H, et al. Evaluation of an easy, cost-effective strategy to cut benzodiazepine use in general practice. Br J Gen Pract. 1994;44:5-8

9. Heather NA, Bowie A, Ashton H, et al. Randomized controlled trial of two brief interventions against long-term benzodiazepine use: outcome of intervention. Addict Res Theory. 2004;12:141-145.

References

1. Mugunthan K, McGuire T, Glasziou P. Minimal interventions to decrease long-term use of benzodiazepines in primary care: a systematic review and meta-analysis. Br J Gen Pract. 2011;61:e573-e578.

2. Tyrer PJ. Benzodiazepines on trial. Br Med J. 1984;288:1101-1102.

3. Centers for Disease Control and Prevention. Deaths: Leading causes for 2008. June 6, 2012. Available at: http://www.cdc.gov/nchs/data/nvsr/nvsr60/nvsr60_06.pdf. Accessed October 10, 2012.

4. National Institute on Drug Abuse. Topics in brief: Prescription drug abuse. Available at: http://www.drugabuse.gov/publications/topics-in-brief/prescription-drug-abuse. Accessed October 11, 2012.

5. Morin CM, Bastien C, Guay B, et al. Randomized clinical trail of supervised tapering and cognitive behavior therapy to facilitate benzodiazepine discontinuation in older adults with chronic insomnia. Am J Psychiatry. 2004;161:332-342.

6. Oude Voshaar RC, Couvee JE, van Balkorn AJ, et al. Strategies for discontinuing long-term benzodiazepine use-meta-analysis. Br J Psychiatr. 2006;189:213-220.

7. Bashir K, King M, Ashworth M. Controlled evaluation of brief intervention by general practitioners to reduce chronic use of benzodiazepines. Br J Gen Pract. 1994;44:408-412.

8. Cormack MA, Sweeney KG, Hughes-Jones H, et al. Evaluation of an easy, cost-effective strategy to cut benzodiazepine use in general practice. Br J Gen Pract. 1994;44:5-8

9. Heather NA, Bowie A, Ashton H, et al. Randomized controlled trial of two brief interventions against long-term benzodiazepine use: outcome of intervention. Addict Res Theory. 2004;12:141-145.

Issue
The Journal of Family Practice - 61(11)
Issue
The Journal of Family Practice - 61(11)
Page Number
671-672
Page Number
671-672
Publications
Publications
Topics
Article Type
Display Headline
Patient overusing antianxiety meds? Say so (in a letter)
Display Headline
Patient overusing antianxiety meds? Say so (in a letter)
Legacy Keywords
Robert Levy;MD; Shailendra Prasad;MBBS;MPH; benzodiazepines; minimal interventions; inclusion criteria; primary care patients; short consultation; letter; withdrawal rates
Legacy Keywords
Robert Levy;MD; Shailendra Prasad;MBBS;MPH; benzodiazepines; minimal interventions; inclusion criteria; primary care patients; short consultation; letter; withdrawal rates
Sections
Article Source

PURLs Copyright

Copyright © 2012 The Family Physicians Inquiries Network. All rights reserved.

Inside the Article

Article PDF Media